RES: [obm-l] Problema Interessante de Geometria

2015-06-09 Por tôpico Albert Bouskela
Olá, Ralph,

 

O arquivo GeoGebra (“Hexagons.ggb”) foi bloqueado pelo sistema que administra 
esta Lista, em face da possibilidade de vírus (por tratar-se de um arquivo 
executável).

 

Peço, então, que envie o respectivo arquivo diretamente para o meu e-mail.

 

Prometo (como sempre…) tentar encontrar uma solução ainda mais complicada do 
que as já disponíveis na literatura e (para compensar!) válida somente para uns 
poucos casos particulares.

 

Sds.,

  _  

Albert Bouskelá

 mailto:bousk...@gmail.com bousk...@gmail.com

 

De: owner-ob...@mat.puc-rio.br [mailto:owner-ob...@mat.puc-rio.br] Em nome de 
Ralph Teixeira
Enviada em: segunda-feira, 8 de junho de 2015 21:03
Para: obm-l@mat.puc-rio.br
Assunto: [obm-l] {Filename?} Problema Interessante de Geometria

 

Ola a todos.

 

Eu e minha aluna de Mestrado Fabiola encontramos um problema bem facil de 
enunciar que esclareceria um ponto da dissertacao de mestrado dela... No 
entanto, a gente soh encontrou umas solucoes bem complicadas na literatura, e 
mesmo assim parecem ser apenas para alguns casos particulares simetricos... 
Entao coloco aqui -- quem tiver uma solucao elegante ganha um agradecimento na 
dissertacao! :) :)

 

(Eu pensei ateh em sugerir esse problema para alguma OBM, mas como ainda nao 
sei resolver e acabei mostrando a alguns alunos, vou soltar logo ele aqui.)

 

Sao dados dois poligonos convexos P1P2...Pn e Q1Q2...Qn (onde n4) contendo a 
origem O em seu interior. Sabe-se que:

-- Eles tem lados respectivamente paralelos (isto eh, PiP_{i+1} // QiQ_{i+1} 
para i=1,2,...,n, indices modulo n);

-- Triangulos com vertice em O e um lado do poligono tem areas respectivamente 
iguais (isto eh, Area(OPiP_{i+1}) = Area(OQiQ_{i+1}) para i=1,2,...n, indices 
modulo n).

Pergunta-se: os poligonos tem que ser congruentes?

 

Quem quiser brincar, vide o Geogebra anexo que ilustra o caso n=6 (fiz uma 
copia de Q longe da origem para facilitar a visualizacao -- a origem para Q 
eh O_1). Pode brincar como quiser com os Q's, e com P_1 -- os outros pontos sao 
calculados para satisfazer as condicoes acima... Mas alguem consegue fazer o 
poligono P fechar (isto eh, P1=P7) sem que ele seja congruente ao Q (mas 
mantendo ambos convexos e mantendo a origem O dentro de P?)

 

Nota: se n=4, dois paralelogramos distintos de mesma area centrados na origem 
sao contra-exemplo!

 

Abraco, Ralph.


-- 
Esta mensagem foi verificada pelo sistema de antiv�rus e
 acredita-se estar livre de perigo.



[obm-l] Re: [obm-l] RES: [obm-l] Re: [obm-l] Re: [obm-l] RES: [obm-l] Problema da Olimpíada de Matemática de Moscou

2014-09-05 Por tôpico Mauricio de Araujo
é o antigo noves fora que minha mãe usava...


2014-09-04 21:25 GMT-03:00 Albert Bouskela bousk...@ymail.com:

 Olá!



 Pois é! Problemas (equações) que envolvem um determinado número (natural)
 e a soma dos algarismos que o compõem, geralmente, são resolvidos através
 da propriedade mencionada pelo Ralph:



 S(x) = x (mod. 9)



 Ou, o que dá no mesmo, mas as vezes pode ser mais útil:



 “x” e S(x) deixam o mesmo resto na divisão por 9.



 Exemplo:



 Mostre que a soma da soma da soma (3 vezes) dos algarismos de 50^50 e
 770^770 são iguais.



 

 Albert Bouskelá

 bousk...@ymail.com



 *De:* owner-ob...@mat.puc-rio.br [mailto:owner-ob...@mat.puc-rio.br] *Em
 nome de *Ralph Teixeira
 *Enviada em:* quarta-feira, 3 de setembro de 2014 21:27
 *Para:* obm-l@mat.puc-rio.br
 *Assunto:* [obm-l] Re: [obm-l] Re: [obm-l] RES: [obm-l] Problema da
 Olimpíada de Matemática de Moscou



 Ah, eh verdade, dah para acelerar MUITO notando que:



 S(x) = x (mod 9)



 Entao x+S(x)+S(S(x)) = 3x (mod 9)



 Isto eh, x+S(x)+S(S(x)) eh sempre divisivel por 3 -- e portanto nunca pode
 ser 1993.



 Abraco,

  Ralph



 2014-09-03 19:42 GMT-03:00 Mauricio de Araujo 
 mauricio.de.ara...@gmail.com:

 não tem solução!! hehehe



 2014-09-03 19:07 GMT-03:00 Albert Bouskela bousk...@ymail.com:

 Olá!



 A melhor solução é pelo “cheiro”



 1) x1899 | 1899+S(1899)+SS(1899)=19351993

 2) x1959 | 1959+S(1959)+SS(1959)=19891993

 3) S≥16 (x=1960) e SS≥2 (S=20)

 4) x=1993-16-2=1975

 5) 1960=x=1975

 6) Agora é no braço…

 7) Mas há uma surpresa no final!



 

 Albert Bouskelá

 bousk...@ymail.com

 *De:* owner-ob...@mat.puc-rio.br [mailto:owner-ob...@mat.puc-rio.br] *Em
 nome de *Mauricio de Araujo
 *Enviada em:* quarta-feira, 3 de setembro de 2014 11:36
 *Para:* obm-l@mat.puc-rio.br
 *Assunto:* [obm-l] Problema da Olimpíada de Matemática de Moscou



 Seja S(x) a soma dos dígitos de um inteiro positivo x.



 Resolver: x + S(x) + S(S(x)) = 1993.



 --

 Abraços


 oɾnɐɹɐ ǝp oıɔıɹnɐɯ

 --
 Esta mensagem foi verificada pelo sistema de antivírus e
 acredita-se estar livre de perigo.




-- 
Abraços

oɾnɐɹɐ ǝp oıɔıɹnɐɯ

-- 
Esta mensagem foi verificada pelo sistema de antiv�rus e
 acredita-se estar livre de perigo.



[obm-l] Re: [obm-l] RES: [obm-l] Re: [obm-l] Re: [obm-l] RES: [obm-l] Problema da Olimpíada de Matemática de Moscou

2014-09-05 Por tôpico Douglas Oliveira de Lima
Existe uma questão muito legal que acabei de fazer desse mesmo assunto,
caso seja do seu interesse praticar ai vai.
A soma dos algarismos de um numero n vale 100, e a soma dos digitos do
numero 44n vale 800, Calcular a soma dos digitos de 3n.
Douglas Oliveira


Em 4 de setembro de 2014 21:25, Albert Bouskela bousk...@ymail.com
escreveu:

 Olá!



 Pois é! Problemas (equações) que envolvem um determinado número (natural)
 e a soma dos algarismos que o compõem, geralmente, são resolvidos através
 da propriedade mencionada pelo Ralph:



 S(x) = x (mod. 9)



 Ou, o que dá no mesmo, mas as vezes pode ser mais útil:



 “x” e S(x) deixam o mesmo resto na divisão por 9.



 Exemplo:



 Mostre que a soma da soma da soma (3 vezes) dos algarismos de 50^50 e
 770^770 são iguais.



 

 Albert Bouskelá

 bousk...@ymail.com



 *De:* owner-ob...@mat.puc-rio.br [mailto:owner-ob...@mat.puc-rio.br] *Em
 nome de *Ralph Teixeira
 *Enviada em:* quarta-feira, 3 de setembro de 2014 21:27
 *Para:* obm-l@mat.puc-rio.br
 *Assunto:* [obm-l] Re: [obm-l] Re: [obm-l] RES: [obm-l] Problema da
 Olimpíada de Matemática de Moscou



 Ah, eh verdade, dah para acelerar MUITO notando que:



 S(x) = x (mod 9)



 Entao x+S(x)+S(S(x)) = 3x (mod 9)



 Isto eh, x+S(x)+S(S(x)) eh sempre divisivel por 3 -- e portanto nunca pode
 ser 1993.



 Abraco,

  Ralph



 2014-09-03 19:42 GMT-03:00 Mauricio de Araujo 
 mauricio.de.ara...@gmail.com:

 não tem solução!! hehehe



 2014-09-03 19:07 GMT-03:00 Albert Bouskela bousk...@ymail.com:

 Olá!



 A melhor solução é pelo “cheiro”



 1) x1899 | 1899+S(1899)+SS(1899)=19351993

 2) x1959 | 1959+S(1959)+SS(1959)=19891993

 3) S≥16 (x=1960) e SS≥2 (S=20)

 4) x=1993-16-2=1975

 5) 1960=x=1975

 6) Agora é no braço…

 7) Mas há uma surpresa no final!



 

 Albert Bouskelá

 bousk...@ymail.com

 *De:* owner-ob...@mat.puc-rio.br [mailto:owner-ob...@mat.puc-rio.br] *Em
 nome de *Mauricio de Araujo
 *Enviada em:* quarta-feira, 3 de setembro de 2014 11:36
 *Para:* obm-l@mat.puc-rio.br
 *Assunto:* [obm-l] Problema da Olimpíada de Matemática de Moscou



 Seja S(x) a soma dos dígitos de um inteiro positivo x.



 Resolver: x + S(x) + S(S(x)) = 1993.



 --

 Abraços


 oɾnɐɹɐ ǝp oıɔıɹnɐɯ

 --
 Esta mensagem foi verificada pelo sistema de antivírus e
 acredita-se estar livre de perigo.


-- 
Esta mensagem foi verificada pelo sistema de antiv�rus e
 acredita-se estar livre de perigo.



[obm-l] Re: [obm-l] RES: [obm-l] Re: [obm-l] Re: [obm-l] RES: [obm-l] Problema da Olimpíada de Matemática de Moscou

2014-09-05 Por tôpico Pedro José
Boa tarde!

O número de algarismos x não nulos de 50^50 é igual ao número de
algarismos de 5^50

seja t = [x], t Ɛ e  x - 1  t  = x

x = [50*log(5)]+1 = 35 == S (50^50) = 9*35 = 315 == S(S(50^50)) = 2+9
+9 = 13 == S(S(S50^50) 10 ==

== S(S(50^50)) só tem um algarismo

S(S(S(50^50))) ≡ 3* (50^50) mod 9

50^50 ≡ 5^50 * 10^50 ≡  5^50 mod 9 pois, 10 ≡ 1 mod9

5^50 ≡ (5^6)^8 * 5^2 ≡ 7 mod 9 (pois, 5^6 ≡  1 mod 9)  == S(S(S(50^50)))
≡  3* 7 ≡ 3 mod9. Como  0= S(S(S(50^50))) 10 ==

==  S(S(S(50^50))) = 3 (i)

O número de algarismos y  não nulos de 770^770 é gual ao número de
algarismos de 77^770

y =  [770* log (77)] + 1 == y = 1453 == S(770^770) = 13077 (9*1453)  ==
S(S(770^770)) = 30 (1 + 2 + 9 + 9 + 9) ==

S(S(S(770^770))) = 11 (2+9)

Se S(S(S(770^770))) possui dois algarismos 770^770 ≡ 1 mod 9 ou 770^770 ≡ 2
mod 9

770^770 ≡ 77^770 * 10^770 ≡  77^770 mod 9 pois 10 ≡ 1 mod9

77^770 ≡ (77^6)^128 * 7^2 ≡ 4 mod9  (pois, 5^6 ≡  1 mod 9) ==
S(S(S(770^770))) só possui um algarismo == 0= S(S(S(770^770))) 10

== S(S(S(770^770))) 3* 4 ≡ 3 mod9. Como  0= S(S(S(770^770)))10 ==
S(S(S(770^770))) = 3 (ii)

(i) e (ii) == S(S(S(50^50))) = S(S(S(770^770)))

Saudações,
PJMS.



Em 4 de setembro de 2014 21:25, Albert Bouskela bousk...@ymail.com
escreveu:

 Olá!



 Pois é! Problemas (equações) que envolvem um determinado número (natural)
 e a soma dos algarismos que o compõem, geralmente, são resolvidos através
 da propriedade mencionada pelo Ralph:



 S(x) = x (mod. 9)



 Ou, o que dá no mesmo, mas as vezes pode ser mais útil:



 “x” e S(x) deixam o mesmo resto na divisão por 9.



 Exemplo:



 Mostre que a soma da soma da soma (3 vezes) dos algarismos de 50^50 e
 770^770 são iguais.



 

 Albert Bouskelá

 bousk...@ymail.com



 *De:* owner-ob...@mat.puc-rio.br [mailto:owner-ob...@mat.puc-rio.br] *Em
 nome de *Ralph Teixeira
 *Enviada em:* quarta-feira, 3 de setembro de 2014 21:27
 *Para:* obm-l@mat.puc-rio.br
 *Assunto:* [obm-l] Re: [obm-l] Re: [obm-l] RES: [obm-l] Problema da
 Olimpíada de Matemática de Moscou



 Ah, eh verdade, dah para acelerar MUITO notando que:



 S(x) = x (mod 9)



 Entao x+S(x)+S(S(x)) = 3x (mod 9)



 Isto eh, x+S(x)+S(S(x)) eh sempre divisivel por 3 -- e portanto nunca pode
 ser 1993.



 Abraco,

  Ralph



 2014-09-03 19:42 GMT-03:00 Mauricio de Araujo 
 mauricio.de.ara...@gmail.com:

 não tem solução!! hehehe



 2014-09-03 19:07 GMT-03:00 Albert Bouskela bousk...@ymail.com:

 Olá!



 A melhor solução é pelo “cheiro”



 1) x1899 | 1899+S(1899)+SS(1899)=19351993

 2) x1959 | 1959+S(1959)+SS(1959)=19891993

 3) S≥16 (x=1960) e SS≥2 (S=20)

 4) x=1993-16-2=1975

 5) 1960=x=1975

 6) Agora é no braço…

 7) Mas há uma surpresa no final!



 

 Albert Bouskelá

 bousk...@ymail.com

 *De:* owner-ob...@mat.puc-rio.br [mailto:owner-ob...@mat.puc-rio.br] *Em
 nome de *Mauricio de Araujo
 *Enviada em:* quarta-feira, 3 de setembro de 2014 11:36
 *Para:* obm-l@mat.puc-rio.br
 *Assunto:* [obm-l] Problema da Olimpíada de Matemática de Moscou



 Seja S(x) a soma dos dígitos de um inteiro positivo x.



 Resolver: x + S(x) + S(S(x)) = 1993.



 --

 Abraços


 oɾnɐɹɐ ǝp oıɔıɹnɐɯ

 --
 Esta mensagem foi verificada pelo sistema de antivírus e
 acredita-se estar livre de perigo.


-- 
Esta mensagem foi verificada pelo sistema de antiv�rus e
 acredita-se estar livre de perigo.



[obm-l] RES: [obm-l] Problema da Olimpiada de Matemática de Moscou

2014-09-03 Por tôpico Albert Bouskela
Olá!

 

A melhor solução é pelo “cheiro”

 

1) x1899 | 1899+S(1899)+SS(1899)=19351993

2) x1959 | 1959+S(1959)+SS(1959)=19891993

3) S≥16 (x=1960) e SS≥2 (S=20)

4) x≤1993-16-2=1975

5) 1960≤x≤1975

6) Agora é no braço…

7) Mas há uma surpresa no final!

 

  _  

Albert Bouskelá

 mailto:bousk...@ymail.com bousk...@ymail.com

 

De: owner-ob...@mat.puc-rio.br [mailto:owner-ob...@mat.puc-rio.br] Em nome de 
Mauricio de Araujo
Enviada em: quarta-feira, 3 de setembro de 2014 11:36
Para: obm-l@mat.puc-rio.br
Assunto: [obm-l] Problema da Olimpiada de Matemática de Moscou

 

Seja S(x) a soma dos dígitos de um inteiro positivo x.

 

Resolver: x + S(x) + S(S(x)) = 1993.

 

-- 

Abraços


oɾnɐɹɐ ǝp oıɔıɹnɐɯ

 


-- 
Esta mensagem foi verificada pelo sistema de antiv�s e 
acredita-se estar livre de perigo. 


-- 
Esta mensagem foi verificada pelo sistema de antiv�rus e
 acredita-se estar livre de perigo.



[obm-l] Re: [obm-l] RES: [obm-l] Problema da Olimpiada de Matemática de Moscou

2014-09-03 Por tôpico Mauricio de Araujo
não tem solução!! hehehe


2014-09-03 19:07 GMT-03:00 Albert Bouskela bousk...@ymail.com:

 Olá!



 A melhor solução é pelo “cheiro”



 1) x1899 | 1899+S(1899)+SS(1899)=19351993

 2) x1959 | 1959+S(1959)+SS(1959)=19891993

 3) S≥16 (x=1960) e SS≥2 (S=20)

 4) x≤1993-16-2=1975

 5) 1960≤x≤1975

 6) Agora é no braço…

 7) Mas há uma surpresa no final!


 --

 *Albert Bouskelá*

 bousk...@ymail.com



 *De:* owner-ob...@mat.puc-rio.br [mailto:owner-ob...@mat.puc-rio.br] *Em
 nome de *Mauricio de Araujo
 *Enviada em:* quarta-feira, 3 de setembro de 2014 11:36
 *Para:* obm-l@mat.puc-rio.br
 *Assunto:* [obm-l] Problema da Olimpiada de Matemática de Moscou



 Seja S(x) a soma dos dígitos de um inteiro positivo x.



 Resolver: x + S(x) + S(S(x)) = 1993.



 --

 Abraços


 oɾnɐɹɐ ǝp oıɔıɹnɐɯ




 --
 Esta mensagem foi verificada pelo sistema de antiv?s e
 acredita-se estar livre de perigo.


 --
 Esta mensagem foi verificada pelo sistema de antivírus e
 acredita-se estar livre de perigo.




-- 
Abraços

oɾnɐɹɐ ǝp oıɔıɹnɐɯ

-- 
Esta mensagem foi verificada pelo sistema de antiv�rus e
 acredita-se estar livre de perigo.



[obm-l] Re: [obm-l] Re: [obm-l] RES: [obm-l] Problema da Olimpiada de Matemática de Moscou

2014-09-03 Por tôpico Ralph Teixeira
Ah, eh verdade, dah para acelerar MUITO notando que:

S(x) = x (mod 9)

Entao x+S(x)+S(S(x)) = 3x (mod 9)

Isto eh, x+S(x)+S(S(x)) eh sempre divisivel por 3 -- e portanto nunca pode
ser 1993.

Abraco,
 Ralph


2014-09-03 19:42 GMT-03:00 Mauricio de Araujo mauricio.de.ara...@gmail.com
:

 não tem solução!! hehehe


 2014-09-03 19:07 GMT-03:00 Albert Bouskela bousk...@ymail.com:

 Olá!



 A melhor solução é pelo “cheiro”



 1) x1899 | 1899+S(1899)+SS(1899)=19351993

 2) x1959 | 1959+S(1959)+SS(1959)=19891993

 3) S≥16 (x=1960) e SS≥2 (S=20)

 4) x≤1993-16-2=1975

 5) 1960≤x≤1975

 6) Agora é no braço…

 7) Mas há uma surpresa no final!


 --

 *Albert Bouskelá*

 bousk...@ymail.com



 *De:* owner-ob...@mat.puc-rio.br [mailto:owner-ob...@mat.puc-rio.br] *Em
 nome de *Mauricio de Araujo
 *Enviada em:* quarta-feira, 3 de setembro de 2014 11:36
 *Para:* obm-l@mat.puc-rio.br
 *Assunto:* [obm-l] Problema da Olimpiada de Matemática de Moscou



 Seja S(x) a soma dos dígitos de um inteiro positivo x.



 Resolver: x + S(x) + S(S(x)) = 1993.



 --

 Abraços


 oɾnɐɹɐ ǝp oıɔıɹnɐɯ




 --
 Esta mensagem foi verificada pelo sistema de antiv?s e
 acredita-se estar livre de perigo.


 --
 Esta mensagem foi verificada pelo sistema de antivírus e
 acredita-se estar livre de perigo.




 --
 Abraços

 oɾnɐɹɐ ǝp oıɔıɹnɐɯ


 --
 Esta mensagem foi verificada pelo sistema de antivírus e
 acredita-se estar livre de perigo.

-- 
Esta mensagem foi verificada pelo sistema de antiv�rus e
 acredita-se estar livre de perigo.



RES: [obm-l] Problema do Cavalo

2014-02-19 Por tôpico Benedito
OK Bernado.
Vou dar uma olhada.
Obrigado.
Benedito

-Mensagem original-
De: owner-ob...@mat.puc-rio.br [mailto:owner-ob...@mat.puc-rio.br] Em nome
de Bernardo Freitas Paulo da Costa
Enviada em: terça-feira, 18 de fevereiro de 2014 18:00
Para: Lista de E-mails da OBM
Assunto: Re: [obm-l] Problema do Cavalo

2014-02-18 14:30 GMT-03:00 Benedito bened...@ufrnet.br:

 É infinito nos quatro quadrantes, que é para permitir muitos movimentos.

 De: owner-ob...@mat.puc-rio.br [mailto:owner-ob...@mat.puc-rio.br] Em
 nome de terence thirteen Enviada em: segunda-feira, 17 de fevereiro de
 2014 08:16
 Para: obm-l
 Assunto: Re: [obm-l] Problema do Cavalo

 Ele é infinito nos quatro quadrantes?

 Eu tentaria algo como construir um grafo infinito, mas vou pensar antes...

Eu tenho uma idéia de solução no braço. Supondo que a questão seja:
Qual é o número de casas diferentes em que um cavalo pode terminar uma
seqüência de N movimentos. Assim, para n = 1, temos 8 casas (brancas), e
para n = 2 temos 33 casas (pretas, incluindo a casa preta original!).

Para n maior, a seqüência fica assim (feito num computador, na marra):

8; 33; 76; 129; 196; 277; 372; 481; 604; 741; 892; 1057; 1236; 1429; ...

Agora, vem o chute principal (que é o que vai ajudar a gente a fazer
indução): Calcule as diferenças sucessivas dos elementos! Isso dá:

25; 43; 53; 67; 81; 95; 109; 123; 137; 151; 165; 179; 193; ...

Ainda não parece bom ? Não tem problema... Mais uma vez, faça as diferenças:

18; 10; 14; 14; 14; 14; 14; 14; 14; 14; 14; 14; ...

Ah ! Parece que é uma PA de segunda ordem, a partir de um certo
ponto...

Vamos entender essa idéia. No longo prazo, o cavalo vai se afastando do
centro, e portanto ele pode cobrir uma área no máximo proporcional a N^2.
Isso por si só já justifica tentar achar uma PA de segunda ordem. O que é
interessante é que a parte perto do centro (depois do início, onde ainda há
um monte de buracos meio aleatórios) estará completamente coberta depois de
um certo tempo, e o que interessa é o que acontece nas coroas. Agora, tem
que justificar que as coroas têm uma espessura constante depois de passada a
parte transiente
inicial.

Como eu usei um computador, e posso calcular mais do que n = 10 (por exemplo
n = 100) e os 14 continuam até esse ponto. Para mim, isso é mais do que
suficiente para eu ter certeza que a resposta é essa, mas admito que falta
um argumento garantindo que basta observar um número finito de passos para
acertar a recorrência. Eu diria que, como um cavalo completa a vizinhança
do ponto inicial (o 3x3 em volta da
origem) em uma quantidade finita de passos (basta chegar na profundidade 3
do grafo do Torres) a recorrência não pode ser de ordem muito maior do que
isso. Para melhorar, veja que a partir de 3 passos, o que temos é um
octógono, TODO preenchido, dos quadrados brancos (que são os únicos em que o
cavalo pode estar!). Daí pra frente, não é difícil ver que a cada etapa
teremos um octógono com lado aumentando de 1 a cada vez. Veja também que a
partir do 3o termo da segunda diferença, só tem 14. Não é coincidência.

Agora, eu deixo a indução para você completar!

Abraços,
--
Bernardo Freitas Paulo da Costa

--
Esta mensagem foi verificada pelo sistema de antivírus e  acredita-se estar
livre de perigo.


=
Instruções para entrar na lista, sair da lista e usar a lista em
http://www.mat.puc-rio.br/~obmlistas/obm-l.html
=


---
Este email está limpo de vírus e malwares porque a proteção do avast! Antivírus 
está ativa.
http://www.avast.com


-- 
Esta mensagem foi verificada pelo sistema de antivírus e
 acredita-se estar livre de perigo.


=
Instruções para entrar na lista, sair da lista e usar a lista em
http://www.mat.puc-rio.br/~obmlistas/obm-l.html
=


RES: [obm-l] Problema do Cavalo

2014-02-18 Por tôpico Benedito
É infinito nos quatro quadrantes, que é para permitir muitos movimentos.



De: owner-ob...@mat.puc-rio.br [mailto:owner-ob...@mat.puc-rio.br] Em nome de 
terence thirteen
Enviada em: segunda-feira, 17 de fevereiro de 2014 08:16
Para: obm-l
Assunto: Re: [obm-l] Problema do Cavalo



Ele é infinito nos quatro quadrantes?

Eu tentaria algo como construir um grafo infinito, mas vou pensar antes...





Em 10 de fevereiro de 2014 09:11, Benedito bened...@ufrnet.br 
mailto:bened...@ufrnet.br  escreveu:

Estou tentando uma solução para o problema seguinte, usando Indução. Alguém 
pode me ajudar?

Problema

Num tabuleiro infinito, um cavalo (peça do jogo de xadrez) está situado na 
origem, digamos numa casa preta, e começa a se movimentar.

No total, quantas casas possíveis o cavalo pode atingir depois de n movimentos?

Nota - O movimento de um cavalo no jogo de xadrez é em forma de L (formado por 
4 casas, a partir da casa em que se encontra)



  _


 http://www.avast.com/

Este email está limpo de vírus e malwares porque a proteção do avast! Antivírus 
http://www.avast.com/  está ativa.



--
Esta mensagem foi verificada pelo sistema de antivírus e
acredita-se estar livre de perigo.




--
/**/
神が祝福

Torres


--
Esta mensagem foi verificada pelo sistema de antiv�s e
acredita-se estar livre de perigo.



---
Este email está limpo de vírus e malwares porque a proteção do avast! Antivírus 
está ativa.
http://www.avast.com

-- 
Esta mensagem foi verificada pelo sistema de antiv�rus e
 acredita-se estar livre de perigo.



RES: [obm-l] problema

2013-12-13 Por tôpico Albert Bouskela
Olá!

 

Faça o gráfico das 2 funções [ f(x)=2^x; g(x)=x ] e você verá o que
acontece…

 

  _  

Albert Bouskela

 mailto:bousk...@ymail.com bousk...@ymail.com

 

De: owner-ob...@mat.puc-rio.br [mailto:owner-ob...@mat.puc-rio.br] Em nome
de saulo nilson
Enviada em: sexta-feira, 13 de dezembro de 2013 17:58
Para: obm-l@mat.puc-rio.br
Assunto: [obm-l] problema

 

encontre todas as soluçoes de 2^x=x


-- 
Esta mensagem foi verificada pelo sistema de antivírus e 
acredita-se estar livre de perigo. 


-- 
Esta mensagem foi verificada pelo sistema de antivírus e
 acredita-se estar livre de perigo.



RES: [obm-l] Problema 4 Cone Sul 1996

2013-04-30 Por tôpico EPVN
Se entendi seu argumento podemos trocar os primeiros 998 números pela média
dos primeiros 998 números. O enunciado claramente não permite essa operação.
Apenas um deles deve ser trocado pela média.
Sem querer abusar da sua bondade, poderia esclarecer esse ponto.
Abraço.
Osmundo.

-Mensagem original-
De: owner-ob...@mat.puc-rio.br [mailto:owner-ob...@mat.puc-rio.br] Em nome
de Carlos Yuzo Shine
Enviada em: terça-feira, 30 de abril de 2013 13:11
Para: obm-l@mat.puc-rio.br
Assunto: Re: [obm-l] Problema 4 Cone Sul 1996

O erro foi supor que na situação anterior os números na sequência ficariam
a,b,b,b,...,b.
 
Poderia muito bem ser, digamos, 997/998, 1, 1995/1996, 1995/1996, ...,
1995/1996.
 
Se você ainda quer pensar no problema, pare de ler aqui. Caso contrário,
continue.
 
O que você pode fazer para resolver o problema é fazer a média dos primeiros
998 números, obtendo 998 números iguais a 997/998 e depois fazer pares com
997/998 e 1 (fazendo a operação mais 998 vezes). Note que esse argumento
funciona para qualquer número composto no lugar do 1996.
 
E no caso em que trocamos 1996 por um primo p (um 0 e p-1 uns)? Aí não dá,
porque no final o denominador tem que p (todo mundo teria que ser igual a
(p-1)/p, já que a soma de todos nunca muda), e isso obrigaria a gente a, em
algum momento, dividir tudo por p, o que não é possível.
 
Mas e se a soma dos p números é múltiplo de p? Mais uma boa pergunta, não?
 
[]'s
Shine
 

From: EPVN barz...@dglnet.com.br
To: obm-l@mat.puc-rio.br 
Sent: Monday, April 22, 2013 11:57 AM
Subject: [obm-l] Problema 4 Cone Sul 1996




O enunciado é: 

A seqüência 0, 1, 1, 1,
... , 1 contém 1996 números, sendo o primeiro zero e todos os demais um. Se
escolhem
dois ou mais números da seqüência (mas não todos) e se sustitui um deles
pela
média aritmética dos números escolhidos, obtendo-se assim uma nova seqüência
de
1996 números.  
Provar que, com a
repetição desta operação, é possível obter uma seqüência na qual os 1996
números são iguais.  

NOTA: Não é necessário
escolher a mesma quantidade de números em cada operação. 

Um colega apresentou a seguinte argumentação: 

Se essa operação levasse a uma seqüência com todos os números
idênticos então no penúltimo estágio teríamos algo assim: 

a,b,b,b,..,b , com um único número diferente que
deve ser tornado igual aos demais com mais um passo. Bem, se tomarmos p
números
b e mais o número a, 
obteremos o número (a + pb)/ (p + 1 ), igualando a b teríamos
a=b. 
Parece que isso prova que esse penúltimo estágio nunca é
atingido e, portanto, o último também não. 

Se algum colega puder nos ajudar a esclarecer a situação
ficamos muito gratos. 

Um abraço. 

Osmundo Bragança.   

=
Instruções para entrar na lista, sair da lista e usar a lista em
http://www.mat.puc-rio.br/~obmlistas/obm-l.html
=


=
Instruções para entrar na lista, sair da lista e usar a lista em
http://www.mat.puc-rio.br/~obmlistas/obm-l.html
=


RES: [obm-l] Problema 4 Cone Sul 1996

2013-04-30 Por tôpico EPVN
Então há, de fato, um erro na tradução.
Isso, é claro, muda tudo.
Agora vamos trabalhar com essa versão e mais as suas perguntas.
Obrigado pela atenção.
Osmundo Bragança

-Mensagem original-
De: owner-ob...@mat.puc-rio.br [mailto:owner-ob...@mat.puc-rio.br] Em nome
de Carlos Yuzo Shine
Enviada em: terça-feira, 30 de abril de 2013 16:34
Para: obm-l@mat.puc-rio.br
Assunto: Re: [obm-l] Problema 4 Cone Sul 1996

Ah, no enunciado original trocamos cada um deles pela média aritmética
(talvez houve algum erro na hora de transcrever o problema para o site). Eu
sei porque eu fui nessa Cone Sul, e exatamente por isso eu nem li o
enunciado que foi enviado pela lista. A solução que postei foi a que dei na
prova.

[]'s
Shine


- Original Message -
From: Bernardo Freitas Paulo da Costa bernardo...@gmail.com
To: obm-l@mat.puc-rio.br
Cc: 
Sent: Tuesday, April 30, 2013 1:24 PM
Subject: Re: [obm-l] Problema 4 Cone Sul 1996

  Se escolhem
  dois ou mais números da seqüência (mas não todos) e se sustitui um deles
pela
  média aritmética dos números escolhidos, obtendo-se assim uma nova
seqüência de
  1996 números.
 
2013/4/30 Carlos Yuzo Shine cysh...@yahoo.com:
 já que a soma de todos nunca muda

Confesso que não entendi direito. Imagine que você escolhe os 4
primeiros números, 0, 1, 1, 1. Qual é o resultado da operação? Da
forma como o enunciado parece indicar, isso DEPENDE de uma OUTRA
escolha, a saber a do um deles a ser substituído por 3/4. Ou seja,
podemos ficar com 3/4, 1, 1, 1; ou 0, 3/4, 1, 1 (ou uma permutação). O
Shine parece argumentar que o resultado será 3/4, 3/4, 3/4, 3/4. Mas
isso só seria o caso se estivesse escrito .. se substitui CADA UM
deles ..., não?

-- 
Bernardo Freitas Paulo da Costa

=
Instruções para entrar na lista, sair da lista e usar a lista em
http://www.mat.puc-rio.br/~obmlistas/obm-l.html
=


=
Instruções para entrar na lista, sair da lista e usar a lista em
http://www.mat.puc-rio.br/~obmlistas/obm-l.html
=


=
Instruções para entrar na lista, sair da lista e usar a lista em
http://www.mat.puc-rio.br/~obmlistas/obm-l.html
=


RES: [obm-l] Problema

2013-03-22 Por tôpico Benedito
A idéia é usar Cálculo (Coordenadas Polares). Mas, fazer na região descrita
no problema eu acho mais interessante.

Benedito

 

De: owner-ob...@mat.puc-rio.br [mailto:owner-ob...@mat.puc-rio.br] Em nome
de João Maldonado
Enviada em: sexta-feira, 22 de março de 2013 17:13
Para: obm-l@mat.puc-rio.br
Assunto: RE: [obm-l] Problema

 

Eu consegui fazer para o caso geral (M e Q pode estar em qualquer região do
círculo, não apenas em regiões opostas determinadas por um diâmetro)
E a resolução ficou bem feia também (tive que usar cálculo)

*Sendo P1 um ponto a uma distância x fixa do centro do círculo, qual a
probabilidade de escolhermos outro ponto no círculo tal que a distância
entre P1 e P2 seja menor que um?
Podemos tracejar um círculo de raio 1 em torno de P1. A intersecção desse
círculo com o círculo original é a região dos pontos cuja a distância a P1 é
 1.
A área dessa região sobre a área do círculo simboliza a probabilidade de
escolhermos outro ponto P2 no círculo tal que a distância entre P1 e P2 seja
menor que um.
A área pode ser facilmente calculada por matemática básica

A/Atotal = 1/Pi (2 ArcCos[x/2] - x sqrt (1- (x/2)²))

O peso dessa probabilidade é proporcional à área que ela ocupa (temos
muito mais pontos a uma distância 1 do que a uma distância 1/2 por exemplo)
O peso vale 2 Pi x dx/Pi = 2 x dx
Integrando de 0 a 1

P = Integral[ 2 x dx/Pi (2 ArcCos[x/2] - x sqrt (1- (x/2)²))]  de 0 a 1

P = 58.6%

[]'s
João

  _  

From: bened...@ufrnet.br mailto:bened...@ufrnet.br 
To: obm-l@mat.puc-rio.br mailto:obm-l@mat.puc-rio.br 
Subject: [obm-l] Problema
Date: Fri, 22 Mar 2013 05:16:50 -0300

Problema

Dois pontos, M e Q, são escolhidos aleatoriamente num disco unitário, mas em
regiões opostas, determinadas por um diâmetro AB. 

Qual é a probabilidade de que a distância entre M e Q seja menor do que 1?



Re: RES: [obm-l] problema

2013-02-17 Por tôpico grego
Obrigado, grande mestre!
A coisa é, de fato, violenta.
Um abraço!
Grego




 De: Albert Bouskela bousk...@msn.com
Para: obm-l@mat.puc-rio.br 
Enviadas: Sábado, 16 de Fevereiro de 2013 0:54
Assunto: RES: [obm-l] problema
 

Olá!
 
Este é um problema da Teoria dos Números bastante conhecido. Acredito (a 
confirmar!) que não exista uma solução analítica – o jeito é fazer “no braço” 
(“brute force”).
 
Bem, na Internet, encontrei a solução abaixo (bastante “arrumadinha”): – 
http://www.prise2tete.fr/forum/viewtopic.php?id=6370 
 
Bonjour,
Je n'ai trouvé qu'une méthode empirique et dichotomique aboutissant à 14 
solutions.
Supposons 1  a ≤ b ≤ c ≤ d.
Alors 1/a + 1/b + 1/c + 1/d ≤ 4/a donc 1 ≤ 4/a donc a ≤ 4 (et a  1). D'où 3 
cas à analyser : a=4, a=3 et a=2.

1) a = 4
Alors 1/4 + 1/b + 1/c + 1/d = 1 ≤ 1/4 + 3/b donc 3/4 ≤ 3/b donc b ≤ 4.
Et comme b ≥ a, on voit que b=4. De la même façon c=4 et d=4.
Solution1 : 1/4 + 1/4 + 1/4 + 1/4 = 1

2) a = 3 
Alors 1/3 + 1/b + 1/c + 1/d = 1 ≤ 1/3 + 3/b donc 2/3 ≤ 3/b donc b ≤ 9/2.
Et comme b ≥ a, on voit que b=4 ou b=3.

2a) b = 4
Alors 1/3 + 1/4 + 1/c + 1/d = 7/12 + 1/c + 1/d = 1 ≤ 7/12 + 2/c donc 5/12 ≤ 2/c 
donc c ≤ 24/5.
Et comme c ≥ b, on voit que c=4
Solution2 : 1/3 + 1/4 + 1/4 + 1/6 = 1

2b) b = 3
Alors 1/3 + 1/3 + 1/c + 1/d = 2/3 + 1/c + 1/d = 1 ≤ 2/3 + 2/c donc 1/3 ≤ 2/c 
donc c ≤ 6.
Et comme c ≥ b, on voit que c=3, c=4, c=5 ou c=6.
Solution3 : 1/3 + 1/3 + 1/4 + 1/12 = 1
Solution4 : 1/3 + 1/3 + 1/6 + 1/6 = 1

3) a = 2
Alors 1/2 + 1/b + 1/c + 1/d = 1 ≤ 1/2 + 3/b donc 1/2 ≤ 3/b donc b ≤ 6.
Et comme b ≥ a, on voit que b=6, b=5, b=4 ou b=3 (b=2 ne marche pas).

3a) b = 6
Alors 1/2 + 1/6 + 1/c + 1/d = 2/3 + 1/c + 1/d = 1 ≤ 2/3 + 2/c donc 1/3 ≤ 2/c 
donc c ≤ 6.
Et comme c ≥ b, on voit que c=6 et par suite que d=6.
Solution5 : 1/2 + 1/6 + 1/6 + 1/6 = 1

3b) b = 5
Alors 1/2 + 1/5 + 1/c + 1/d = 7/10 + 1/c + 1/d = 1 ≤ 7/10 + 2/c donc 3/10 ≤ 2/c 
donc c ≤ 20/3.
Et comme c ≥ b, on voit que c=5 ou c=6.
Si c=5, alors 1/d = 1 - 1/2 - 1/5 - 1/5 = 1/10
Solution6 : 1/2 + 1/5 + 1/5 + 1/10 = 1
Si c= 6, alors 1/d = 1 - 1/2 - 1/5 - 1/6 = 4/30 = 2/15 non réductible

3c) b = 4
Alors 1/2 + 1/4 + 1/c + 1/d = 3/4 + 1/c + 1/d = 1 ≤ 3/4 + 2/c donc 1/4 ≤ 2/c 
donc c ≤ 8.
Et comme c ≥ b, on voit que c=8, c=7, c=6 ou c=5 (c=4 ne marche pas).
Si c=8, alors 1/d = 1 - 1/2 - 1/4 - 1/8 = 1/8
Solution7 : 1/2 + 1/4 + 1/8 + 1/8 = 1
Si c=7, alors 1/d = 1 - 1/2 - 1/4 - 1/7 = 3/28 non réductible
Si c=6, alors 1/d = 1 - 1/2 - 1/4 - 1/6 = 1/12
Solution8 : 1/2 + 1/4 + 1/6 + 1/12 = 1
Si c=5, alors 1/d = 1 - 1/2 - 1/4 - 1/5 = 1/20
Solution9 : 1/2 + 1/4 + 1/5 + 1/20 = 1

3d) b = 3
Alors 1/2 + 1/3 + 1/c + 1/d = 5/6 + 1/c + 1/d = 1 ≤ 5/6 + 2/c donc 1/6 ≤ 2/c 
donc c ≤ 12.
Et comme 1/2 + 1/3 + 1/c  1 donc 1/c  1/6 donc c  6, on voit que c=12, c= 
11, c=10, c= 9, c=8 ou c=7.
Si c=12, alors 1/d = 1 - 1/2 - 1/3 - 1/12 = 1/12
Solution10 : 1/2 + 1/3 + 1/12 + 1/12 = 1
Si c=11, alors 1/d = 1 - 1/2 - 1/3 - 1/11 = 5/66 non réductible
Si c=10, alors 1/d = 1 - 1/2 - 1/3 - 1/10 = 2/30 = 1/15
Solution11 : 1/2 + 1/3 + 1/10 + 1/15 = 1
Si c=9, alors 1/d = 1 - 1/2 - 1/3 - 1/9 = 1/18
Solution12 : 1/2 + 1/3 + 1/9 + 1/18 = 1
Si c=8, alors 1/d = 1 - 1/2 - 1/3 - 1/8 = 1/24
Solution13 : 1/2 + 1/3 + 1/8 + 1/24 = 1
Si c=7, alors 1/d = 1 - 1/2 - 1/3 - 1/7 = 1/42
Solution14 : 1/2 + 1/3 + 1/7 + 1/42 = 1

FIN de la démonstration *** ouf... ***



J'ai tellement besoin de temps pour ne rien faire, qu'il ne m'en reste plus 
assez pour travailler.
 



Albert Bouskela
bousk...@msn.com
 
De:owner-ob...@mat.puc-rio.br [mailto:owner-ob...@mat.puc-rio.br] Em nome de 
grego
Enviada em: sexta-feira, 15 de fevereiro de 2013 22:31
Para: obm-l@mat.puc-rio.br
Assunto: [obm-l] problema
 
Olá, companheiros!
Um aluno me perguntou o seguinte:
a =b=c=d
1/a+1/b+1/c+1/d=1
Quantas quádruplas ordenadas (a, b, c, d) de naturais satisfazem a igualdade?
Um abraço!
Grego

RES: [obm-l] problema

2013-02-15 Por tôpico Albert Bouskela
Olá!

 

Este é um problema da Teoria dos Números bastante conhecido. Acredito (a 
confirmar!) que não exista uma solução analítica – o jeito é fazer “no braço” 
(“brute force”).

 

Bem, na Internet, encontrei a solução abaixo (bastante “arrumadinha”): – 
http://www.prise2tete.fr/forum/viewtopic.php?id=6370 

 

Bonjour,
Je n'ai trouvé qu'une méthode empirique et dichotomique aboutissant à 14 
solutions.
Supposons 1  a ≤ b ≤ c ≤ d.
Alors 1/a + 1/b + 1/c + 1/d ≤ 4/a donc 1 ≤ 4/a donc a ≤ 4 (et a  1). D'où 3 
cas à analyser : a=4, a=3 et a=2.

1) a = 4
Alors 1/4 + 1/b + 1/c + 1/d = 1 ≤ 1/4 + 3/b donc 3/4 ≤ 3/b donc b ≤ 4.
Et comme b ≥ a, on voit que b=4. De la même façon c=4 et d=4.
Solution1 : 1/4 + 1/4 + 1/4 + 1/4 = 1

2) a = 3 
Alors 1/3 + 1/b + 1/c + 1/d = 1 ≤ 1/3 + 3/b donc 2/3 ≤ 3/b donc b ≤ 9/2.
Et comme b ≥ a, on voit que b=4 ou b=3.

2a) b = 4
Alors 1/3 + 1/4 + 1/c + 1/d = 7/12 + 1/c + 1/d = 1 ≤ 7/12 + 2/c donc 5/12 ≤ 2/c 
donc c ≤ 24/5.
Et comme c ≥ b, on voit que c=4
Solution2 : 1/3 + 1/4 + 1/4 + 1/6 = 1

2b) b = 3
Alors 1/3 + 1/3 + 1/c + 1/d = 2/3 + 1/c + 1/d = 1 ≤ 2/3 + 2/c donc 1/3 ≤ 2/c 
donc c ≤ 6.
Et comme c ≥ b, on voit que c=3, c=4, c=5 ou c=6.
Solution3 : 1/3 + 1/3 + 1/4 + 1/12 = 1
Solution4 : 1/3 + 1/3 + 1/6 + 1/6 = 1

3) a = 2
Alors 1/2 + 1/b + 1/c + 1/d = 1 ≤ 1/2 + 3/b donc 1/2 ≤ 3/b donc b ≤ 6.
Et comme b ≥ a, on voit que b=6, b=5, b=4 ou b=3 (b=2 ne marche pas).

3a) b = 6
Alors 1/2 + 1/6 + 1/c + 1/d = 2/3 + 1/c + 1/d = 1 ≤ 2/3 + 2/c donc 1/3 ≤ 2/c 
donc c ≤ 6.
Et comme c ≥ b, on voit que c=6 et par suite que d=6.
Solution5 : 1/2 + 1/6 + 1/6 + 1/6 = 1

3b) b = 5
Alors 1/2 + 1/5 + 1/c + 1/d = 7/10 + 1/c + 1/d = 1 ≤ 7/10 + 2/c donc 3/10 ≤ 2/c 
donc c ≤ 20/3.
Et comme c ≥ b, on voit que c=5 ou c=6.
Si c=5, alors 1/d = 1 - 1/2 - 1/5 - 1/5 = 1/10
Solution6 : 1/2 + 1/5 + 1/5 + 1/10 = 1
Si c= 6, alors 1/d = 1 - 1/2 - 1/5 - 1/6 = 4/30 = 2/15 non réductible

3c) b = 4
Alors 1/2 + 1/4 + 1/c + 1/d = 3/4 + 1/c + 1/d = 1 ≤ 3/4 + 2/c donc 1/4 ≤ 2/c 
donc c ≤ 8.
Et comme c ≥ b, on voit que c=8, c=7, c=6 ou c=5 (c=4 ne marche pas).
Si c=8, alors 1/d = 1 - 1/2 - 1/4 - 1/8 = 1/8
Solution7 : 1/2 + 1/4 + 1/8 + 1/8 = 1
Si c=7, alors 1/d = 1 - 1/2 - 1/4 - 1/7 = 3/28 non réductible
Si c=6, alors 1/d = 1 - 1/2 - 1/4 - 1/6 = 1/12
Solution8 : 1/2 + 1/4 + 1/6 + 1/12 = 1
Si c=5, alors 1/d = 1 - 1/2 - 1/4 - 1/5 = 1/20
Solution9 : 1/2 + 1/4 + 1/5 + 1/20 = 1

3d) b = 3
Alors 1/2 + 1/3 + 1/c + 1/d = 5/6 + 1/c + 1/d = 1 ≤ 5/6 + 2/c donc 1/6 ≤ 2/c 
donc c ≤ 12.
Et comme 1/2 + 1/3 + 1/c  1 donc 1/c  1/6 donc c  6, on voit que c=12, c= 
11, c=10, c= 9, c=8 ou c=7.
Si c=12, alors 1/d = 1 - 1/2 - 1/3 - 1/12 = 1/12
Solution10 : 1/2 + 1/3 + 1/12 + 1/12 = 1
Si c=11, alors 1/d = 1 - 1/2 - 1/3 - 1/11 = 5/66 non réductible
Si c=10, alors 1/d = 1 - 1/2 - 1/3 - 1/10 = 2/30 = 1/15
Solution11 : 1/2 + 1/3 + 1/10 + 1/15 = 1
Si c=9, alors 1/d = 1 - 1/2 - 1/3 - 1/9 = 1/18
Solution12 : 1/2 + 1/3 + 1/9 + 1/18 = 1
Si c=8, alors 1/d = 1 - 1/2 - 1/3 - 1/8 = 1/24
Solution13 : 1/2 + 1/3 + 1/8 + 1/24 = 1
Si c=7, alors 1/d = 1 - 1/2 - 1/3 - 1/7 = 1/42
Solution14 : 1/2 + 1/3 + 1/7 + 1/42 = 1

FIN de la démonstration *** ouf... ***

  _  

J'ai tellement besoin de temps pour ne rien faire, qu'il ne m'en reste plus 
assez pour travailler.

 

  _  

Albert Bouskela

 mailto:bousk...@msn.com bousk...@msn.com

 

De: owner-ob...@mat.puc-rio.br [mailto:owner-ob...@mat.puc-rio.br] Em nome de 
grego
Enviada em: sexta-feira, 15 de fevereiro de 2013 22:31
Para: obm-l@mat.puc-rio.br
Assunto: [obm-l] problema

 

Olá, companheiros!

Um aluno me perguntou o seguinte:

a =b=c=d

1/a+1/b+1/c+1/d=1

Quantas quádruplas ordenadas (a, b, c, d) de naturais satisfazem a igualdade?

Um abraço!

Grego



Res: [obm-l] Problema

2009-09-24 Por tôpico Joel Castro
usa soma de uma pa que deve sair a resposta





De: Paulo Barclay Ribeiro paulobarc...@yahoo.com.br
Para: obm-l@mat.puc-rio.br
Enviadas: Quinta-feira, 24 de Setembro de 2009 13:54:40
Assunto: [obm-l] Problema


Prezados, 

Peço uma ajuda (orientação)na resolução do seguinte problema:
Qual o valor da soma de todos os numeros naturais de três algarismos?

Desde  já agradeço a gentileza

Paulo Barclay  

Veja quais são os assuntos do momento no Yahoo! + Buscados: Top 10 - 
Celebridades - Música - Esportes


  

Veja quais são os assuntos do momento no Yahoo! +Buscados
http://br.maisbuscados.yahoo.com

Res: Res: [obm-l] problema interessante!!!

2009-05-04 Por tôpico Cleuber Eduardo
Bom, então vamos lá:
Fazendo o desenho e construindo o triangulo equilátero auxiliar que te falei 
vamos precisar provar que AD=BE. Mas ai é simples pois sendo P a instersecção 
da circunferência circunscrita aos triangulos BCD e ACE, então D^MC=60º, 
A^PC=120º, vemos que P está em BE e de também em AD. Agora aplica-se ptlomeu 
nos respectivos quadriláteros APCE,  BPCD temos as seguintes relações:PE=AP+CP, 
PD= PB+PC, temos que: AD=AP+CP+PB=AE. Agora apliquemos o lei dos cossenos no 
triangulo ABE, BE^2=AB^2+AE^2+AB*AE*3^1/2, como AD=AE, de forma análoga AE=AC. 
Temos: AD^2=AB^2+AC^2+AB*AC*3^1/2 

 




De: Márcio Pinheiro profmar...@yahoo.com.br
Para: obm-l@mat.puc-rio.br
Enviadas: Quinta-feira, 30 de Abril de 2009 12:33:47
Assunto: Re: Res: [obm-l] problema interessante!!!


De nada.
Fiquei curioso quanto à tua solução por Ptolomeu.
Qual é o ponto P?
Valeu, Cleuber.

--- Em qui, 30/4/09, Cleuber Eduardo cleubersa...@yahoo.com.br escreveu:

De: Cleuber Eduardo cleubersa...@yahoo.com.br
Assunto: Res: [obm-l] problema interessante!!!
Para: obm-l@mat.puc-rio.br
Cc: cleubersa...@yahoo.com.br
Data: Quinta-feira, 30 de Abril de 2009, 10:18


Valeu Márcio!!. Quando eu peguei esse problema a princípio  eu tratei o 
problema de uma forma parecida com a tua. Mas ontem eu percebi que se 
construíssemos  um triangulo equilátero auxiliar ACE e depois  ptlolomeu no 
quadriátero APCE e BPCD. E so no final usa-se a lei dos cossenos pra terminar. 

Obrigado





De: Márcio Pinheiro profmar...@yahoo.com.br
Para: obm-l@mat.puc-rio.br
Enviadas: Quinta-feira, 30 de Abril de 2009 8:39:51
Assunto: Re: [obm-l] problema interessante!!!


A idéia inicial pode ser expressar AD em função de AB = c e de AC = b (essa é a 
parte realmente enfadonha). Um caminho (não acessível a quem ainda não tem 
conhecimentos razoáveis de números complexos) é adotar um plano de Argand-Gauss 
em que A é a origem e os eixos contêm os catetos. Em um tal plano, sejam A (0, 
0), B (0, c) e C (b, 0), por exemplo. O vetor CB pode ser obtido por meio de 
uma rotação do vetor CD de pi/3 em torno de C. Lembrando que o vetor PQ, com P 
(m, n) e Q (p, q), pode ser associado biunivocamente tanto a P - Q (m - p, n - 
q) quanto ao complexo (m - p) + (n - q)i, i^2 = -1, conclui-se que:
(vetor CD)*(cospi/3 + isenpi/3) = (vetor CB), o que equivale a [(x - b) + 
iy]*[(1/2) + i(sqrt3)/2] = - b + ci, sendo D (x, y).
Da igualdade das partes real e imaginária, impõe-se que x = (b + csqrt3)/2 e y 
= (c + bsqrt3)/2 (espero não ter errado as contas, feitas de cabeça :D).
Finalmente, obtém-se que AD^2 = b^2 + c^2 + bcsqrt3, através da distância entre 
os pontos A e D. Supondo que b e c sejam racionais, conclui-se que b^2, c^2 e 
bc também o 
são. Logo AD^2 seria irracional. Mas, caso AD fosse racional, AD^2 deveria 
acompanhar essa racionalidade. Por conseguinte, AD não pode ser, também, 
racional.
É possível obter AD por caminhos sintéticos, usando a Lei dos Cossenos, por 
exemplo, nos triângulos ABD e ACD, juntamente com mais alguma trigonometria. 
Entretanto, aí sim a solução fica bem mais bizarra...
Espero ter contribuído.
Márcio Pinheiro.

--- Em qua, 29/4/09, Cleuber Eduardo cleubersa...@yahoo.com.br escreveu:

De: Cleuber Eduardo cleubersa...@yahoo.com.br
Assunto: [obm-l] problema interessante!!!
Para: obm-l@mat.puc-rio.br
Cc: cleubersa...@yahoo.com.br
Data: Quarta-feira, 29 de Abril de 2009, 16:06


Bom, amigos da lista estou pensando nesse problema a alguns dias, no entanto a 
forma como o fiz é bastante enfadonha.1.Let ABC be a right triangle (∠A = 
90◦). On the hypotenuse BC constructBCD. Prove that the lengths of the segments 
AB,AC, and AD cannot all be rational. donha. Obrigado desde já.

Veja quais são os assuntos do momento no Yahoo! + Buscados: Top 10 - 
Celebridades - Música - Esportes
in the exterior the equilateral triangle  

Veja quais são os assuntos do momento no Yahoo! + Buscados: Top 10 - 
Celebridades - Música - Esportes

Veja quais são os assuntos do momento no Yahoo! + Buscados: Top 10 - 
Celebridades - Música - Esportes 

Veja quais são os assuntos do momento no Yahoo! + Buscados: Top 10 - 
Celebridades - Música - Esportes


  Veja quais são os assuntos do momento no Yahoo! +Buscados
http://br.maisbuscados.yahoo.com

Re: Res: [obm-l] problema interessante!!!

2009-05-02 Por tôpico Márcio Pinheiro
De nada.
Fiquei curioso quanto à tua solução por Ptolomeu.
Qual é o ponto P?
Valeu, Cleuber.

--- Em qui, 30/4/09, Cleuber Eduardo cleubersa...@yahoo.com.br escreveu:

De: Cleuber Eduardo cleubersa...@yahoo.com.br
Assunto: Res: [obm-l] problema interessante!!!
Para: obm-l@mat.puc-rio.br
Cc: cleubersa...@yahoo.com.br
Data: Quinta-feira, 30 de Abril de 2009, 10:18






Valeu Márcio!!. Quando eu peguei esse problema a princípio  eu tratei o 
problema de uma forma parecida com a tua. Mas ontem eu percebi que se 
construíssemos  um triangulo equilátero auxiliar ACE e depois  ptlolomeu no 
quadriátero APCE e BPCD. E so no final usa-se a lei dos cossenos pra terminar. 
 
Obrigado





De: Márcio Pinheiro profmar...@yahoo.com.br
Para: obm-l@mat.puc-rio.br
Enviadas: Quinta-feira, 30 de Abril de 2009 8:39:51
Assunto: Re: [obm-l] problema interessante!!!






A idéia inicial pode ser expressar AD em função de AB = c e de AC = b (essa é a 
parte realmente enfadonha). Um caminho (não acessível a quem ainda não tem 
conhecimentos razoáveis de números complexos) é adotar um plano de Argand-Gauss 
em que A é a origem e os eixos contêm os catetos. Em um tal plano, sejam A (0, 
0), B (0, c) e C (b, 0), por exemplo. O vetor CB pode ser obtido por meio de 
uma rotação do vetor CD de pi/3 em torno de C. Lembrando que o vetor PQ, com P 
(m, n) e Q (p, q), pode ser associado biunivocamente tanto a P - Q (m - p, n - 
q) quanto ao complexo (m - p) + (n - q)i, i^2 = -1, conclui-se que:
(vetor CD)*(cospi/3 + isenpi/3) = (vetor CB), o que equivale a [(x - b) + 
iy]*[(1/2) + i(sqrt3)/2] = - b + ci, sendo D (x, y).
Da igualdade das partes real e imaginária, impõe-se que x = (b + csqrt3)/2 e y 
= (c + bsqrt3)/2 (espero não ter errado as contas, feitas de cabeça :D).
Finalmente, obtém-se que AD^2 = b^2 + c^2 + bcsqrt3, através da distância entre 
os pontos A e D. Supondo que b e c sejam racionais, conclui-se que b^2, c^2 e 
bc também o 
são. Logo AD^2 seria irracional. Mas, caso AD fosse racional, AD^2 deveria 
acompanhar essa racionalidade. Por conseguinte, AD não pode ser, também, 
racional.
É possível obter AD por caminhos sintéticos, usando a Lei dos Cossenos, por 
exemplo, nos triângulos ABD e ACD, juntamente com mais alguma trigonometria. 
Entretanto, aí sim a solução fica bem mais bizarra...
Espero ter contribuído.
Márcio Pinheiro.

--- Em qua, 29/4/09, Cleuber Eduardo cleubersa...@yahoo.com.br escreveu:

De: Cleuber Eduardo cleubersa...@yahoo.com.br
Assunto: [obm-l] problema interessante!!!
Para: obm-l@mat.puc-rio.br
Cc: cleubersa...@yahoo.com.br
Data: Quarta-feira, 29 de Abril de 2009, 16:06






Bom, amigos da lista estou pensando nesse problema a alguns dias, no entanto a 
forma como o fiz é bastante enfadonha. 
1.Let ABC be a right triangle (∠A = 90◦). On the hypotenuse BC construct
in the exterior the equilateral triangle BCD. Prove that the lengths of the 
segments AB,
AC, and AD cannot all be rational.donha. Obrigado desde já.


Veja quais são os assuntos do momento no Yahoo! + Buscados: Top 10 - 
Celebridades - Música - Esportes


Veja quais são os assuntos do momento no Yahoo! + Buscados: Top 10 - 
Celebridades - Música - Esportes


Veja quais são os assuntos do momento no Yahoo! + Buscados: Top 10 - 
Celebridades - Música - Esportes


  Veja quais são os assuntos do momento no Yahoo! +Buscados
http://br.maisbuscados.yahoo.com

Res: Res: [obm-l] problema interessante!!!

2009-05-02 Por tôpico Cleuber Eduardo
Então, vamos lá:
 
Fazendo o desenho e que te disse e a construção auxiliar do triangulo 
equilátero ACE. Vamos usar um colorário.
COLORÁRIO: BE=AD
DEMONSTRAÇÃO: Sendo P a intersecção da circunferência circunscrita aos 
respectivos triangulos ACE e BCD. Logo D^PC=pi/3 e A^MC=2pi/3. Então P está em 
AD, e de forma análoga P está em BE. Finalmente aplicando 
Ptolomeu!!!quadriláteros BPCD e APCE temos as relações  PE=AP +PC, PD=PB+PC. 
Logo AD=PM+PB+PC=BE cqd. Em ABE aplica-se lei dos cossenos BE^2= AB^2+AE^2 
+AE*AB*3^1/2. No entanto, BE=AD, AE=AC. Então: AD^2=AB^2+AC^2+AB.AC*3^1/2 cqd. 
Bom cheguei no mesmo resultado que vc obteve praticament. É um bom problema, 
enfim!!! 
 



De: Márcio Pinheiro profmar...@yahoo.com.br
Para: obm-l@mat.puc-rio.br
Enviadas: Quinta-feira, 30 de Abril de 2009 12:33:47
Assunto: Re: Res: [obm-l] problema interessante!!!


De nada.
Fiquei curioso quanto à tua solução por Ptolomeu.
Qual é o ponto P?
Valeu, Cleuber.

--- Em qui, 30/4/09, Cleuber Eduardo cleubersa...@yahoo.com.br escreveu:

De: Cleuber Eduardo cleubersa...@yahoo.com.br
Assunto: Res: [obm-l] problema interessante!!!
Para: obm-l@mat.puc-rio.br
Cc: cleubersa...@yahoo.com.br
Data: Quinta-feira, 30 de Abril de 2009, 10:18


Valeu Márcio!!. Quando eu peguei esse problema a princípio  eu tratei o 
problema de uma forma parecida com a tua. Mas ontem eu percebi que se 
construíssemos  um triangulo equilátero auxiliar ACE e depois  ptlolomeu no 
quadriátero APCE e BPCD. E so no final usa-se a lei dos cossenos pra terminar. 

Obrigado





De: Márcio Pinheiro profmar...@yahoo.com.br
Para: obm-l@mat.puc-rio.br
Enviadas: Quinta-feira, 30 de Abril de 2009 8:39:51
Assunto: Re: [obm-l] problema interessante!!!


A idéia inicial pode ser expressar AD em função de AB = c e de AC = b (essa é a 
parte realmente enfadonha). Um caminho (não acessível a quem ainda não tem 
conhecimentos razoáveis de números complexos) é adotar um plano de Argand-Gauss 
em que A é a origem e os eixos contêm os catetos. Em um tal plano, sejam A (0, 
0), B (0, c) e C (b, 0), por exemplo. O vetor CB pode ser obtido por meio de 
uma rotação do vetor CD de pi/3 em torno de C. Lembrando que o vetor PQ, com P 
(m, n) e Q (p, q), pode ser associado biunivocamente tanto a P - Q (m - p, n - 
q) quanto ao complexo (m - p) + (n - q)i, i^2 = -1, conclui-se que:
(vetor CD)*(cospi/3 + isenpi/3) = (vetor CB), o que equivale a [(x - b) + 
iy]*[(1/2) + i(sqrt3)/2] = - b + ci, sendo D (x, y).
Da igualdade das partes real e imaginária, impõe-se que x = (b + csqrt3)/2 e y 
= (c + bsqrt3)/2 (espero não ter errado as contas, feitas de cabeça :D).
Finalmente, obtém-se que AD^2 = b^2 + c^2 + bcsqrt3, através da distância entre 
os pontos A e D. Supondo que b e c sejam racionais, conclui-se que b^2, c^2 e 
bc também o 
são. Logo AD^2 seria irracional. Mas, caso AD fosse racional, AD^2 deveria 
acompanhar essa racionalidade. Por conseguinte, AD não pode ser, também, 
racional.
É possível obter AD por caminhos sintéticos, usando a Lei dos Cossenos, por 
exemplo, nos triângulos ABD e ACD, juntamente com mais alguma trigonometria. 
Entretanto, aí sim a solução fica bem mais bizarra...
Espero ter contribuído.
Márcio Pinheiro.

--- Em qua, 29/4/09, Cleuber Eduardo cleubersa...@yahoo.com.br escreveu:

De: Cleuber Eduardo cleubersa...@yahoo.com..br
Assunto: [obm-l] problema interessante!!!
Para: obm-l@mat.puc-rio.br
Cc: cleubersa...@yahoo.com.br
Data: Quarta-feira, 29 de Abril de 2009, 16:06


Bom, amigos da lista estou pensando nesse problema a alguns dias, no entanto a 
forma como o fiz é bastante enfadonha.1.Let ABC be a right triangle (∠A = 
90◦). On the hypotenuse BC constructBCD. Prove that the lengths of the segments 
AB,AC, and AD cannot all be rational. donha. Obrigado desde já.

Veja quais são os assuntos do momento no Yahoo! + Buscados: Top 10 - 
Celebridades - Música - Esportes
in the exterior the equilateral triangle  

Veja quais são os assuntos do momento no Yahoo! + Buscados: Top 10 - 
Celebridades - Música - Esportes

Veja quais são os assuntos do momento no Yahoo! + Buscados: Top 10 - 
Celebridades - Música - Esportes 

Veja quais são os assuntos do momento no Yahoo! + Buscados: Top 10 - 
Celebridades - Música - Esportes


  Veja quais são os assuntos do momento no Yahoo! +Buscados
http://br.maisbuscados.yahoo.com

Res: [obm-l] problema interessante!!!

2009-04-30 Por tôpico Cleuber Eduardo
Valeu Márcio!!. Quando eu peguei esse problema a princípio  eu tratei o 
problema de uma forma parecida com a tua. Mas ontem eu percebi que se 
construíssemos  um triangulo equilátero auxiliar ACE e depois  ptlolomeu no 
quadriátero APCE e BPCD. E so no final usa-se a lei dos cossenos pra terminar. 

Obrigado





De: Márcio Pinheiro profmar...@yahoo.com.br
Para: obm-l@mat.puc-rio.br
Enviadas: Quinta-feira, 30 de Abril de 2009 8:39:51
Assunto: Re: [obm-l] problema interessante!!!


A idéia inicial pode ser expressar AD em função de AB = c e de AC = b (essa é a 
parte realmente enfadonha). Um caminho (não acessível a quem ainda não tem 
conhecimentos razoáveis de números complexos) é adotar um plano de Argand-Gauss 
em que A é a origem e os eixos contêm os catetos. Em um tal plano, sejam A (0, 
0), B (0, c) e C (b, 0), por exemplo. O vetor CB pode ser obtido por meio de 
uma rotação do vetor CD de pi/3 em torno de C. Lembrando que o vetor PQ, com P 
(m, n) e Q (p, q), pode ser associado biunivocamente tanto a P - Q (m - p, n - 
q) quanto ao complexo (m - p) + (n - q)i, i^2 = -1, conclui-se que:
(vetor CD)*(cospi/3 + isenpi/3) = (vetor CB), o que equivale a [(x - b) + 
iy]*[(1/2) + i(sqrt3)/2] = - b + ci, sendo D (x, y).
Da igualdade das partes real e imaginária, impõe-se que x = (b + csqrt3)/2 e y 
= (c + bsqrt3)/2 (espero não ter errado as contas, feitas de cabeça :D).
Finalmente, obtém-se que AD^2 = b^2 + c^2 + bcsqrt3, através da distância entre 
os pontos A e D. Supondo que b e c sejam racionais, conclui-se que b^2, c^2 e 
bc também o 
são. Logo AD^2 seria irracional. Mas, caso AD fosse racional, AD^2 deveria 
acompanhar essa racionalidade. Por conseguinte, AD não pode ser, também, 
racional.
É possível obter AD por caminhos sintéticos, usando a Lei dos Cossenos, por 
exemplo, nos triângulos ABD e ACD, juntamente com mais alguma trigonometria. 
Entretanto, aí sim a solução fica bem mais bizarra...
Espero ter contribuído.
Márcio Pinheiro.

--- Em qua, 29/4/09, Cleuber Eduardo cleubersa...@yahoo.com.br escreveu:

De: Cleuber Eduardo cleubersa...@yahoo.com.br
Assunto: [obm-l] problema interessante!!!
Para: obm-l@mat.puc-rio.br
Cc: cleubersa...@yahoo.com.br
Data: Quarta-feira, 29 de Abril de 2009, 16:06


Bom, amigos da lista estou pensando nesse problema a alguns dias, no entanto a 
forma como o fiz é bastante enfadonha.1.Let ABC be a right triangle (∠A = 
90◦). On the hypotenuse BC constructBCD. Prove that the lengths of the segments 
AB,AC, and AD cannot all be rational.donha. Obrigado desde já.

Veja quais são os assuntos do momento no Yahoo! + Buscados: Top 10 - 
Celebridades - Música - Esportes
in the exterior the equilateral triangle  

Veja quais são os assuntos do momento no Yahoo! + Buscados: Top 10 - 
Celebridades - Música - Esportes


  Veja quais são os assuntos do momento no Yahoo! +Buscados
http://br.maisbuscados.yahoo.com

RES: [obm-l] problema análise

2009-01-08 Por tôpico Artur Costa Steiner


-Mensagem original-
De: owner-ob...@mat.puc-rio.br [mailto:owner-ob...@mat.puc-rio.br]em nome de 
Murilo Krell
Enviada em: quarta-feira, 7 de janeiro de 2009 17:10
Para: obm-l@mat.puc-rio.br
Assunto: [obm-l] problema análise


Prezados amigos,
poderiam me ajudar com esses problemas?

a) Se uma sequência é monótona tem uma subsequência convergente, prove que a 
sequência é, ela própria convergente.

  Sejam x_n uma sequencia monotona crescente e x_n_k uma subsequencia 
convergente de x_n. Por ser convergente, x_n_k é limitada, havendo assim M  0 
tal que x_n_k  M para todo k =1,2,3. Pela definicao de subsequencia, para 
todo n existe k tal que n_k  n. Como x_n eh monotona crescente, temos então 
que x_n = x_n_k  M, do que deduzimos que x_n, alem de monotona crescente, eh 
limitada superiormente por M. Logo, x_n eh convergente.
Se x_n for monotona decrescente, o raciocinio eh similar.

b) A fim de que o número real a seja valor de aderência de (xn) é necessário e 
suficiente, que, para todo eps0 e todo k pertencente a N dados, exista
nk tal que o modulo de xn-aeps

Suponhamos que a seja ponto de aderencia de x_n. Existe, entao, uma 
subsequencia x_n_i de x_n que converge para a. Para todo eps 0, temos entao 
que, com possivel excecao de um numero finito de termos, a bola aberta de 
centro em a e raio eps contem todos os termos de x_n_i. Para todo k de N, 
existe entao, pela definicao de subsequencia, i pertencente a N tal que n_i  k 
e x_n_i esta na citada bola. Fazendo-se n = n_i, obtemos n  k tal que |x_n - 
a|  eps.



Suponhamos agora, por outro lado, que, para todo eps0 e todo k pertencente a N 
dados, exista n tal que |x_n - a|  eps. Fazendo-se eps = 1/1 e k = 1, obtemos 
n_1  1 tal que |x_n_1 - a|  1. De forma indutiva, suponhamos que, para algum 
i de N, existam n_1  n_2n_i tais que |x_n_1 - a|  1, |x_n_2 - a|  
1/2...|x_n_i - a|  1/i. Fazendo-se k = n_i e eps = 1/(i +1), obtemos n_(i + 1) 
 n_i tal que |x_n_(i + 1) - a|  eps. Com isto, construimos uma subsequencia 
x_n_i de x_n tal que, para todo i, |x_n_i - a|  1/i. Como 1/i  - 0, x_n_i - 
a. Concluimos, assim, que a eh ponto de aderencia de x_n.

Artur

Problemas do livro de análise do Elon


desde já agradeço imensamente a ajuda,
abraços,

Jhonata



RES: [obm-l] Problema - Campeonato Paulista

2008-08-22 Por tôpico Dória
Pessoal, muito obrigada pela ajuda!
Um abraço.


From: *Dória* [EMAIL PROTECTED]
Date: 2008/8/21
To: obm-l@mat.puc-rio.br

Olá!
 Podem me ajudar nesse exercício, por favor?

No Campeonato Paulista de Futebol, participam 20 clubes. Se todas as equipes
jogam entre si uma única vez, qual o total de partidas deste campeonato?

[ ]'s

--
From: *Fernando Lima Gama Junior* [EMAIL PROTECTED]
Date: 2008/8/21
To: obm-l@mat.puc-rio.br


20*19/2 = 190


2008/8/21 Dória [EMAIL PROTECTED]


--
From: *Dória* [EMAIL PROTECTED]
Date: 2008/8/21
To: obm-l@mat.puc-rio.br


 Quando faço 20*19 o que eu encontro?

Obrigada.

2008/8/21 Fernando Lima Gama Junior [EMAIL PROTECTED]


--
From: *Iuri* [EMAIL PROTECTED]
Date: 2008/8/21
To: obm-l@mat.puc-rio.br


Na primeira escolha vc tem 20 times pra escolher. Na segunda, tem 19, já que
um deles foi escolhido anteriormente. Como escolher primeiro o time A e
depois o time B ou primeiro escolher o time B e depois o time A são a mesma
coisa, vc divide o resultado por 2.

--
From: *Fernando Lima Gama Junior* [EMAIL PROTECTED]
Date: 2008/8/21
To: obm-l@mat.puc-rio.br


Encontra todas as combinações, importando a ordem. Assim, tem que X - Y é
diferente de Y - X. Quando se divide por 2, tem-se apenas uma combinação.

Ou seja, para a primeira opção, temos 20 times. Para a segunda, 19. Assim,
haveria 380 jogos (20x19) se os jogos fossem de ida e de volta. Como é
apenas uma partida entre cada time, temos q diviidr por 2.

Abraços,

--
From: *João Luís* [EMAIL PROTECTED]
Date: 2008/8/21
To: obm-l@mat.puc-rio.br


 Veja, Dória, se são 20 clubes, cada um joga 19 vezes, certo? Então seriam
20*19 jogos!

Seriam, porque, fazendo a conta desse modo cada jogo foi contado duas vezes;
para corrigir isso, divid-se o total por 2: 20*19/2, 190 jogos.

Compreendeu?

Um abraço,

João Luís

--
From: *Bruno França dos Reis* [EMAIL PROTECTED]
Date: 2008/8/21
To: obm-l@mat.puc-rio.br


Vc encontra 380.

Brincadeira, não resisti.

Esse 20*19 é o que chamamos de *arranjo*.
A(n, r)  =  arranjos de  n elementos, tomados r a r = n! / (n - r)!
O número A(n, r) é a quantidade de r-uplas (ordenadas) distintas que podemos
formar a partir dos elementos de um conjunto de n elementos.
No nosso caso, n = 20, r = 2. Então esse é o número de *pares ordenados* que
podemos formar a partir dos 20 elementos (clubes).

A divisão por 2 é para desconsiderarmos pares que difiram apenas na ordem de
seus elementos, o que nos leva a outro conceito importante, o de *
combinações*.

C(n, r) = combinações de n elementos, tomados r a r = n! / (r! (n-r)!)
Esse número é a quantidade de subconjuntos de tamanho r que podemos formar a
partir dos elementos de um *conjunto *de tamanho n. A diferença sutil é que
antes fazíamos r-uplas, nas quais a ordem é importante ( (A, B) é diferente
de (B, A)), e agora fazemos conjuntos, objeto no qual não importa a ordem (
{A, B} é o mesmo que {B, A} ).



Abraço
Bruno
-- 
Bruno FRANÇA DOS REIS

msn: [EMAIL PROTECTED]
skype: brunoreis666
tel: +33 (0)6 28 43 42 16

e^(pi*i)+1=0


Res: [obm-l] Problema das Vigas

2008-03-06 Por tôpico Eduardo Estrada
Não, pois os ângulos inferiores, na figura, são retos.

- Mensagem original 
De: Joao Victor Brasil [EMAIL PROTECTED]
Para: obm-l@mat.puc-rio.br
Enviadas: Quinta-feira, 6 de Março de 2008 15:46:26
Assunto: Re: [obm-l] Problema das Vigas

AB=CD???

On 3/6/08, Eduardo Estrada [EMAIL PROTECTED] wrote:Olá,

Alguém conhece uma solução simples para o Problema das Vigas? Consiste no 
seguinte:

Imagine a seguinte figura:

 ||
A  ||
 ||
 ||
 || D
 ||
 ||
 |__|
B   C
   

AC = 30 m, BD = 20 m, AC e BD interceptam-se em P, que dista 8 m de BC. 
Pede-se, calcular o tamanho de BC. Aparentemente simples, para resolver este 
problema, caímos numa equação de grau maior do que 2. Então, a pergunta, existe 
alguma solução simples?

Um abraço,
Eduardo



Abra sua conta no Yahoo! Mail, o único sem limite de espaço para armazenamento! 








  Abra sua conta no Yahoo! Mail, o único sem limite de espaço para 
armazenamento!
http://br.mail.yahoo.com/

RES: [obm-l] problema de cálculo

2008-01-21 Por tôpico Artur Costa Steiner
Não entendi este enunciado. |p| eh o valor absoluto do vetor, nao um vetor de 
R^n. Nao estou vendo sentindo
Artur

-Mensagem original-
De: [EMAIL PROTECTED] [mailto:[EMAIL PROTECTED] nome de Francisco
Enviada em: sexta-feira, 18 de janeiro de 2008 15:47
Para: Lista de discursão
Assunto: [obm-l] problema de cálculo


Olá Pessoal!

Alguém pode me ajudar no problema abaixo? Não parece difícil, mas não consigo o 
truque!!!

Problema: Seja f: IR^n -- IR diferenciável não constante. Dado c  0, mostre 
que existe p em IR^n tal que |p| = c e p é paralelo ao gradiente de f em p.

Obrigado desde já,
  Francisco.

PS.: Aliás, não consigo nenhuma possível interpretação (geométrica) para o 
enunciado!

  _

Veja mapas e encontre as melhores rotas para fugir do trânsito com o Live 
Search Maps! Experimente já!http://www.livemaps.com.br/index.aspx?tr=true



RE: RES: [obm-l] problema de cálculo

2008-01-21 Por tôpico Francisco

Olá Artur.

Acho que realmente escrevi errado, o problema é que existe alguns livros de 
cálculo que escreve para norma de um vetor simplesmente  |.|

|p| = ||p|| , ou seja, |p| significa norma de do vetor p.

Francisco 

From: [EMAIL PROTECTED]
To: obm-l@mat.puc-rio.br
Date: Mon, 21 Jan 2008 09:44:11 -0200
Subject: RES: [obm-l] problema de cálculo











Não entendi este enunciado. |p| eh o valor absoluto do vetor, nao um vetor de 
R^n. Nao estou vendo sentindo
Artur  

-Mensagem original-

De: [EMAIL PROTECTED] [mailto:[EMAIL PROTECTED] nome de
Francisco

Enviada em: sexta-feira, 18 de janeiro de 2008 15:47

Para: Lista de discursão

Assunto: [obm-l] problema de cálculo




Olá Pessoal!



Alguém pode me ajudar no problema abaixo? Não parece difícil, mas não consigo o 
truque!!!



Problema: Seja f: IR^n -- IR diferenciável não constante. Dado c  0, mostre 
que existe p em IR^n tal que |p| = c e p é paralelo ao gradiente de f em p.



Obrigado desde já,

  Francisco.



PS.: Aliás, não consigo nenhuma possível interpretação (geométrica) para o 
enunciado!


Veja mapas e encontre as melhores rotas para fugir do trânsito com o Live 
Search Maps!
Experimente já!




_
Confira vídeos com notícias do NY Times, gols direto do Lance, videocassetadas 
e muito mais no MSN Video!
http://video.msn.com/?mkt=pt-br

RES: RES: [obm-l] Problema com polinômios

2008-01-15 Por tôpico Artur Costa Steiner
Eh, tem toda a razao, pode haver outras raizes. Obrigado pela correcao.

Mas creio que dah para aproveitar o raciocinio anterior.

Temos que Q(x) = (x -a)(x - b)(x -c)(x - d).T(x).

O polinomio (x -a)(x - b)(x -c)(x - d) eh monico e tem coeficientes inteiros. 
Como Q tem tambem coeficientes inteiros e eh monico, o algoritmo da divisao de 
polinomios implica que T seja monico e tenha coeficiente inteiros. Se p(k) = 8 
para algum inteiro k, entao Q(k) = 3 e

3 = (k -a)(k - b)(k -c)(k - d) T(k)

Como T(k) eh inteiro, vemos que 3 eh dado por um produto de 5 inteiros, dos 
quais 4 sao distintos 2 a 2. Isto implica que 3 tenha pelo menos 4 divisores, 
contrariando o fato de que 3 eh primo.

Agora estah certo, nao estah?

Artur


-Mensagem original-
De: [EMAIL PROTECTED] [mailto:[EMAIL PROTECTED] nome de Carlos Nehab
Enviada em: segunda-feira, 14 de janeiro de 2008 19:20
Para: obm-l@mat.puc-rio.br
Assunto: Re: RES: [obm-l] Problema com polinômios


Oi, Arthur,

Acho que podem existir outras raízes e, como conseqüência, Q(x) = (x -a)(x - 
b)(x -c)(x - d).T(x), onde o polinômio quociente T(x) não seria identicamente 
igual a 1...   Confesso que dei uma tentada por ai mas empaquei, pois não achei 
contra exemplo nem tampouco provei que T(k) seria inteiro...  Onde será que 
estou voando?

Abração,
Nehab

Artur Costa Steiner escreveu:

Definamos Q(x) = P(x) - 5. Entao, Q eh um polinomio monico (pois P eh monico) e 
admite a, b, c e d como raizes, distintas 2 a 2. Segue-se que

Q(x) = (x -a) (x -b ) (x -c ) (x - d). Se P(k) = 8 para algum inteiro k, entao 
Q(k) = 3 e

Q(k)  = 3 = (k-a) (k -b) (k -c) (k -d). Como k eh inteiro e a, b, c e d sao 
inteiros distintos 2 a 2, isto signfica que 3 eh dado pelo produto de 4 numeros 
inteiros distintos 2 a 2.  Mas isto é impossivel, pois 3 eh primo. Logo, nao 
existe nenhum inteiro k com P(k) = 8.

Accho que estah certo.


[Artur Costa Steiner]
 -Mensagem original-
De: [EMAIL PROTECTED]mailto:[EMAIL PROTECTED] [ mailto:[EMAIL PROTECTED] nome 
de Marcelo Salhab Brogliato
Enviada em: segunda-feira, 14 de janeiro de 2008 10:23
Para: obm-l@mat.puc-rio.brmailto:obm-l@mat.puc-rio.br
Assunto: Re: [obm-l] Problema com polinômios



Olá Igor,

estou tentando encontrar um contra-exemplo (pra mim, é um ótimo jeito de se 
encontrar uma demonstração.. hehe!)

p(x) = x^n + a_1*x^(n-1) + a_2*x^(n-2) + ... + a_(n-1)*x + a_n

vamos supor que: p(a) = p(b) = p(c) = p(d) = 5, e p(k) = 8
onde a, b, c, d, k sao primos entre si dois a dois.
deste modo:
p(a) = 5 = a_n (mod a)
p(b) = 5 = a_n (mod b)
p(c) = 5 = a_n (mod c)
p(d) = 5 = a_n (mod d)
p(k) = 8 = a_n (mod k)

pelo teorema chines do resto, conseguimos determinar a_n (mod a.b.c.d.k)
fazendo: g(x) = [p(x) - a_n]/x, temos que: g(a), g(b), g(c), g(d) e g(k) estão 
definidos..
então, usando a mesma idéia, determinamos a_(n-1).
seguindo esta idéia, conseguimos determinar todos os coeficientes do polinômio!

qual o erro nesta idéia? não encontrei...

abraços,
Salhab







2008/1/12 Igor Battazza  [EMAIL PROTECTED]mailto:[EMAIL PROTECTED]:


Olá pessoal,
estou com dúvidas na seguinte questão:

Dado o polinômio p(x) = x^n + a_1*x^(n-1) + a_2*x^(n-2) + ... +
a_(n-1)*x + a_n com coeficientes inteiros a_1, a_2, ..., a_n, e dado
que também existem 4 inteiros distintos a, b, c e d tal que p(a) =
p(b) = p(c) = p(d) = 5, mostre que não existe inteiro k tal que p(k) =
8.

Não consigo pensar em nenhuma restrição que implique nisso.

Obrigado,
Igor.

=
Instruções para entrar na lista, sair da lista e usar a lista em
http://www.mat.puc-rio.br/~obmlistas/obm-l.htmlhttp://www.mat.puc-rio.br/%7Eobmlistas/obm-l.html
=



= 
Instruções para entrar na lista, sair da lista e usar a lista em 
http://www.mat.puc-rio.br/~obmlistas/obm-l.html 
=



RES: [obm-l] Problema com polinômios

2008-01-14 Por tôpico Artur Costa Steiner
Definamos Q(x) = P(x) - 5. Entao, Q eh um polinomio monico (pois P eh monico) e 
admite a, b, c e d como raizes, distintas 2 a 2. Segue-se que

Q(x) = (x -a) (x -b ) (x -c ) (x - d). Se P(k) = 8 para algum inteiro k, entao 
Q(k) = 3 e

Q(k)  = 3 = (k-a) (k -b) (k -c) (k -d). Como k eh inteiro e a, b, c e d sao 
inteiros distintos 2 a 2, isto signfica que 3 eh dado pelo produto de 4 numeros 
inteiros distintos 2 a 2.  Mas isto é impossivel, pois 3 eh primo. Logo, nao 
existe nenhum inteiro k com P(k) = 8.

Accho que estah certo.


[Artur Costa Steiner]
 -Mensagem original-
De: [EMAIL PROTECTED] [mailto:[EMAIL PROTECTED] nome de Marcelo Salhab Brogliato
Enviada em: segunda-feira, 14 de janeiro de 2008 10:23
Para: obm-l@mat.puc-rio.br
Assunto: Re: [obm-l] Problema com polinômios



Olá Igor,

estou tentando encontrar um contra-exemplo (pra mim, é um ótimo jeito de se 
encontrar uma demonstração.. hehe!)

p(x) = x^n + a_1*x^(n-1) + a_2*x^(n-2) + ... + a_(n-1)*x + a_n

vamos supor que: p(a) = p(b) = p(c) = p(d) = 5, e p(k) = 8
onde a, b, c, d, k sao primos entre si dois a dois.
deste modo:
p(a) = 5 = a_n (mod a)
p(b) = 5 = a_n (mod b)
p(c) = 5 = a_n (mod c)
p(d) = 5 = a_n (mod d)
p(k) = 8 = a_n (mod k)

pelo teorema chines do resto, conseguimos determinar a_n (mod a.b.c.d.k)
fazendo: g(x) = [p(x) - a_n]/x, temos que: g(a), g(b), g(c), g(d) e g(k) estão 
definidos..
então, usando a mesma idéia, determinamos a_(n-1).
seguindo esta idéia, conseguimos determinar todos os coeficientes do polinômio!

qual o erro nesta idéia? não encontrei...

abraços,
Salhab







2008/1/12 Igor Battazza  [EMAIL PROTECTED]mailto:[EMAIL PROTECTED]:


Olá pessoal,
estou com dúvidas na seguinte questão:

Dado o polinômio p(x) = x^n + a_1*x^(n-1) + a_2*x^(n-2) + ... +
a_(n-1)*x + a_n com coeficientes inteiros a_1, a_2, ..., a_n, e dado
que também existem 4 inteiros distintos a, b, c e d tal que p(a) =
p(b) = p(c) = p(d) = 5, mostre que não existe inteiro k tal que p(k) =
8.

Não consigo pensar em nenhuma restrição que implique nisso.

Obrigado,
Igor.

=
Instruções para entrar na lista, sair da lista e usar a lista em
http://www.mat.puc-rio.br/~obmlistas/obm-l.htmlhttp://www.mat.puc-rio.br/%7Eobmlistas/obm-l.html
=





[obm-l] RES: [obm-l] Problema de funções do Artur

2007-08-31 Por tôpico Artur Costa Steiner
Muito obrigado
Artur

-Mensagem original-
De: [EMAIL PROTECTED] [mailto:[EMAIL PROTECTED]
nome de Nicolau C. Saldanha
Enviada em: sexta-feira, 31 de agosto de 2007 11:28
Para: [EMAIL PROTECTED]; obm-l@mat.puc-rio.br
Assunto: [obm-l] Problema de funções do Artur


On Thu, Aug 23, 2007 at 01:47:08PM -0300, Artur Costa Steiner wrote:
 Seja f definida em (0, oo), nao negativa e monotonicamente decrescente.
 Podemos provar, sem maiores dificuldades, que lim (n -- oo)  [f(1) +
 f(2)+  f(n) - Int (1 a n) f(t) dt ] existe. Isto é decorrência direta do
 carater monotonicamente decrescente de f. Mesmo que a serie e a integral
 infinita divirjam, o limite sempre existe. A sequencia é limitada
 inferiormente por 0 e eh monotica decrescente. Como f eh monotonica a
 integral existe em qualquer intervalo compacto.
 
 Suponhamos agora que, para cada x = 0 fixo, f_x seja definida em [1, oo) por
 f_x(t) = 1/t^x. Entao, f_x eh estritamente decrescente para x  0 e constante
 em 1 pra x = 0. Definamos g(x) = lim (n -- oo)  [1/1^x + 1/2^x .1/n^x -
 Int (1 a n) f_x(t) dt ]. Pelo que vimos, este limite existe para todo x e g
 estah bem definida. Se x1,
 
 g(x) = lim (n -- oo)  [1/1^x + 1/2^x .1/n^x -  (n^(1 - x) - 1)/(1 - x) ] 
  e , se x=1
 
 g(1) = lim (n -- oo)  [1/1 + 1/2 .1/nx - ln(n)] , que é a famosa
 constante de Euler/Mascheroni, pouco maior que 0, 5
 
 Se x 1, na definição de g a série e a integral convergem, e temos que 
 
 g(x) =  lim (n -- oo)  [1/1^x + 1/2^x .1/n^x -  1/(x -1)] = Z(x) - 1/(x
 -1), sendo Z a funcao zeta de Riemann. Da análise complexa, sabe-se que Zé
 analítica, apresentando assim derivadas de todas as ordens também na reta
 real. Logo, g é difrenciavel em (1, oo) e g'(x) = Z'(x) + 1/(x-1)^2
 
 Se x estiver em (0, 1], entao a integral e serie divergem. Ttentei provar que 
  g é derivável tambem em [0,1], mas nap consegui.

Escreva

g(s) = SOMA_{n=1}^{infinito} h_n(s),
h_n(s) = 1/n^s - (int_n^(n+1) dt/t^s)
   = n^(-s) - (int_n^(n+1) t^(-s) dt)
   = exp(-s log n) - (int_n^(n+1) exp(-s log t) dt).

Assim g fica escrita como uma série de funções.
Note que a função t^(-s) é decrescente em t logo

0 = h_n(s) = n^(-s) - (n+1)^(-s)

e um argumento telescópico prova a convergência da série para s  0.
Para verificar que g é derivável devemos estimar as derivadas h_n'(s):

h_n'(s) = H(s,n) - int_n^(n+1) H(s,t) dt,
H(s,t) = - log t exp(-s log t).

A derivada parcial de H em relação a t é

H_t(s,t) = (s log t - 1) exp(-s log t) / t

donde H_t(s,t)  0 para t  exp(1/s).
Ou seja, em qualquer intervalo compacto contido em (0,infinito)
existe um N a partir do qual

H(s,n) - H(s,n+1) = h_n'(s) = 0

e novamente por um argumento telescópico a série SOMA h_n'(s)
converge uniforme e absolutamente para uma função contínua
que será g'(s).

Mas o melhor mesmo é provar que a sua função g é *inteira*.
Considere a fórmula que você provou para s  1: g(s) = Z(s) - 1/(s-1).
Ora, é sabido que a função zeta tem uma única singularidade em C:
um polo simples em s=1. Ao subtrair 1/(s-1), você obteve uma função
inteira g_1(s) = Z(s) - 1/(s-1). O que você quer provar portanto
é que o limite que você usou para definir g continua convergindo para
o valor correto g_1(s) para s no intervalo (0,1].
Tudo isso pode ser feito estimando as funções h_n(s) acima
em vizinhanças compactas apropriadas de reais x em (0,infinito).

Note finalmente que o ponto s = 0  não pode ser tratado desta
forma e tenho quase certeza que o seu limite original dá a resposta errada.

[]s, N.



=
Instruções para entrar na lista, sair da lista e usar a lista em
http://www.mat.puc-rio.br/~nicolau/olimp/obm-l.html
=

=
Instruções para entrar na lista, sair da lista e usar a lista em
http://www.mat.puc-rio.br/~nicolau/olimp/obm-l.html
=


[obm-l] RE: [obm-l] RE: [obm-l] RES: [obm-l] Problema de maximiza ção

2007-06-07 Por tôpico Rhilbert Rivera

 
Obrigado amigo, pelos esclarecimentos. 
 
[ ]'s To: obm-l@mat.puc-rio.br From: [EMAIL PROTECTED] Subject: Re: [obm-l] 
RE: [obm-l] RES: [obm-l] Problema de maximização Date: Thu, 7 Jun 2007 
00:00:41 -0300  X é o número total de novilhos. E não o número de novilhos q 
excedem os  20...   O modelo q usei pro P só vale pra x = 20. É basado no 
texto q fala:  - Permite 20 novilhos.  - A cada novilho acrescentado, o peso 
médio (nesse caso, o peso médio entre  os novilhos) cai 22,5 kg.   Ou seja, 
até x=20, o peso médio por novilho é 900 kg. Após isso, perdem-se  22,5 Kg 
por cada novilho q exceder os 20, ou seja, perdem-se 22,5*(x-20).  Logo:  P = 
900 - 22,5*(x-20)   Na sua conta, tem um erro. O coeficiente de x é (900 + 
450) e não (900 -  450). Corrigindo isso, o x q maximiza a função é 30 mesmo, 
como tinha  falado.   Falou!   Em (00:08:55), obm-l@mat.puc-rio.br 
escreveu:Interessante esse seu raciocínio do pesso com relação à área. 
Não havia  pensado nisso...Não consigo entender o modelo feito para o 
peso de cada novilho: P = 900 -  22,5(x-20).x seriam os novilhos que 
se acrescenta no pasto além dos 20 que já estão  lá? Se for isso, quando se 
acrescenta 1 novilho, por exemplo, vai se ter um  ganho no peso e não perda, 
como o problema dizCaso esse modelo esteja correto, o que estaria 
errado se eu fizesseP = x[900 - 22,5(x-20)]P = 450x- 22,5 x^2  
  Se P'=0 então, x = 10 e P = 2250 ?Obrigado por qualquer 
esclarecimento.[ ] ' s:31 -0300 Eu acho q a idéia 
pode ser maximizar o peso médio com relação a área (e  não   com relação ao 
número de novilhos).   Vc coloca um novilho, o peso médio com relação a área 
pode aumentar ou   diminuir. Até os 20, qnd não há perda, com certeza 
aumenta... Resolveria assim: Número de novilhos: x   
Peso de cada novilho (considerando 20 ou mais): P = 900 -22,5*(x-20) Kg   
Peso médio na área: Pm = x*P/50 Maximizando o peso médio na área, 
temos x=30, P = 675 e Pm = 20250. Era isso? Em (15:28:34), 
obm-l@mat.puc-rio.br escreveu:   Este enunciado deve estar 
errado. Da   maneira como foi formulado, o peso médio decresce com o número 
de   novilhos e o ideal é colocar so 1 novilho, jah que peso medio para 0  
 novilhos nao eh definido.   Artur   -Mensagem  
  original-De: [EMAIL PROTECTED][mailto:[EMAIL 
PROTECTED] Em nome de RhilbertRiveraEnviada em: terça-feira, 5 
de junho de 200717:30Para: obm-l@mat.puc-rio.brAssunto: 
[obm-l] Problemade maximização  Olá Colegas  A 
solução dada aoproblema abaixo não me convenceu (isso pode se dever a 
minha ignorânciamesmo), por isso peço uma ajuda na solução do problema. 
Quem sabe dessa   vez euentenda.   Uma fazenda de gado 
permite 20 novilhos por 50 metrosquadrados de pasto. O peso médio de 
seus novilhos no mercado é de 900  kg.Estimativas do Departamento de 
Agricultura (EUA) indicam que o peso  médioficará reduzido em 22,5 kg 
para cada novilho que for acrescentado nos  50metros quadrados de 
pasto. Quantos novilhos devem ser colocados nos 50   metrosquadrados 
para que o peso médio deles seja o maiorpossível?  
Obrigado  [ ]'s   Obtenha o novo Windows Live 
Messenger! Experimente!  -- Instale o 
novo Windows Live Messenger! É grátis! Experimente!--   
_
Receba as últimas notícias do Brasil e do mundo direto no seu Messenger com 
Alertas MSN! É GRÁTIS!
http://alertas.br.msn.com/

[obm-l] RES: [obm-l] Problema de maximização

2007-06-06 Por tôpico Artur Costa Steiner
Este enunciado deve estar errado. Da maneira como foi formulado, o peso médio 
decresce com o número de novilhos e o ideal é colocar so 1 novilho, jah que 
peso medio para 0 novilhos nao eh definido.
 
Artur

-Mensagem original-
De: [EMAIL PROTECTED] [mailto:[EMAIL PROTECTED] nome de Rhilbert Rivera
Enviada em: terça-feira, 5 de junho de 2007 17:30
Para: obm-l@mat.puc-rio.br
Assunto: [obm-l] Problema de maximização



Olá Colegas
 
A solução dada ao problema abaixo não me convenceu (isso pode se dever a minha 
ignorância mesmo), por isso peço uma ajuda na solução do problema. Quem sabe 
dessa vez eu entenda.
 
 Uma fazenda de gado permite 20 novilhos por 50 metros quadrados de pasto. O 
peso médio de seus novilhos no mercado é de 900 kg. Estimativas do Departamento 
de Agricultura (EUA) indicam que o peso médio ficará reduzido em 22,5 kg para 
cada novilho  que for acrescentado nos 50 metros quadrados de pasto. Quantos 
novilhos devem ser colocados nos 50 metros quadrados para que o peso médio 
deles seja o maior possível?
 
Obrigado
 
[ ]'s
 
 


  _  

Obtenha o novo Windows Live Messenger! Experimente! 
http://get.live.com/messenger/overview  



Re: [obm-l] RES: [obm-l] Problema de maximiza�

2007-06-06 Por tôpico edneiramaral
Eu acho q a idéia pode ser maximizar o peso médio com relação a área (e não 
com relação ao número de novilhos). 
Vc coloca um novilho, o peso médio com relação a área pode aumentar ou 
diminuir. Até os 20, qnd não há perda, com certeza aumenta... 

Resolveria assim: 

Número de novilhos: x 
Peso de cada novilho (considerando 20 ou mais): P = 900 -22,5*(x-20) Kg 
Peso médio na área: Pm = x*P/50 

Maximizando o peso médio na área, temos x=30, P = 675 e Pm = 20250. 

Era isso? 

Em (15:28:34), obm-l@mat.puc-rio.br escreveu: 


Este enunciado deve estar errado. Da 
maneira como foi formulado, o peso médio decresce com o número de 
novilhos e o ideal é colocar so 1 novilho, jah que peso medio para 0 
novilhos nao eh definido. 
 
 Artur 
 
 -Mensagem 
 original- 
 De: [EMAIL PROTECTED] 
 [mailto:[EMAIL PROTECTED] Em nome de Rhilbert 
 Rivera 
 Enviada em: terça-feira, 5 de junho de 2007 
 17:30 
 Para: obm-l@mat.puc-rio.br 
 Assunto: [obm-l] Problema 
 de maximização 
 
Olá Colegas 
 
A solução dada ao 
 problema abaixo não me convenceu (isso pode se dever a minha ignorância 
 mesmo), por isso peço uma ajuda na solução do problema. Quem sabe dessa 
vez eu 
 entenda. 
 
 Uma fazenda de gado permite 20 novilhos por 50 metros 
 quadrados de pasto. O peso médio de seus novilhos no mercado é de 900 kg. 
 Estimativas do Departamento de Agricultura (EUA) indicam que o peso médio 
 ficará reduzido em 22,5 kg para cada novilho que for acrescentado nos 50 
 metros quadrados de pasto. Quantos novilhos devem ser colocados nos 50 
metros 
 quadrados para que o peso médio deles seja o maior 
 possível? 
 
Obrigado 
 
[ ]'s 
 
 Obtenha o novo Windows Live Messenger! Experimente! 
 
-- 




[obm-l] RE: [obm-l] RES: [obm-l] Problema de maximização

2007-06-06 Por tôpico Rhilbert Rivera

Interessante esse seu raciocínio do pesso com relação à área. Não havia pensado 
nisso...
 
Não consigo entender  o modelo feito para o peso de cada novilho: P = 900 - 
22,5(x-20).
 
x seriam os novilhos que se acrescenta no pasto além dos 20 que já estão lá? Se 
for isso, quando se acrescenta 1 novilho, por exemplo,  vai se ter um ganho no 
peso e não perda, como o problema diz
 
Caso esse modelo esteja correto, o que estaria errado se eu fizesse
 
P = x[900 - 22,5(x-20)]
P = 450x- 22,5 x^2 
Se P'=0 então, x = 10  e P = 2250 ?
Obrigado por qualquer esclarecimento.
 
[ ] ' s:31 -0300  Eu acho q a idéia pode ser maximizar o peso médio com 
relação a área (e não  com relação ao número de novilhos).  Vc coloca um 
novilho, o peso médio com relação a área pode aumentar ou  diminuir. Até os 
20, qnd não há perda, com certeza aumenta...   Resolveria assim:   Número 
de novilhos: x  Peso de cada novilho (considerando 20 ou mais): P = 900 
-22,5*(x-20) Kg  Peso médio na área: Pm = x*P/50   Maximizando o peso médio 
na área, temos x=30, P = 675 e Pm = 20250.   Era isso?   Em (15:28:34), 
obm-l@mat.puc-rio.br escreveu:Este enunciado deve estar errado. Da  
maneira como foi formulado, o peso médio decresce com o número de  novilhos 
e o ideal é colocar so 1 novilho, jah que peso medio para 0  novilhos nao eh 
definido. Artur -Mensagem   original-   De: [EMAIL 
PROTECTED]   [mailto:[EMAIL PROTECTED] Em nome de Rhilbert   Rivera   
Enviada em: terça-feira, 5 de junho de 2007   17:30   Para: 
obm-l@mat.puc-rio.br   Assunto: [obm-l] Problema   de maximização
Olá ColegasA solução dada ao   problema abaixo não me convenceu 
(isso pode se dever a minha ignorância   mesmo), por isso peço uma ajuda na 
solução do problema. Quem sabe dessa  vez eu   entenda. Uma fazenda 
de gado permite 20 novilhos por 50 metros   quadrados de pasto. O peso médio 
de seus novilhos no mercado é de 900 kg.   Estimativas do Departamento de 
Agricultura (EUA) indicam que o peso médio   ficará reduzido em 22,5 kg para 
cada novilho que for acrescentado nos 50   metros quadrados de pasto. Quantos 
novilhos devem ser colocados nos 50  metros   quadrados para que o peso 
médio deles seja o maior   possível?Obrigado[ ]'s 
Obtenha o novo Windows Live Messenger! Experimente!--   
_
Instale o novo Windows Live Messenger! É grátis!
http://get.live.com/messenger/overview

Re: [obm-l] RE: [obm-l] RES: [obm-l] Problema de maximiza�

2007-06-06 Por tôpico edneiramaral
 X é o número total de novilhos. E não o número de novilhos q excedem os 
20... 

O modelo q usei pro P só vale pra x = 20. É basado no texto q fala: 
- Permite 20 novilhos. 
- A cada novilho acrescentado, o peso médio (nesse caso, o peso médio entre 
os novilhos) cai 22,5 kg. 

Ou seja, até x=20, o peso médio por novilho é 900 kg. Após isso, perdem-se 
22,5 Kg por cada novilho q exceder os 20, ou seja, perdem-se 22,5*(x-20). 
Logo: 
P = 900 - 22,5*(x-20) 

Na sua conta, tem um erro. O coeficiente de x é (900 + 450) e não (900 - 
450). Corrigindo isso, o x q maximiza a função é 30 mesmo, como tinha 
falado. 

Falou! 

Em (00:08:55), obm-l@mat.puc-rio.br escreveu: 


Interessante esse seu raciocínio do pesso com relação à área. Não havia 
pensado nisso... 
 
Não consigo entender o modelo feito para o peso de cada novilho: P = 900 - 
22,5(x-20). 
 
x seriam os novilhos que se acrescenta no pasto além dos 20 que já estão 
lá? Se for isso, quando se acrescenta 1 novilho, por exemplo, vai se ter um 
ganho no peso e não perda, como o problema diz 
 
Caso esse modelo esteja correto, o que estaria errado se eu fizesse 
 
P = x[900 - 22,5(x-20)] 
 
P = 450x- 22,5 x^2 
 
Se P'=0 então, x = 10 e P = 2250 ? 
 
Obrigado por qualquer esclarecimento. 
 
[ ] ' s 
 
:31 -0300 
 
 Eu acho q a idéia pode ser maximizar o peso médio com relação a área (e 
não 
 com relação ao número de novilhos). 
 Vc coloca um novilho, o peso médio com relação a área pode aumentar ou 
 diminuir. Até os 20, qnd não há perda, com certeza aumenta... 
 
 Resolveria assim: 
 
 Número de novilhos: x 
 Peso de cada novilho (considerando 20 ou mais): P = 900 -22,5*(x-20) Kg 
 Peso médio na área: Pm = x*P/50 
 
 Maximizando o peso médio na área, temos x=30, P = 675 e Pm = 20250. 
 
 Era isso? 
 
 Em (15:28:34), obm-l@mat.puc-rio.br escreveu: 
 
 
 Este enunciado deve estar errado. Da 
 maneira como foi formulado, o peso médio decresce com o número de 
 novilhos e o ideal é colocar so 1 novilho, jah que peso medio para 0 
 novilhos nao eh definido. 
  
  Artur 
  
  -Mensagem 
  original- 
  De: [EMAIL PROTECTED] 
  [mailto:[EMAIL PROTECTED] Em nome de Rhilbert 
  Rivera 
  Enviada em: terça-feira, 5 de junho de 2007 
  17:30 
  Para: obm-l@mat.puc-rio.br 
  Assunto: [obm-l] Problema 
  de maximização 
  
 Olá Colegas 
  
 A solução dada ao 
  problema abaixo não me convenceu (isso pode se dever a minha ignorância 
  mesmo), por isso peço uma ajuda na solução do problema. Quem sabe dessa 
 vez eu 
  entenda. 
  
  Uma fazenda de gado permite 20 novilhos por 50 metros 
  quadrados de pasto. O peso médio de seus novilhos no mercado é de 900 
kg. 
  Estimativas do Departamento de Agricultura (EUA) indicam que o peso 
médio 
  ficará reduzido em 22,5 kg para cada novilho que for acrescentado nos 
50 
  metros quadrados de pasto. Quantos novilhos devem ser colocados nos 50 
 metros 
  quadrados para que o peso médio deles seja o maior 
  possível? 
  
 Obrigado 
  
 [ ]'s 
  
  Obtenha o novo Windows Live Messenger! Experimente! 
  
 -- 
 
 
 
 Instale o novo Windows Live Messenger! É grátis! Experimente! 
 
-- 




Re: RES: [obm-l] Problema sobre valor minimo

2007-05-15 Por tôpico Bruno Carvalho
Arthur e demais amigos da lista. mais uma vez agradeço a atenção e a 
consideração de vocês.
   
  Muito obrigado.
   
  Um abraço grande.
   
  Bruno
   
  

Artur Costa Steiner [EMAIL PROTECTED] escreveu:
  Uma forma de resolver isso eh por multiplicadores de Lagrange.
   
  Seja g(x,y,z) = x + y + z - L ( x.y^2.z^3 - 864). L eh o multiplicador de 
Lagrange
   
  Igualando a 0 as derivadas parciais de g com relacao a x, y, z e L, obtemos
   
  1 - L y^2 z^3 =0
  1 - 2L xy z^3 =0
  1 - 3L x y^2 z^2 =0
   x.y^2.z^3 - 864 = 0 
   
  Da 1a equacao, L = 1/(y^2z^3). Substituindo nas demais, vem
   
  1 - 2 x/y = 0 = y = 2x
   
  1 - 3x/z = 0 = z = 3x
   
  Substituindo na ultima, vem entao
   
  x . 4 x^2 . 27 x^3 = 864 = 108 x^6 = 864 = x^6 = 8 = x = raiz(2), y = 2 
raiz(2), z = 3 raiz(2) 
   
  Veja que, mantendo y constante num valor positivo e fazendo x - oo, podemos 
sempre encontrar um valor positivo para z tal que  x.y^2.z^3 =  864. Assim, 
atendendo-se à restricao, eh possivel  fazer x + y + z - oo. Desta forma, a 
solucao encontrada nao eh maximo global, o problema nao tem isso. A solucao 
encontrada eh unica e as diferenciabilidades da funcao objetivo e da restricao 
indicam que, se houvesse outro ponto extremo, haveria outra solucao para o 
sistema acima (o que fizemos foi anular o chamado Lagrangeano).  Como x + y + z 
0 para todos (x,y,z) viaveis, a funcao objetivo tem infimo. Acho que isso 
nospermiter garantir que eh minimo global sem entrarmos na matriz Hessiana. 
   
  Assim, o valor minimo eh 6 raiz(2). Eh possivel que haja uma outra solucao 
sem usar o calculo, talvez ateh mais facil
   
  Artur
   
   
   
   
  l
   
   
   
  
[Artur Costa Steiner] 
   sagem original-
De: [EMAIL PROTECTED] [mailto:[EMAIL PROTECTED] nome de Bruno Carvalho
Enviada em: quinta-feira, 10 de maio de 2007 13:06
Para: obm-l@mat.puc-rio.br
Assunto: [obm-l] Problema sobre valor minimo


Peço ajuda na resolução do seguinte problema.
   
  Se x,y e z são números reais positivos  e  x.y^2.z^3 = 864 , qual o mínimo 
valor possível para x+y+z ?
   
  Opções:
  a)6 raiz de 2
  b)4raiz de três
  c)9
  d)6raiz de três.
   
  Desde já agradeço a ajuda.
   
  Bruno
  __
Fale com seus amigos de graça com o novo Yahoo! Messenger 
http://br.messenger.yahoo.com/ 


 __
Fale com seus amigos  de graça com o novo Yahoo! Messenger 
http://br.messenger.yahoo.com/ 

RES: [obm-l] Problema sobre valor minimo

2007-05-10 Por tôpico Artur Costa Steiner
Uma forma de resolver isso eh por multiplicadores de Lagrange.
 
Seja g(x,y,z) = x + y + z - L ( x.y^2.z^3 - 864). L eh o multiplicador de 
Lagrange
 
Igualando a 0 as derivadas parciais de g com relacao a x, y, z e L, obtemos
 
1 - L y^2 z^3 =0
1 - 2L xy z^3 =0
1 - 3L x y^2 z^2 =0
 x.y^2.z^3 - 864 = 0 
 
Da 1a equacao, L = 1/(y^2z^3). Substituindo nas demais, vem
 
1 - 2 x/y = 0 = y = 2x
 
1 - 3x/z = 0 = z = 3x
 
Substituindo na ultima, vem entao
 
x . 4 x^2 . 27 x^3 = 864 = 108 x^6 = 864 = x^6 = 8 = x = raiz(2), y = 2 
raiz(2), z = 3 raiz(2) 
 
Veja que, mantendo y constante num valor positivo e fazendo x - oo, podemos 
sempre encontrar um valor positivo para z tal que  x.y^2.z^3 =  864. Assim, 
atendendo-se à restricao, eh possivel  fazer x + y + z - oo. Desta forma, a 
solucao encontrada nao eh maximo global, o problema nao tem isso. A solucao 
encontrada eh unica e as diferenciabilidades da funcao objetivo e da restricao 
indicam que, se houvesse outro ponto extremo, haveria outra solucao para o 
sistema acima (o que fizemos foi anular o chamado Lagrangeano).  Como x + y + z 
0 para todos (x,y,z) viaveis, a funcao objetivo tem infimo. Acho que isso 
nospermiter garantir que eh minimo global sem entrarmos na matriz Hessiana. 
 
Assim, o valor minimo eh 6 raiz(2). Eh possivel que haja uma outra solucao sem 
usar o calculo, talvez ateh mais facil
 
Artur
 
 
 
 
l
 
 
 

[Artur Costa Steiner] 
 sagem original-
De: [EMAIL PROTECTED] [mailto:[EMAIL PROTECTED] nome de Bruno Carvalho
Enviada em: quinta-feira, 10 de maio de 2007 13:06
Para: obm-l@mat.puc-rio.br
Assunto: [obm-l] Problema sobre valor minimo



Peço ajuda na resolução do seguinte problema.
 
Se x,y e z são números reais positivos  e  x.y^2.z^3 = 864 , qual o mínimo 
valor possível para x+y+z ?
 
Opções:
a)6 raiz de 2
b)4raiz de três
c)9
d)6raiz de três.
 
Desde já agradeço a ajuda.
 
Bruno

__
Fale com seus amigos de graça com o novo Yahoo! Messenger 
http://br.messenger.yahoo.com/ 



Re: RES: [obm-l] Problema sobre valor minimo

2007-05-10 Por tôpico Rafael

Gostaria de saber porque nao da certo resolver desse jeito (sei que
nao da, por causa da resposta acima, que deve ser a correta):

S = x+y+z = 3(raiz cubica de xyz) (media aritm =media geom.)
igualdade em x=y=z
S = 3x
x . y^2 . z^3 = x^6 = 864
S = 3(raiz sexta de 864) ~ 9,26 que nao é a resposta certa.


On 5/10/07, Artur Costa Steiner [EMAIL PROTECTED] wrote:

Uma forma de resolver isso eh por multiplicadores de Lagrange.

Seja g(x,y,z) = x + y + z - L ( x.y^2.z^3 - 864). L eh o multiplicador de
Lagrange

Igualando a 0 as derivadas parciais de g com relacao a x, y, z e L, obtemos

1 - L y^2 z^3 =0
1 - 2L xy z^3 =0
1 - 3L x y^2 z^2 =0
 x.y^2.z^3 - 864 = 0

Da 1a equacao, L = 1/(y^2z^3). Substituindo nas demais, vem

1 - 2 x/y = 0 = y = 2x

1 - 3x/z = 0 = z = 3x

Substituindo na ultima, vem entao

x . 4 x^2 . 27 x^3 = 864 = 108 x^6 = 864 = x^6 = 8 = x = raiz(2), y = 2
raiz(2), z = 3 raiz(2)

Veja que, mantendo y constante num valor positivo e fazendo x - oo, podemos
sempre encontrar um valor positivo para z tal que  x.y^2.z^3 =  864. Assim,
atendendo-se à restricao, eh possivel  fazer x + y + z - oo. Desta forma, a
solucao encontrada nao eh maximo global, o problema nao tem isso. A solucao
encontrada eh unica e as diferenciabilidades da funcao objetivo e da
restricao indicam que, se houvesse outro ponto extremo, haveria outra
solucao para o sistema acima (o que fizemos foi anular o chamado
Lagrangeano).  Como x + y + z 0 para todos (x,y,z) viaveis, a funcao
objetivo tem infimo. Acho que isso nospermiter garantir que eh minimo global
sem entrarmos na matriz Hessiana.

Assim, o valor minimo eh 6 raiz(2). Eh possivel que haja uma outra solucao
sem usar o calculo, talvez ateh mais facil

Artur




l




[Artur Costa Steiner]
 sagem original-
De: [EMAIL PROTECTED] [mailto:[EMAIL PROTECTED] nome de
Bruno Carvalho
Enviada em: quinta-feira, 10 de maio de 2007 13:06
Para: obm-l@mat.puc-rio.br
Assunto: [obm-l] Problema sobre valor minimo



Peço ajuda na resolução do seguinte problema.

Se x,y e z são números reais positivos  e  x.y^2.z^3 = 864 , qual o mínimo
valor possível para x+y+z ?

Opções:
a)6 raiz de 2
b)4raiz de três
c)9
d)6raiz de três.

Desde já agradeço a ajuda.

Bruno

__
Fale com seus amigos de graça com o novo Yahoo! Messenger
http://br.messenger.yahoo.com/





--
-
 RAFAEL

=
Instruções para entrar na lista, sair da lista e usar a lista em
http://www.mat.puc-rio.br/~nicolau/olimp/obm-l.html
=


Re: RES: [obm-l] Problema sobre valor minimo

2007-05-10 Por tôpico Henrique Rennó

On 5/10/07, Rafael [EMAIL PROTECTED] wrote:


Gostaria de saber porque nao da certo resolver desse jeito (sei que
nao da, por causa da resposta acima, que deve ser a correta):

S = x+y+z = 3(raiz cubica de xyz) (media aritm =media geom.)
igualdade em x=y=z



Por que você considera x=y=z ???

S = 3x

x . y^2 . z^3 = x^6 = 864
S = 3(raiz sexta de 864) ~ 9,26 que nao é a resposta certa.


On 5/10/07, Artur Costa Steiner [EMAIL PROTECTED] wrote:
 Uma forma de resolver isso eh por multiplicadores de Lagrange.

 Seja g(x,y,z) = x + y + z - L ( x.y^2.z^3 - 864). L eh o multiplicador
de
 Lagrange

 Igualando a 0 as derivadas parciais de g com relacao a x, y, z e L,
obtemos

 1 - L y^2 z^3 =0
 1 - 2L xy z^3 =0
 1 - 3L x y^2 z^2 =0
  x.y^2.z^3 - 864 = 0

 Da 1a equacao, L = 1/(y^2z^3). Substituindo nas demais, vem

 1 - 2 x/y = 0 = y = 2x

 1 - 3x/z = 0 = z = 3x

 Substituindo na ultima, vem entao

 x . 4 x^2 . 27 x^3 = 864 = 108 x^6 = 864 = x^6 = 8 = x = raiz(2), y =
2
 raiz(2), z = 3 raiz(2)

 Veja que, mantendo y constante num valor positivo e fazendo x - oo,
podemos
 sempre encontrar um valor positivo para z tal que  x.y^2.z^3 =  864.
Assim,
 atendendo-se à restricao, eh possivel  fazer x + y + z - oo. Desta
forma, a
 solucao encontrada nao eh maximo global, o problema nao tem isso. A
solucao
 encontrada eh unica e as diferenciabilidades da funcao objetivo e da
 restricao indicam que, se houvesse outro ponto extremo, haveria outra
 solucao para o sistema acima (o que fizemos foi anular o chamado
 Lagrangeano).  Como x + y + z 0 para todos (x,y,z) viaveis, a funcao
 objetivo tem infimo. Acho que isso nospermiter garantir que eh minimo
global
 sem entrarmos na matriz Hessiana.

 Assim, o valor minimo eh 6 raiz(2). Eh possivel que haja uma outra
solucao
 sem usar o calculo, talvez ateh mais facil

 Artur




 l




 [Artur Costa Steiner]
  sagem original-
 De: [EMAIL PROTECTED] [mailto:[EMAIL PROTECTED]
nome de
 Bruno Carvalho
 Enviada em: quinta-feira, 10 de maio de 2007 13:06
 Para: obm-l@mat.puc-rio.br
 Assunto: [obm-l] Problema sobre valor minimo



 Peço ajuda na resolução do seguinte problema.

 Se x,y e z são números reais positivos  e  x.y^2.z^3 = 864 , qual o
mínimo
 valor possível para x+y+z ?

 Opções:
 a)6 raiz de 2
 b)4raiz de três
 c)9
 d)6raiz de três.

 Desde já agradeço a ajuda.

 Bruno

 __
 Fale com seus amigos de graça com o novo Yahoo! Messenger
 http://br.messenger.yahoo.com/




--
-
  RAFAEL

=
Instruções para entrar na lista, sair da lista e usar a lista em
http://www.mat.puc-rio.br/~nicolau/olimp/obm-l.html
=





--
Henrique


Re: RES: [obm-l] Problema sobre valor minimo

2007-05-10 Por tôpico Rafael

Mas se MA=MG seu valor minimo é MG. Preciso da igualdade, que ocorre
se x=y=z , nao é?

On 5/10/07, Henrique Rennó [EMAIL PROTECTED] wrote:

On 5/10/07, Rafael [EMAIL PROTECTED] wrote:

 Gostaria de saber porque nao da certo resolver desse jeito (sei que
 nao da, por causa da resposta acima, que deve ser a correta):

 S = x+y+z = 3(raiz cubica de xyz) (media aritm =media geom.)
 igualdade em x=y=z


Por que você considera x=y=z ???

S = 3x
 x . y^2 . z^3 = x^6 = 864
 S = 3(raiz sexta de 864) ~ 9,26 que nao é a resposta certa.


 On 5/10/07, Artur Costa Steiner [EMAIL PROTECTED] wrote:
  Uma forma de resolver isso eh por multiplicadores de Lagrange.
 
  Seja g(x,y,z) = x + y + z - L ( x.y^2.z^3 - 864). L eh o multiplicador
 de
  Lagrange
 
  Igualando a 0 as derivadas parciais de g com relacao a x, y, z e L,
 obtemos
 
  1 - L y^2 z^3 =0
  1 - 2L xy z^3 =0
  1 - 3L x y^2 z^2 =0
   x.y^2.z^3 - 864 = 0
 
  Da 1a equacao, L = 1/(y^2z^3). Substituindo nas demais, vem
 
  1 - 2 x/y = 0 = y = 2x
 
  1 - 3x/z = 0 = z = 3x
 
  Substituindo na ultima, vem entao
 
  x . 4 x^2 . 27 x^3 = 864 = 108 x^6 = 864 = x^6 = 8 = x = raiz(2), y =
 2
  raiz(2), z = 3 raiz(2)
 
  Veja que, mantendo y constante num valor positivo e fazendo x - oo,
 podemos
  sempre encontrar um valor positivo para z tal que  x.y^2.z^3 =  864.
 Assim,
  atendendo-se à restricao, eh possivel  fazer x + y + z - oo. Desta
 forma, a
  solucao encontrada nao eh maximo global, o problema nao tem isso. A
 solucao
  encontrada eh unica e as diferenciabilidades da funcao objetivo e da
  restricao indicam que, se houvesse outro ponto extremo, haveria outra
  solucao para o sistema acima (o que fizemos foi anular o chamado
  Lagrangeano).  Como x + y + z 0 para todos (x,y,z) viaveis, a funcao
  objetivo tem infimo. Acho que isso nospermiter garantir que eh minimo
 global
  sem entrarmos na matriz Hessiana.
 
  Assim, o valor minimo eh 6 raiz(2). Eh possivel que haja uma outra
 solucao
  sem usar o calculo, talvez ateh mais facil
 
  Artur
 
 
 
 
  l
 
 
 
 
  [Artur Costa Steiner]
   sagem original-
  De: [EMAIL PROTECTED] [mailto:[EMAIL PROTECTED]
 nome de
  Bruno Carvalho
  Enviada em: quinta-feira, 10 de maio de 2007 13:06
  Para: obm-l@mat.puc-rio.br
  Assunto: [obm-l] Problema sobre valor minimo
 
 
 
  Peço ajuda na resolução do seguinte problema.
 
  Se x,y e z são números reais positivos  e  x.y^2.z^3 = 864 , qual o
 mínimo
  valor possível para x+y+z ?
 
  Opções:
  a)6 raiz de 2
  b)4raiz de três
  c)9
  d)6raiz de três.
 
  Desde já agradeço a ajuda.
 
  Bruno
 
  __
  Fale com seus amigos de graça com o novo Yahoo! Messenger
  http://br.messenger.yahoo.com/
 
 


 --
 -
   RAFAEL

 =
 Instruções para entrar na lista, sair da lista e usar a lista em
 http://www.mat.puc-rio.br/~nicolau/olimp/obm-l.html
 =




--
Henrique




--
-
 RAFAEL

=
Instruções para entrar na lista, sair da lista e usar a lista em
http://www.mat.puc-rio.br/~nicolau/olimp/obm-l.html
=


RES: [obm-l] Problema

2007-01-30 Por tôpico Ralph Teixeira
Eu chutei o balde: fiz um diagrama com todas as possibilidades de vizinhos, 
ficou assim (use texto de largura fixa para ver isto):

-- -- -- -- -- -- -- 01 08 15 22 29 36 43
-- -- -- -- -- -- 03 10 17 24 31 38 45
-- -- -- -- -- 05 12 19 26 33 40 47
-- -- -- -- 07 14 21 28 35 42 49
-- -- 02 09 16 23 30 37 44
-- 04 11 18 25 32 39 46
06 13 20 27 34 41 48
15 22 29 36 43

(Note que a última linha repete um pedaço da lá de cima).
Agora é fazer um caminho com segmentos horizontais ou verticais andando pelas 
células numeradas acima, passando por cada célula apenas uma vez.

Se eu me lembro bem, este é o clássico problema de encontrar um caminho 
Hamiltoniano para um grafo. E clássico não é fácil -- apesar de muito 
pesquisado, encontrar este caminho num grafo geral é um problema NP ou 
NP-completo ou algo assim (eu não sei quase nada de Teoria dos Grafos; quem 
souber mais, como o Nicolau, por exemplo, pode me corrigir). :)

Mas nosso problema não é o caso geral, então há esperança: comece olhando os 
cantos, onde só há uma opção (por exemplo, só passo pelo 05 se for fazendo 12 
05 14)... faça o desenho com as conexões, marque as absolutamente necessárias 
por causa dos cantos, corte conexões com números que já foram gastos, aparecem 
novos cantos que só tem uma opção, etc. Fica melhor no papel do que no 
computador.

Quem quiser tentar agora por conta própria, pare de ler aqui...

Até este diagrama abaixo, não há opções (hmmm mentira, um dos tais cantos 
poderia ser uma ponta... vou ignorar isto e torcer para achar UMA resposta):
   ||||||
 01=08 15 22=29 36=43
 ||   ||||
  03=10 17 24=31 38=45
  || ||
   05=12 19=26 33 40=47
   ||||   ||
07=14 21=28 35 42=49
||||||
  02=09 16 23=30 37=44
  || ||
   04=11 18 25 32 39=46
   || ||
06=13 20=27 34 41=48
||||||
15 22=29 36=43

Agora há opções eu tentei juntar 09 com 16 e me dei mal... então tentei 
juntar 09 com 18 e 41 com 32... Fiz umas outras escolhas arbitrárias, mantendo 
a simetria por razões puramente estéticas, e cheguei numa possibilidade 
bonitinha:

   ||||||
 01=08=15 22=29 36=43
 ||   ||||
  03=10 17=24=31 38=45
  || ||
   05=12 19=26=33 40=47
   ||||   ||
07=14 21=28 35=42=49
||||||
  02=09 16 23=30 37=44
  || || || ||||
   04=11 18 25 32 39=46
   |||| || || ||
06=13 20=27 34 41=48
||||||

Então uma solução possível é:
17-24-31-22-29-20-27-18-09-02-11-04-13-06-15-08-01-10-03-12-05-14-07-16-25)
25)-34-43-36-45-38-47-40-49-42-35-44-37-46-39-48-41-32-23-30-21-28-19-26-33.

Uff!

Perguntas adicionais:

i) Há outras respostas? (Sim: troque 17-24 e 26-33 por 17-26 e 24-33...)
ii) 17 e 33 têm de ser as pontas? (Não: troque 49-40 por 40-33 e você tem um 
outro caminho; aliás, trocando também 1-10 por 10-17, dá uma solução COMEÇANDO 
no 1 e TERMINANDO no 49!! Vou escrever esta só de farra:

01-08-15-06-13-04-11-02-09-18-27-20-29-22-31-24-17-10-03-12-05-14-07-16-25)
(25-34-43-36-45-38-47-40-33-26-19-28-21-30-23-32-41-48-39-46-37-44-35-42-49

Legal!

Abraço,
Ralph

-Mensagem original-
De: [EMAIL PROTECTED] [mailto:[EMAIL PROTECTED] nome de Carlos Gomes
Enviada em: segunda-feira, 29 de janeiro de 2007 14:44
Para: obm-l@mat.puc-rio.br
Assunto: [obm-l] Problema


Vê se alguém tem alguma sugestão para essa questao;
Disponha em linha reta, numa ordem, os números inteiros de 1 até 49, de modo 
que o valor absoluto da diferença de quaisquer dois vizinhos, nessa ordem, seja 
ou 7 ou 9.

Obg
C.Gomes

=
Instruções para entrar na lista, sair da lista e usar a lista em
http://www.mat.puc-rio.br/~nicolau/olimp/obm-l.html
=


RES: [obm-l] Problema de teoria dos numeros

2006-12-05 Por tôpico Artur Costa Steiner
Oi Paulo, 
Vc nao tinha que considera tambem os numeros impares?
 
A prova que eu encontrei foi a seguinte:
 
Suponhamos que n seja impar. Entao a(n) = 2^n +1 eh divisivel por 3.
Para n=1, a(n) =3 e a condicao eh satisfeita. Suponhamos que, para algum
impar n, a(n) seja multiplo de 3. Para o impar subsequente, n+2, temos que
a(n+2) = 2^(n+2) + 1 = 4* 2^n + 1 = 4(a(n) -1) + 1 = 4a(n) - 3. Dado que,
pela hipotese indutiva, 3|a(n), temos que 3|a(n+2), completando-se assim a
prova.
 
Suponhamos agora quer n seja par. Ai vale a sua solucao, alias muito bonita.
Uma outra possibilidade, noa tao bo quanto a sua, eh a seguinte: Se n for da
forma n=2k, com k impar, entao a(n) = 4^k +1. Potências impares de 4 tem, na
base decimal,  algarismo das unidades 4. Logo a(n) = 4^k + 1 tem algarismo
das unidades 5, sendo portanto divisivel por 5.   
 
Vemos assim que, se a(n) for primo e n se enquadrar num dos casos acima,
entao n=1 = 2^0 ou n= 2*1 = 2, casos em que n eh potencia de 2
(considerando-se 1 como potencia 0 de 2). Nos casos acima, outros valores de
n levam a numeros compostos. Se, a(n) for primo e n nao se enquadrar nos
casos acima, entao n eh par e não eh multiplo de nenhum impar 1, sendo
portanto potencia de 2. Isso conclui a prova.
 
Uma outra prova da infinitude dos primos eh a seguinte:  Para todo
n=1,2,3, n! eh divisivel por 2,3.n. Entao, nenhum destes numeros
divide n! + 1. Pelo teorema fundamental da aritmetica, n! + 1 pode ser
representado por um produto de primos, dentre os quais, em virtude do que
vimos, não se enquadra nenhum primo = n. Logo, para todo n existe um primo
p n, do que concluimos que o conjunto dos primos eh ilimitado e, portanto,
infinito.
 
Abracos
Artur
  

-Mensagem original-
De: [EMAIL PROTECTED] [mailto:[EMAIL PROTECTED] nome de
Paulo Santa Rita
Enviada em: segunda-feira, 4 de dezembro de 2006 21:46
Para: obm-l@mat.puc-rio.br
Assunto: RE: [obm-l] Problema de teoria dos numeros


Ola carissimo Artur e demais
colegas desta lista ... OBM-L,

Seja M um primo tal que M = (2^N) + 1 e suponhamos que N nao e potencia de
2. Neste caso N e da forma : (2^P)*i, onde P e um inteiro nao-negativo e i
um impar maior que 1. Segue daqui que M = (2^A)^ i  + 1 com A= 2^P . Fazendo
2^A = X teremos que M = X^i + 1. Este polinomio e claramente divisivel  por
X + 1 em virtude do teorema D'Alembert, pois  sendo i impar temos que
(-1)^i + 1 = 0. Assim :  M = X^i + 1 = (X + 1)*Q(X)   =  M nao e primo  ...
ABSURDO  ! 

A nossa tese e portanto insustentavel e somos obrigados a admitir que N e
potencia de 2, como queriamos demonstrar. Eis aqui outro bonitinho, porem
nao tao simples como este : (Fermat propoe, Euler resolve ) Mostre que  a
equacao X^3 = Y^2 + 2 tem uma unica solucao no anel dos inteiros.

Estas questoes de Teoria dos Numeros me levaram a alguns anos atras, quando
eu me correspondia sobre topologia com um colega que esta atualmente fazendo
doutorado em Bio-Matematica na Alemanha. Ele conclui o doutorado agora. Mas
o que importa e que naquela epoca, quando ele ainda fazia Mestrado na
Unicamp, nos combinamos que em cada carta era obrigatorio haver uma prova da
existencia de uma infinidade de numeros primos. O Marcelo mostrou uma prova
muito simples, mas belissima e que eu passo pra vocês :

EXISTEM INFINITOS NUMEROS PRIMOS :

Suponha que a quantidade de numeros primos e finita. Digamos : p1  p2  ...
 pn. Consideremos agora o numero P=p1*p2*...*pn, claramente maior que
qualquer dos primos pi. O numero P - 1 e portanto composto. Segue que existe
pi que divide  P - 1. Mas pi tambem divide P, logo, pi deve dividir P - (P -
1 ) = 1 ... ABSURDO !

A todos,  com  os melhores
votos de paz profunda, sou
Paulo Santa Rita
1,1540,041206







  _  

From: [EMAIL PROTECTED]
To: obm-l@mat.puc-rio.br
Subject: [obm-l] Problema de teoria dos numeros
Date: Mon, 4 Dec 2006 20:14:35 -0200


Achei este problema de teoris dos numeros (nao eh dos mais dificeis) bem
bonitinho.
 
Mostre que, se 2^n +1, n=0, 1,2for primo, entao n eh potencia de 2 
 
Artur


  _  

Ligue para os seus amigos grátis. Faça chamadas de PC-para-PC pelo
messenger-- GRÁTIS Experimente agora!
http://get.live.com/messenger/overview  



RE: RES: [obm-l] Problema de teoria dos numeros

2006-12-05 Por tôpico Paulo Santa Rita

Ola carissimo artur e demais
colegas desta lista ... OBM-L,

O Ronaldo Alonso estava com a mesma duvida  e eu enviei um esclarecimento mas 
nao deve ter chegado na lista. Quando eu disse que se N nao e potencia de 2 
entao N e da forma (2^P)*i com P INTEIRO NAO-NEGATIVO e i um impar maior que 
1 estou admitindo P=0 para incluir todos os impares. Exemplo : N=13 = 
N=(2^0)*13 ; N=28 = N=(2^2)*7.

Um Abracao
Paulo Santa Rita
3,1100,051206


 From: [EMAIL PROTECTED]
 To: obm-l@mat.puc-rio.br
 Subject: RES: [obm-l] Problema de teoria dos numeros
 Date: Tue, 5 Dec 2006 10:34:55 -0200
 
 Oi Paulo,
 Vc nao tinha que considera tambem os numeros impares?
 A prova que eu encontrei foi a seguinte:
 Suponhamos que n seja impar. Entao a(n) = 2^n +1 eh divisivel por 3.
 Para n=1, a(n) =3 e a condicao eh satisfeita. Suponhamos que, para algum 
 impar n, a(n) seja multiplo de 3. Para o impar subsequente, n+2, temos que 
 a(n+2) = 2^(n+2) + 1 = 4* 2^n + 1 = 4(a(n) -1) + 1 = 4a(n) - 3. Dado que, 
 pela hipotese indutiva, 3|a(n), temos que 3|a(n+2), completando-se assim a 
 prova.
 Suponhamos agora quer n seja par. Ai vale a sua solucao, alias muito bonita. 
 Uma outra possibilidade, noa tao bo quanto a sua, eh a seguinte: Se n for da 
 forma n=2k, com k impar, entao a(n) = 4^k +1. Potências impares de 4 tem, na 
 base decimal,  algarismo das unidades 4. Logo a(n) = 4^k + 1 tem algarismo 
 das unidades 5, sendo portanto divisivel por 5.
 Vemos assim que, se a(n) for primo e n se enquadrar num dos casos acima, 
 entao n=1 = 2^0 ou n= 2*1 = 2, casos em que n eh potencia de 2 
 (considerando-se 1 como potencia 0 de 2). Nos casos acima, outros valores de 
 n levam a numeros compostos. Se, a(n) for primo e n nao se enquadrar nos 
 casos acima, entao n eh par e não eh multiplo de nenhum impar 1, sendo 
 portanto potencia de 2. Isso conclui a prova.
 Uma outra prova da infinitude dos primos eh a seguinte:  Para todo 
 n=1,2,3, n! eh divisivel por 2,3.n. Entao, nenhum destes numeros 
 divide n! + 1. Pelo teorema fundamental da aritmetica, n! + 1 pode ser 
 representado por um produto de primos, dentre os quais, em virtude do que 
 vimos, não se enquadra nenhum primo = n. Logo, para todo n existe um primo p 
 n, do que concluimos que o conjunto dos primos eh ilimitado e, portanto, 
 infinito.
 Abracos
 Artur
 -Mensagem original-
 De: [EMAIL PROTECTED] [mailto:[EMAIL PROTECTED] nome de Paulo Santa Rita
 Enviada em: segunda-feira, 4 de dezembro de 2006 21:46
 Para: obm-l@mat.puc-rio.br
 Assunto: RE: [obm-l] Problema de teoria dos numeros
 Ola carissimo Artur e demais
 colegas desta lista ... OBM-L,
 Seja M um primo tal que M = (2^N) + 1 e suponhamos que N nao e potencia de 2. 
 Neste caso N e da forma : (2^P)*i, onde P e um inteiro nao-negativo e i um 
 impar maior que 1. Segue daqui que M = (2^A)^ i  + 1 com A= 2^P . Fazendo 2^A 
 = X teremos que M = X^i + 1. Este polinomio e claramente divisivel  por X + 1 
 em virtude do teorema D'Alembert, pois  sendo i impar temos que (-1)^i + 1 
 = 0. Assim :  M = X^i + 1 = (X + 1)*Q(X)   =  M nao e primo  ... ABSURDO  !
 A nossa tese e portanto insustentavel e somos obrigados a admitir que N e 
 potencia de 2, como queriamos demonstrar. Eis aqui outro bonitinho, porem nao 
 tao simples como este : (Fermat propoe, Euler resolve ) Mostre que  a equacao 
 X^3 = Y^2 + 2 tem uma unica solucao no anel dos inteiros.
 Estas questoes de Teoria dos Numeros me levaram a alguns anos atras, quando 
 eu me correspondia sobre topologia com um colega que esta atualmente fazendo 
 doutorado em Bio-Matematica na Alemanha. Ele conclui o doutorado agora. Mas o 
 que importa e que naquela epoca, quando ele ainda fazia Mestrado na Unicamp, 
 nos combinamos que em cada carta era obrigatorio haver uma prova da 
 existencia de uma infinidade de numeros primos. O Marcelo mostrou uma prova 
 muito simples, mas belissima e que eu passo pra vocês :
 EXISTEM INFINITOS NUMEROS PRIMOS :
 Suponha que a quantidade de numeros primos e finita. Digamos : p1  p2  ... 
  pn. Consideremos agora o numero P=p1*p2*...*pn, claramente maior que 
 qualquer dos primos pi. O numero P - 1 e portanto composto. Segue que existe 
 pi que divide  P - 1. Mas pi tambem divide P, logo, pi deve dividir P - (P - 
 1 ) = 1 ... ABSURDO !
 A todos,  com  os melhores
 votos de paz profunda, sou
 Paulo Santa Rita
 1,1540,041206
 
 From: [EMAIL PROTECTED]
 To: obm-l@mat.puc-rio.br
 Subject: [obm-l] Problema de teoria dos numeros
 Date: Mon, 4 Dec 2006 20:14:35 -0200
 Achei este problema de teoris dos numeros (nao eh dos mais dificeis) bem 
 bonitinho.
 Mostre que, se 2^n +1, n=0, 1,2for primo, entao n eh potencia de 2
 Artur
 
 Ligue para os seus amigos grátis. Faça chamadas de PC-para-PC pelo 
 messenger-- GRÁTIS Experimente agora!http://get.live.com/messenger/overview

[obm-l] RES: [obm-l] Problema de Cálculo

2006-05-15 Por tôpico Artur Costa Steiner
Facamos f(x) = e^(2x) - k*sqrt(x). Para todo real k, temos que f(0) = 1 e
que f(x) -- oo quando x -- oo.
Se k=0, f eh estritamente postiva em [0, oo). Logo, f so podera admitir
zeros se k0. Temos que f'(x) = 2*e^(2x) - k/(2*sqrt(x). A funcao 2*e^(2x)
eh estritamente crescente em (o, oo) ao passo que k/(2*sqrt(x) eh
estritamente decrescente. A primeira tende a 2 en x=0+ e a segunda tende a
oo. Quando x --- oo, a primeira tende a oo e a sgunda tende a zero.  Assim,
temos f'(a) = 0 para um e somente um a0 e, neste ponto x=a, f passo por um
minimo global.   Se em x=a tivermos f(a) = 0, entao  f se anula neste ponto
e apenas nele.

Temos assim que resolver o sistema de equacoes: 

e^(2a) = k*sqrt(a)
2*e^(2a) =k/(2*sqrt(a))

Dividindo membro a membro, jah que nao sao nulos, temos 1/2 = sqrt(a) * 2 *
sqrt(a) = 2a = a = 1/4.

Logo, e^(1/2) = k * sqrt(1/4) = k = 2*e^(1/2).

Faca um grafico para ver se nao hah nenhum engano.

Artur


-Mensagem original-
De: [EMAIL PROTECTED] [mailto:[EMAIL PROTECTED]
nome de bernardoakino
Enviada em: domingo, 14 de maio de 2006 03:16
Para: obm-l@mat.puc-rio.br
Assunto: [obm-l] Problema de Cálculo


Caros colegas da lista, 
   Eu não sei se esse problema já foi discutido anteriormente aqui, mas ele 
esta me tirando algumas horas de sono... Se alguem puder me dar uma 
ajudinha, eu ficaria bastante agradecido: 

Para quais valores de k a equação e^(2x)=k.sqrt(x) tem exatamente uma 
solução? 

Um abraço a todos 
Bernardo 



=
Instruções para entrar na lista, sair da lista e usar a lista em
http://www.mat.puc-rio.br/~nicolau/olimp/obm-l.html
=


Re: [obm-l] RES: [obm-l] RES: [obm-l] Problema dos Remédios

2006-03-08 Por tôpico Chicao Valadares
ótima sacada!!
dah pra facilitar as coisas aqui...
nao precisa por a caixa escolhida na balança basta
enumerar as restantes e tirar os comprimidos do jeito
que o amigo citou...depois olha o valor resultante e
tira o módulo 10.. 
se for 0 é a caixa escolhida..
se for 1 é a caixa numero 9...
...
...
...
se for 9 é a caixa número 1...

O que me chamou atençao é a injetividade
metodo-resposta...isso tah com cara que tem teoria dos
numeros no meio...o nicolau poderia fazer o favor de
formalizar melhor as coisas aqui p/ ficar mais
claro... 



--- David Cardoso [EMAIL PROTECTED] escreveu:

 
 Isso mesmo! Sendo que se a caixa ecolhida fosse
 defeituosa tudo pesaria
 1350g.
 
 Abraço,
 
 David
 
  -Mensagem original-
  De: [EMAIL PROTECTED] 
  [mailto:[EMAIL PROTECTED] Em nome de
 Felipe Avelino
  Enviada em: terça-feira, 7 de março de 2006 19:23
  Para: obm-l@mat.puc-rio.br
  Assunto: Re: [obm-l] RES: [obm-l] Problema dos
 Remédios
  
  ah eh facil!!
   
  escolhe uma caixa qualquer..
  e numera as restantes...
   
   
  tira um comprimido da caixa numero 1 e coloca
 junto tira dois 
  comprimidos da caixa numero 2 ..
  e assim por diante..
  ateh a caixa numero 9
   
  junta todos esses comprimidos e coloca pra pesar
 junto com a 
  caixa escolhida primeiramente
   
  se pesar 1449g  a caixa defeituosa eh a numero 1
 se pesar 
  1448g a caixa defeituosa eh a numero 2 
  se pesar 1450g a caixa defeituosa eh a escolhida .
   
  
   
  Em 07/03/06, David Cardoso
 [EMAIL PROTECTED] escreveu: 
  
  
  Pesar uma vez significa fazer apenas uma leitura
 do 
  peso no visor da
  balança.. uma vez lido qualquer número no visor
 da 
  balança, ela quebra.. :P 
  
  
   -Mensagem original-
   De: [EMAIL PROTECTED]
   [mailto:[EMAIL PROTECTED] ] Em nome de
 
  Chicao Valadares
   Enviada em: terça-feira, 7 de março de 2006
 16:38
   Para: obm-l@mat.puc-rio.br
   Assunto: Re: [obm-l] Problema dos Remédios
   
   Pesar uma unica vez???Se vc supor que o ato de
 pesar é
   colocar alguma coisa e depois tirar, vc somente
 deve ir
   colocando sem tirar cada caixa e se a variaçao
 do peso nao
   for conforme esperado tai sua caixa. 
  
  
   --- David Cardoso [EMAIL PROTECTED]
 escreveu:
  
   
Há 10 caixas de um tipo de remédio, cada
 caixa com 
  100 comprimidos, 
cada comprimido pesando 10g.
Uma(exatamente uma) destas caixas é oriunda
 de um 
  lote defeituoso,
onde os comprimidos pesam 9 g.
Você tem acesso a uma balança digital, que só
 pode ser 
   usada uma vez,
e tem precisão suficiente para lhe dar o
 resultado 
  exato de qqr
pesagem com esses remédios.
   
Qual a sua estratégia de pesagem pra
 determinar, com
   certeza, qual a
caixa de remédio defeituosa?
   
   
   
-- 2a. parte, generalização: ---
   
Qual o número mínimo pesagens necessárias
 para se descobrir 
   k caixas
defeituosas dentro de uma amostragem de n
 caixas?
   
   
   
   
  

==
   === 
Instruções para entrar na lista, sair da
 lista e 
  usar a lista em
   
 http://www.mat.puc-rio.br/~nicolau/olimp/obm-l.html

  

==
   ===
   
  
  
   O Binômio de Newton é tão belo como a Vênus de
 Milo.
   O que há é pouca gente para dar por isso...  
   Fernando Pessoa - Poesias de Alvaro Campos
  
   
 

_
   As informações existentes nessa mensagem e
 no(s) arquivo(s)
   anexado(s) são para uso restrito, sendo seu
 sigilo protegido 
   por lei. Caso não seja destinatário, saiba que
 leitura,
   divulgação ou cópia são proibidas.
   Favor
   apagar as informações e notificar o remetente.
 O uso
   impróprio será tratado conforme as normas da
 empresa e a 
   legislação em vigor. Agradecemos sua
 colaboração.
  
  
   The information mentioned in this message and
 in the archives
   attached are of restricted use, and its privacy
 is protected
   by law. If you are not the addressee, be aware
 that reading, 
   disclosure or copy are forbidden.
   Please
   delete this information and notify the sender.
 Inappropriate
   use will be tracted according to company's
 rules and valid
   laws. Thank you for your cooperation. 
  
  
  
  

___
   Yahoo! Acesso Grátis - Internet rápida e
 grátis. Instale o
   discador agora!
   http://br.acesso.yahoo.com
 http://br.acesso.yahoo.com 
  

==
   ===
   Instruções para entrar na lista

Re: [obm-l] RES: [obm-l] Problema dos Remédi os

2006-03-08 Por tôpico Eduardo Wilner
   A caixa de remédios é defeituosa ou não funciona? Brincadeira... Mas acho que não funciona; por exemplo:(7+11+13)*9+31*10=(7+11+13)*10+31*9.Entretanto, pode ter remédio, pois existem mais do que 10 números  primos entre 6 e 100. Talvez seja o caso de selecionar a decupla que  não possue somas parciais iguais (incluindo, como no exemplo acima, o  número isolado).   David Cardoso [EMAIL PROTECTED] escreveu:  Realmente fica bem mais interessante.Eu tive uma idéia, não tenho certeza se daria certo pra qualquer caso:Enumera todos os primos menores que 100, exceto o 2, 3 e 5 (pq sao fatoresde 10g e 9g).Ou seja, a sequência S seria 7, 11, 17, 23, ...Pesaria S_1 comprimidos da caixa 1, junto com S_2 comprimidos da caixa
 2,..., até S_10.Se eu tiver pensando certo, o resultado da pesagem vai poder ser fatoradoassim:S_1 * K_1 + S_2 * K_2 + ... + S_10 * K_10Cada K_i da fatoração deveria ser ou 9g ou 10g, o que indicaria se a caixacorrespondente é defeituosa ou não.. funciona?![]'sDavid  -Mensagem original- De: [EMAIL PROTECTED]  [mailto:[EMAIL PROTECTED] Em nome de Nicolau C. Saldanha Enviada em: quarta-feira, 8 de março de 2006 07:19 Para: obm-l@mat.puc-rio.br Assunto: Re: [obm-l] Problema dos Remédios  On Tue, Mar 07, 2006 at 02:39:37PM -0300, David Cardoso wrote:Há 10 caixas de um tipo de remédio, cada caixa com 100  comprimidos, cada  comprimido pesando 10g.  Uma(exatamente uma) destas caixas é oriunda de um lote  defeituoso, onde os  comprimidos pesam 9 g. &!
gt; Você
 tem acesso a uma balança digital, que só pode ser  usada uma vez, e tem  precisão suficiente para lhe dar o resultado exato de qqr  pesagem com esses  remédios.Qual a sua estratégia de pesagem pra determinar, com  certeza, qual a caixa  de remédio defeituosa?  Já resolveram o problema como proposto, mas na verdade é  possível resolver uma versão bem mais forte:  Há 10 caixas de um tipo de remédio, cada caixa com 100  comprimidos, cada comprimido pesando 10g. Algumas destas caixas (você não sabe quantas nem quais) são oriundas de um lote defeituoso, onde os comprimidos pesam 9 g. Você tem acesso a uma balança digital, que só pode ser usada  uma vez, e tem precisão suficiente para lhe dar o resultado exato de qqr  pesagem com esses remédios.  Qual a sua es!
tratégia
 de pesagem para determinar, com  certeza, exatamente quais caixas de remédio são defeituosas?  []s, N. == === Instruções para entrar na lista, sair da lista e usar a lista em http://www.mat.puc-rio.br/~nicolau/olimp/obm-l.html == ===  __ NOD32 1.1425 (20060302) Information __  This message was checked by NOD32 antivirus system. http://www.eset.com  =Instruções para entrar na lista, sair da lista e usar a lista emhttp://www.mat.puc-rio.br/~nicolau/olimp/obm-l.html=
		 
Yahoo! Acesso Grátis 
Internet rápida e grátis. Instale o discador agora!

[obm-l] RES: [obm-l] RES: [obm-l] Problema dos Remédios

2006-03-08 Por tôpico David Cardoso

É verdade.. E se uma décupla assim existir? Resolve o problema?

[]'s

 -Mensagem original-
 De: [EMAIL PROTECTED] 
 [mailto:[EMAIL PROTECTED] Em nome de Eduardo Wilner
 Enviada em: quarta-feira, 8 de março de 2006 17:35
 Para: obm-l@mat.puc-rio.br
 Assunto: Re: [obm-l] RES: [obm-l] Problema dos Remédios
 
 
   A caixa de remédios é defeituosa ou não funciona?   Brincadeira...
 
   Mas acho que não funciona; por exemplo:
 
  (7+11+13)*9+31*10=(7+11+13)*10+31*9.
 
  Entretanto, pode ter remédio, pois existem mais do que 10 
 números primos entre 6 e 100. Talvez seja o caso de 
 selecionar a decupla que não possue somas parciais iguais 
 (incluindo, como no exemplo acima, o número isolado). 
 
 
 David Cardoso [EMAIL PROTECTED] escreveu:
 
 
   Realmente fica bem mais interessante.
   Eu tive uma idéia, não tenho certeza se daria certo pra 
 qualquer caso:
   
   Enumera todos os primos menores que 100, exceto o 2, 3 
 e 5 (pq sao fatores
   de 10g e 9g).
   Ou seja, a sequência S seria 7, 11, 17, 23, ...
   
   Pesaria S_1 comprimidos da caixa 1, junto com S_2 
 comprimidos da caixa 2,
   ..., até S_10.
   Se eu tiver pensando certo, o resultado da pesagem vai 
 poder ser fatorado
   assim:
   
   S_1 * K_1 + S_2 * K_2 + ... + S_10 * K_10
   
   Cada K_i da fatoração deveria ser ou 9g ou 10g, o que 
 indicaria se a caixa
   correspondente é defeituosa ou não.. funciona?!
   
   []'s
   
   David
   
   
-Mensagem original-
De: [EMAIL PROTECTED] 
[mailto:[EMAIL PROTECTED] Em nome de 
 Nicolau C. Saldanha
Enviada em: quarta-feira, 8 de março de 2006 07:19
Para: obm-l@mat.puc-rio.br
Assunto: Re: [obm-l] Problema dos Remédios

On Tue, Mar 07, 2006 at 02:39:37PM -0300, David Cardoso wrote:
 
 Há 10 caixas de um tipo de remédio, cada caixa com 100 
comprimidos, cada
 comprimido pesando 10g.
 Uma(exatamente uma) destas caixas é oriunda de um lote 
defeituoso, onde os
 comprimidos pesam 9 g.
! gt; Você tem acesso a uma balança digital, que só pode ser 
usada uma vez, e tem
 precisão suficiente para lhe dar o resultado exato de qqr 
pesagem com esses
 remédios.
 
 Qual a sua estratégia de pesagem pra determinar, com 
certeza, qual a caixa
 de remédio defeituosa?

Já resolveram o problema como proposto, mas na verdade é 
possível resolver
uma versão bem mais forte:

Há 10 caixas de um tipo de remédio, cada caixa com 100 
comprimidos, cada
comprimido pesando 10g.
Algumas destas caixas (você não sabe quantas nem 
 quais) são oriundas
de um lote defeituoso, onde os comprimidos pesam 9 g.
Você tem acesso a uma balança digital, que só pode ser usada 
uma vez, e tem
precisão suficiente para lhe dar o resultado exato de qqr 
pesagem com esses
remédios.

Qual a sua es! tratégia de pesagem para determinar, com 
certeza, exatamente
quais caixas de remédio são defeituosas?

[]s, N.
==
===
Instruções para entrar na lista, sair da lista e usar 
 a lista em
http://www.mat.puc-rio.br/~nicolau/olimp/obm-l.html
==
===

__ NOD32 1.1425 (20060302) Information __

This message was checked by NOD32 antivirus system.
http://www.eset.com


   
   
   
   
 ==
 ===
   Instruções para entrar na lista, sair da lista e usar a lista em
   http://www.mat.puc-rio.br/~nicolau/olimp/obm-l.html
   
 ==
 ===
   
 
 
 
 
 Yahoo! Acesso Grátis
 Internet rápida e grátis. Instale o discador agora! 
 http://us.rd.yahoo.com/mail/br/tagline/homepage_set/*http://b
r.acesso.yahoo.com 
 



=
Instruções para entrar na lista, sair da lista e usar a lista em
http://www.mat.puc-rio.br/~nicolau/olimp/obm-l.html
=


[obm-l] RES: [obm-l] Problema dos Remédios

2006-03-07 Por tôpico David Cardoso

Pesar uma vez significa fazer apenas uma leitura do peso no visor da
balança.. uma vez lido qualquer número no visor da balança, ela quebra.. :P
 

 -Mensagem original-
 De: [EMAIL PROTECTED] 
 [mailto:[EMAIL PROTECTED] Em nome de Chicao Valadares
 Enviada em: terça-feira, 7 de março de 2006 16:38
 Para: obm-l@mat.puc-rio.br
 Assunto: Re: [obm-l] Problema dos Remédios
 
 Pesar uma unica vez???Se vc supor que o ato de pesar é 
 colocar alguma coisa e depois tirar, vc somente deve ir 
 colocando sem tirar cada caixa e se a variaçao do peso nao 
 for conforme esperado tai sua caixa.
 
 
 --- David Cardoso [EMAIL PROTECTED] escreveu:
 
  
  Há 10 caixas de um tipo de remédio, cada caixa com 100 comprimidos, 
  cada comprimido pesando 10g.
  Uma(exatamente uma) destas caixas é oriunda de um lote defeituoso, 
  onde os comprimidos pesam 9 g.
  Você tem acesso a uma balança digital, que só pode ser 
 usada uma vez, 
  e tem precisão suficiente para lhe dar o resultado exato de qqr 
  pesagem com esses remédios.
  
  Qual a sua estratégia de pesagem pra determinar, com 
 certeza, qual a 
  caixa de remédio defeituosa?
  
  
  
  -- 2a. parte, generalização: ---
  
  Qual o número mínimo pesagens necessárias para se descobrir 
 k caixas 
  defeituosas dentro de uma amostragem de n caixas?
  
  
  
 
 ==
 ===
  Instruções para entrar na lista, sair da lista e usar a lista em 
  http://www.mat.puc-rio.br/~nicolau/olimp/obm-l.html
 
 ==
 ===
  
 
 
 O Binômio de Newton é tão belo como a Vênus de Milo.
 O que há é pouca gente para dar por isso... 
 Fernando Pessoa - Poesias de Alvaro Campos
 
 _
 As informações existentes nessa mensagem e no(s) arquivo(s) 
 anexado(s) são para uso restrito, sendo seu sigilo protegido 
 por lei. Caso não seja destinatário, saiba que leitura, 
 divulgação ou cópia são proibidas. 
 Favor
 apagar as informações e notificar o remetente. O uso 
 impróprio será tratado conforme as normas da empresa e a 
 legislação em vigor. Agradecemos sua colaboração.
 
 
 The information mentioned in this message and in the archives 
 attached are of restricted use, and its privacy is protected 
 by law. If you are not the addressee, be aware that reading, 
 disclosure or copy are forbidden. 
 Please
 delete this information and notify the sender. Inappropriate 
 use will be tracted according to company's rules and valid 
 laws. Thank you for your cooperation.
 
 
   
 ___
 Yahoo! Acesso Grátis - Internet rápida e grátis. Instale o 
 discador agora!
 http://br.acesso.yahoo.com
 ==
 ===
 Instruções para entrar na lista, sair da lista e usar a lista 
 em http://www.mat.puc-rio.br/~nicolau/olimp/obm-l.html
 ==
 ===



=
Instruções para entrar na lista, sair da lista e usar a lista em
http://www.mat.puc-rio.br/~nicolau/olimp/obm-l.html
=


Re: [obm-l] RES: [obm-l] Problema dos Remédios

2006-03-07 Por tôpico Felipe Avelino
ah eh facil!!

escolhe uma caixa qualquer..
e numera as restantes...


tira um comprimido da caixa numero 1 e coloca junto 
tira dois comprimidos da caixa numero 2 ..
e assim por diante..
ateh a caixa numero 9

junta todos esses comprimidos e coloca pra pesar junto com a caixa escolhida primeiramente

se pesar1449g a caixa defeituosa eh a numero 1
se pesar 1448g a caixa defeituosa eh a numero 2

se pesar 1450g a caixa defeituosa eh a escolhida
.

Em 07/03/06, David Cardoso [EMAIL PROTECTED] escreveu:
Pesar uma vez significa fazer apenas uma leitura do peso no visor dabalança.. uma vez lido qualquer número no visor da balança, ela quebra.. :P
 -Mensagem original- De: [EMAIL PROTECTED] [mailto:[EMAIL PROTECTED]
] Em nome de Chicao Valadares Enviada em: terça-feira, 7 de março de 2006 16:38 Para: obm-l@mat.puc-rio.br Assunto: Re: [obm-l] Problema dos Remédios
 Pesar uma unica vez???Se vc supor que o ato de pesar é colocar alguma coisa e depois tirar, vc somente deve ir colocando sem tirar cada caixa e se a variaçao do peso nao for conforme esperado tai sua caixa.
 --- David Cardoso [EMAIL PROTECTED] escreveu:   Há 10 caixas de um tipo de remédio, cada caixa com 100 comprimidos,
  cada comprimido pesando 10g.  Uma(exatamente uma) destas caixas é oriunda de um lote defeituoso,  onde os comprimidos pesam 9 g.  Você tem acesso a uma balança digital, que só pode ser
 usada uma vez,  e tem precisão suficiente para lhe dar o resultado exato de qqr  pesagem com esses remédios.   Qual a sua estratégia de pesagem pra determinar, com
 certeza, qual a  caixa de remédio defeituosa? -- 2a. parte, generalização: ---   Qual o número mínimo pesagens necessárias para se descobrir
 k caixas  defeituosas dentro de uma amostragem de n caixas? == ===
  Instruções para entrar na lista, sair da lista e usar a lista em  http://www.mat.puc-rio.br/~nicolau/olimp/obm-l.html 
 == ===  O Binômio de Newton é tão belo como a Vênus de Milo. O que há é pouca gente para dar por isso... 
 Fernando Pessoa - Poesias de Alvaro Campos _ As informações existentes nessa mensagem e no(s) arquivo(s) anexado(s) são para uso restrito, sendo seu sigilo protegido
 por lei. Caso não seja destinatário, saiba que leitura, divulgação ou cópia são proibidas. Favor apagar as informações e notificar o remetente. O uso impróprio será tratado conforme as normas da empresa e a
 legislação em vigor. Agradecemos sua colaboração. The information mentioned in this message and in the archives attached are of restricted use, and its privacy is protected by law. If you are not the addressee, be aware that reading,
 disclosure or copy are forbidden. Please delete this information and notify the sender. Inappropriate use will be tracted according to company's rules and valid laws. Thank you for your cooperation.
 ___ Yahoo! Acesso Grátis - Internet rápida e grátis. Instale o discador agora! 
http://br.acesso.yahoo.com == === Instruções para entrar na lista, sair da lista e usar a lista em 
http://www.mat.puc-rio.br/~nicolau/olimp/obm-l.html == 
Instruções para entrar na lista, sair da lista e usar a lista emhttp://www.mat.puc-rio.br/~nicolau/olimp/obm-l.html=



[obm-l] RES: [obm-l] RES: [obm-l] Problema dos Remédios

2006-03-07 Por tôpico David Cardoso

Isso mesmo! Sendo que se a caixa ecolhida fosse defeituosa tudo pesaria
1350g.

Abraço,

David

 -Mensagem original-
 De: [EMAIL PROTECTED] 
 [mailto:[EMAIL PROTECTED] Em nome de Felipe Avelino
 Enviada em: terça-feira, 7 de março de 2006 19:23
 Para: obm-l@mat.puc-rio.br
 Assunto: Re: [obm-l] RES: [obm-l] Problema dos Remédios
 
 ah eh facil!!
  
 escolhe uma caixa qualquer..
 e numera as restantes...
  
  
 tira um comprimido da caixa numero 1 e coloca junto tira dois 
 comprimidos da caixa numero 2 ..
 e assim por diante..
 ateh a caixa numero 9
  
 junta todos esses comprimidos e coloca pra pesar junto com a 
 caixa escolhida primeiramente
  
 se pesar 1449g  a caixa defeituosa eh a numero 1 se pesar 
 1448g a caixa defeituosa eh a numero 2 
 se pesar 1450g a caixa defeituosa eh a escolhida .
  
 
  
 Em 07/03/06, David Cardoso [EMAIL PROTECTED] escreveu: 
 
 
   Pesar uma vez significa fazer apenas uma leitura do 
 peso no visor da
   balança.. uma vez lido qualquer número no visor da 
 balança, ela quebra.. :P 
   
   
-Mensagem original-
De: [EMAIL PROTECTED]
[mailto:[EMAIL PROTECTED] ] Em nome de 
 Chicao Valadares
Enviada em: terça-feira, 7 de março de 2006 16:38
Para: obm-l@mat.puc-rio.br
Assunto: Re: [obm-l] Problema dos Remédios

Pesar uma unica vez???Se vc supor que o ato de pesar é
colocar alguma coisa e depois tirar, vc somente deve ir
colocando sem tirar cada caixa e se a variaçao do peso nao
for conforme esperado tai sua caixa. 
   
   
--- David Cardoso [EMAIL PROTECTED] escreveu:
   

 Há 10 caixas de um tipo de remédio, cada caixa com 
 100 comprimidos, 
 cada comprimido pesando 10g.
 Uma(exatamente uma) destas caixas é oriunda de um 
 lote defeituoso,
 onde os comprimidos pesam 9 g.
 Você tem acesso a uma balança digital, que só pode ser 
usada uma vez,
 e tem precisão suficiente para lhe dar o resultado 
 exato de qqr
 pesagem com esses remédios.

 Qual a sua estratégia de pesagem pra determinar, com
certeza, qual a
 caixa de remédio defeituosa?



 -- 2a. parte, generalização: ---

 Qual o número mínimo pesagens necessárias para se descobrir 
k caixas
 defeituosas dentro de uma amostragem de n caixas?




==
=== 
 Instruções para entrar na lista, sair da lista e 
 usar a lista em
 http://www.mat.puc-rio.br/~nicolau/olimp/obm-l.html
 
==
===

   
   
O Binômio de Newton é tão belo como a Vênus de Milo.
O que há é pouca gente para dar por isso...  
Fernando Pessoa - Poesias de Alvaro Campos
   

 _
As informações existentes nessa mensagem e no(s) arquivo(s)
anexado(s) são para uso restrito, sendo seu sigilo protegido 
por lei. Caso não seja destinatário, saiba que leitura,
divulgação ou cópia são proibidas.
Favor
apagar as informações e notificar o remetente. O uso
impróprio será tratado conforme as normas da empresa e a 
legislação em vigor. Agradecemos sua colaboração.
   
   
The information mentioned in this message and in the archives
attached are of restricted use, and its privacy is protected
by law. If you are not the addressee, be aware that reading, 
disclosure or copy are forbidden.
Please
delete this information and notify the sender. Inappropriate
use will be tracted according to company's rules and valid
laws. Thank you for your cooperation. 
   
   
   
___
Yahoo! Acesso Grátis - Internet rápida e grátis. Instale o
discador agora!
http://br.acesso.yahoo.com http://br.acesso.yahoo.com 
==
===
Instruções para entrar na lista, sair da lista e usar a lista
em 
 http://www.mat.puc-rio.br/~nicolau/olimp/obm-l.html 
 http://www.mat.puc-rio.br/~nicolau/olimp/obm-l.html 
==
===
   
   
   
   
 ==
 === 
   Instruções para entrar na lista, sair da lista e usar a lista em
   http://www.mat.puc-rio.br/~nicolau/olimp/obm-l.html

RES: [obm-l] Problema

2006-01-02 Por tôpico Artur Costa Steiner
O primeiro matematico recebeu o produto P e concluiu que nao lhe era
possivel precisar quais eram os dois numeros dados. Disto podemos concluir
que (1) - P nao eh o quadrado de um numero primo e (2) - P nao eh o produto
de exatamente dois primos distintos. Se (1) ou (2) ocorressem, seria entao
possivel precisar quais eram os numeros.  

Se P = p^3 para algum primo p, entao os numeros estao perfeitamente
determinados, um eh p e o outro eh p^2. Mas observamos tambem que, se P
puder ser decomposto no produto de pelo menos 3 primos distintos, entao hah
sempre mais de um par de inteiros cujo produto eh P. Ocorrendo esta ultima
situacao, nao hah como identificar os numeros. E se p for uma potencia
inteira n=4 de um primo p, entao hah tambem mais de uma opcao para os
numeros.

Assim, como o 1o matematico nao identificou os numeros, podemos afirmar que
P nao eh o produto de 2 primos distintos, nem o quadrado de um primo e nem o
cubo de um primo. Qualquer outra situacao leva a mais de uma possibilidade
para o par de numeros.


Pela soma S do 2 numeros, o segundo matematico concluiu que o produto P
satisfazia aas condicoes citadas. 

Eh o que pude concluir por ora.

Artur



   



-Mensagem original-
De: [EMAIL PROTECTED] [mailto:[EMAIL PROTECTED]
nome de [EMAIL PROTECTED]
Enviada em: quinta-feira, 29 de dezembro de 2005 00:38
Para: obm-l@mat.puc-rio.br
Assunto: [obm-l] Problema



Me lembrei de outro velho problema que me passaram com dados novos:

Um gênio matemático recebe, num papel, a soma de dois números inteiros entre
2 e
100. Um outro gênio recebe o produto dos mesmos dois números. Os dois
iniciam o
diálogo:
   
- Este produto não é o suficiente para achar os dois números.
- Eu sabia.
- Então, eu conheço estes números.
- Nesse caso, eu também.
- Quais são os dois números?






=
Instruções para entrar na lista, sair da lista e usar a lista em
http://www.mat.puc-rio.br/~nicolau/olimp/obm-l.html
=


[obm-l] RES: [obm-l] Problema das Alianças...

2005-08-30 Por tôpico David Cardoso

Divida as 12 em 3 grupos de 4.
Compara dois grupos na balança.
Com isso, vc determina em qual dos 4 grupos a aliança está.
Pegue esse grupo que vc acabou de terminar, com 4 alianças,  compare duas
elas.
Caso tenha empatado, faça a 3a. pesagem com as 2 alianças restantes e
descubra qual é.

[]'s
David

 -Mensagem original-
 De: [EMAIL PROTECTED] [mailto:[EMAIL PROTECTED] Em nome
 de filipe junqueira
 Enviada em: domingo, 28 de agosto de 2005 23:34
 Para: obm-l@mat.puc-rio.br
 Assunto: [obm-l] Problema das Alianças...
 
 Ola caros amigos da lista...
 Um amigo de meu pai me desafio com o seguinte problema e não consigo obter
 resposta la vai:
 Um homem muito rico tinha 12 alianças de ouro uma delas entretanto era
 mais leve ou mais pesada que as demais.Como descobrir qual aliança é a
 mais
 leve ou pesada com apenas tres pesagens em uma balança tradicional...(
 Aquelas que simbolizam a justiça onde se compara apenas dois pesos).
 (Creio eu que 3 pesagens não são sufucientesmas espero que esteja
 enganado)
 Muito obrigado e boa sorte com o problema
 Filipe Junqueira
 
 
 =
 Instruções para entrar na lista, sair da lista e usar a lista em
 http://www.mat.puc-rio.br/~nicolau/olimp/obm-l.html
 =



=
Instruções para entrar na lista, sair da lista e usar a lista em
http://www.mat.puc-rio.br/~nicolau/olimp/obm-l.html
=


RE: [obm-l] RES: [obm-l] Problema das Alian�as...

2005-08-30 Por tôpico Qwert Smith
Como o prof. Nicolau ja falou esse assunto ja foi mais que discutido.  O 
problema e interessante...atente pro fato que nao se sabe se a alianca e 
mais pesada ou leve.  A primeira pesagem no caso apenas eliminaria 4.   
Tente um pouco mais e sigua os links que o prof. indicou.



From: David Cardoso [EMAIL PROTECTED]

Divida as 12 em 3 grupos de 4.
Compara dois grupos na balança.
Com isso, vc determina em qual dos 4 grupos a aliança está.
Pegue esse grupo que vc acabou de terminar, com 4 alianças,  compare duas
elas.
Caso tenha empatado, faça a 3a. pesagem com as 2 alianças restantes e
descubra qual é.

[]'s
David



=
Instruções para entrar na lista, sair da lista e usar a lista em
http://www.mat.puc-rio.br/~nicolau/olimp/obm-l.html
=


Re: RES: RES: [obm-l] Problema do casal de filhos

2005-08-23 Por tôpico Nicolau C. Saldanha
On Mon, Aug 22, 2005 at 09:31:55PM -0300, Luiz Viola wrote:
 Intuitivamente pra mim é 1/2. Acho que para a maioria das pessoas a quem
 eu propus o problema também responderam 1/2. O livro porém apresenta a
 resposta 1/3, tal como propuseram a solução aqui na lista... eu não
 consigo aceitar 1/3...nem fazendo força...

O livro dá alguma explicação? Seria interessante se você pudesse transcrever
enunciado e resolução para que pudéssemos saber exatamente de que estamos
falando. Pelo que você escreveu até agora, não acho implausível que o livro
esteja simplesmente errado.

[]s, N.
=
Instruções para entrar na lista, sair da lista e usar a lista em
http://www.mat.puc-rio.br/~nicolau/olimp/obm-l.html
=


Re: RES: RES: [obm-l] Problema do casal de filhos

2005-08-23 Por tôpico claudio\.buffara





De:
[EMAIL PROTECTED]




Para:
obm-l@mat.puc-rio.br




Cópia:





Data:
Tue, 23 Aug 2005 10:05:18 -0300




Assunto:
Re: RES: RES: [obm-l] Problema do casal de filhos
 On Mon, Aug 22, 2005 at 09:31:55PM -0300, Luiz Viola wrote:
  Intuitivamente pra mim é 1/2. Acho que para a maioria das pessoas a quem
  eu propus o problema também responderam 1/2. O livro porém apresenta a
  resposta 1/3, tal como propuseram a solução aqui na lista... eu não
  consigo aceitar 1/3...nem fazendo força...
 
 O livro dá alguma explicação? Seria interessante se você pudesse transcrever
 enunciado e resolução para que pudéssemos saber exatamente de que estamos
 falando. Pelo que você escreveu até agora, não acho implausível que o livro
 esteja simplesmente errado.
 
 []s, N.
 

Isso me faz lembrar o problema das três caixas fechadas.
A primeiradelas contém duas moedas de ouro, a segunda, uma de ouro e uma de prata, e a terceira, duas de prata.
Tomamos uma caixa ao acaso e dela retiramos uma moeda.
Dado que a moeda retirada é de ouro, qual a probabilidade de que a outra moeda desta caixa também seja de ouro?

A resposta é 2/3, apesar de muita gente achar que é 1/2.

[]s,
Claudio.




Re: RES: [obm-l] Problema do casal de filhos

2005-08-22 Por tôpico Nicolau C. Saldanha
On Sun, Aug 21, 2005 at 10:37:09PM -0300, Luiz Viola wrote:
  Um homem visita um casal que tem dois filhos. Uma das crianças, um
 menino, vem à sala. Encontre a probabilidade (p) de o outro ser
 também um
 menino, se
 
  (i) sabe-se que a outra criança é mais nova
 
 (ii) nada se sabe sobre a outra criança
 
  A resposta do item (ii) não é 1/2 Alguém consegue enxergar por
 que

Para mim a resposta correta é 1/2 sim (para ambos os itens) e
o raciocínio que foi apresentado para chegar a outro valor está equivocado.
Tudo isto com suposições que me parecem naturais e que não vou explicitar.
Pq exatamente você acha que a resposta deveria ser diferente de 1/2?

[]s, N. 

=
Instruções para entrar na lista, sair da lista e usar a lista em
http://www.mat.puc-rio.br/~nicolau/olimp/obm-l.html
=


RES: RES: [obm-l] Problema do casal de filhos

2005-08-22 Por tôpico Luiz Viola
Intuitivamente pra mim é 1/2. Acho que para a maioria das pessoas a quem
eu propus o problema também responderam 1/2. O livro porém apresenta a
resposta 1/3, tal como propuseram a solução aqui na lista... eu não
consigo aceitar 1/3...nem fazendo força...

Abraço!
Viola

-Mensagem original-
De: [EMAIL PROTECTED] [mailto:[EMAIL PROTECTED] Em
nome de Nicolau C. Saldanha
Enviada em: segunda-feira, 22 de agosto de 2005 13:29
Para: obm-l@mat.puc-rio.br
Assunto: Re: RES: [obm-l] Problema do casal de filhos

On Sun, Aug 21, 2005 at 10:37:09PM -0300, Luiz Viola wrote:
  Um homem visita um casal que tem dois filhos. Uma das crianças, um
 menino, vem à sala. Encontre a probabilidade (p) de o outro ser
 também um
 menino, se
 
  (i) sabe-se que a outra criança é mais nova
 
 (ii) nada se sabe sobre a outra criança
 
  A resposta do item (ii) não é 1/2 Alguém consegue enxergar por
 que

Para mim a resposta correta é 1/2 sim (para ambos os itens) e
o raciocínio que foi apresentado para chegar a outro valor está
equivocado.
Tudo isto com suposições que me parecem naturais e que não vou
explicitar.
Pq exatamente você acha que a resposta deveria ser diferente de 1/2?

[]s, N. 


=
Instruções para entrar na lista, sair da lista e usar a lista em
http://www.mat.puc-rio.br/~nicolau/olimp/obm-l.html

=

E-mail classificado pelo Identificador de Spam Inteligente Terra.
Para alterar a categoria classificada, visite
http://mail.terra.com.br/protected_email/imail/imail.cgi?+_u=lfviola_l=
1,1124729600.942542.12590.niassa.terra.com.br,3404,Des15,Des15

Esta mensagem foi verificada pelo E-mail Protegido Terra.
Scan engine: McAfee VirusScan / Atualizado em 22/08/2005 / Versão:
4.4.00/4564
Proteja o seu e-mail Terra: http://mail.terra.com.br/



=
Instruções para entrar na lista, sair da lista e usar a lista em
http://www.mat.puc-rio.br/~nicolau/olimp/obm-l.html
=


RES: [obm-l] Problema do casal de filhos

2005-08-21 Por tôpico Luiz Viola
Caramba...chegamos a um consenso?

-Mensagem original-
De: [EMAIL PROTECTED] [mailto:[EMAIL PROTECTED] Em
nome de Fábio Dias Moreira
Enviada em: domingo, 21 de agosto de 2005 21:54
Para: Thyago A. Kufner
Assunto: Re: [obm-l] Problema do casal de filhos

[21/08/2005, [EMAIL PROTECTED]:
 [21/08/2005, [EMAIL PROTECTED]:
 
  Um homem visita um casal que tem dois filhos. Uma das crianças, um
 menino, vem à sala. Encontre a probabilidade (p) de o outro ser
também um
 menino, se
 
  (i) sabe-se que a outra criança é mais nova
 
 (ii) nada se sabe sobre a outra criança
 
  A resposta do item (ii) não é 1/2 Alguém consegue enxergar por
 que
 
 Digamos que H representa um filho homem e M uma filha mulher.
 Como o casal teve dois filhos, as possibilidades são (na ordem mais
velha,
 mais nova):

 H, H
 H, M
 M, H
 M, M

 Na primeira situação descrita no problema, sabemos que a criança mais
velha
 é um menino. Só podemos ter duas das quatro situações acima:

 H, M
 H, H

 Ou seja, para a outra criança (a mais nova) ser um menino, só há uma
 situação entre duas possíveis. Por isso que a probabilidade é 1/2.

 Na segunda situação, só sabemos que uma das duas crianças é menino. Ou
seja,
 das quatro situações possíveis, estamos lidando com apenas três (as
que
 possuem no mínimo um H):

 H, H
 H, M
 M, H

 Assim, temos apenas 1 entre 3 possibilidades que satisfazem o
enunciado.
 Portanto, para a situação 2, a probabilidade é 1/3.

 []'s
 Kufner
 www.cursinho.hpg.com.br http://www.cursinho.hpg.com.br

Sim, mas nos casos (H, M) e (M, H) a probabilidade do menino, e não a
menina, entrar na sala, é 1/2 (afinal de contas, o enunciado não diz
nada que poderia sugerir uma assimetria entre um eventual menino e uma
eventual menina). Logo os três casos que você mostrou *não têm* a
mesma probabilidade -- a probabilidade desses dois casos é, digamos,
x, e, como a probabilidade de um menino entrar no caso (H, H) é o
dobro da dos outros casos, a probabilidade de (H, H) é 2x. Como a soma
das probabilidades é 1,

x + x + 2*x = 1 =
x = 1/4 =
2*x = 1/2.

Essa é, na realidade, uma aplicação do Teorema de Bayes -- o argumento
que eu fiz acima foi uma versão intuitiva da demonstração formal:

http://mathworld.wolfram.com/BayesTheorem.html

(E, de fato,

(1/4*1)/(1/4*1/2+1/4*1/2+1/4*1) = 1/2

como se poderia esperar.)

[]s,

-- 
Fábio Dias Moreira


=
Instruções para entrar na lista, sair da lista e usar a lista em
http://www.mat.puc-rio.br/~nicolau/olimp/obm-l.html
=


Re: RES: [obm-l] Problema Subconjuntos

2004-07-22 Por tôpico Poncio
Olá Artur!!!
O 1° lema de Kaplansky diz que o número de p-subconjuntos (isto é, um
subconjunto com p elementos) de {1,2,...,n} nos quais não há números
consecutivos é:
f (n,p) = Combinação(n-p+1,p).
Para maiores detalhes consulte Análise Combinatória e Probabilidade de
Morgado, Pitombeira, P.C.Pinto Carvalho e Pedro Fernandez, da coleção do
Professor de Matemática.
Espero ter ajudado,um grande abraço,
Poncio
- Original Message -
From: Artur Costa Steiner [EMAIL PROTECTED]
To: [EMAIL PROTECTED]
Sent: Wednesday, July 21, 2004 8:14 PM
Subject: Re: RES: [obm-l] Problema Subconjuntos


 C(n-2;3). Basta usar o primeiro lema de Kaplansky.

 Eu nunca ouvi falar deste lema (ignorancia minha). Alguem poderia
 enuncia-lo?
 Obrigado.
 Artur

 
 OPEN Internet
 @ Primeiro provedor do DF com anti-vírus no servidor de e-mails @


 =
 Instruções para entrar na lista, sair da lista e usar a lista em
 http://www.mat.puc-rio.br/~nicolau/olimp/obm-l.html
 =


=
Instruções para entrar na lista, sair da lista e usar a lista em
http://www.mat.puc-rio.br/~nicolau/olimp/obm-l.html
=


Re: RES: [obm-l] Problema Subconjuntos

2004-07-21 Por tôpico Helder Suzuki
achei isso no arquivo da lista:
quote
Kaplansky.
Primeiro lema:
O número de subconjuntos de tamanho p do conjunto {1,
2,..., n} no qual nao figuram numeros consecutivos eh
C(n-p+1, p)
Segundo lema:
Igual ao anterior, mas considerando 1 e n como
consecutivos. O numero de subconjuntos eh 
[n/(n-p)]*C(n-p, p).
/quote

--- Artur Costa Steiner [EMAIL PROTECTED]
escreveu:  C(n-2;3). Basta usar o primeiro lema de
Kaplansky.
 
 Eu nunca ouvi falar deste lema (ignorancia minha).
 Alguem poderia
 enuncia-lo?
 Obrigado.
 Artur





___
Yahoo! Mail agora com 100MB, anti-spam e antivírus grátis!
http://br.info.mail.yahoo.com/
=
Instruções para entrar na lista, sair da lista e usar a lista em
http://www.mat.puc-rio.br/~nicolau/olimp/obm-l.html
=


RES: [obm-l] Problema de Divisibilidade / Primos

2004-07-20 Por tôpico David M. Cardoso

Droga droga droga !!!
Na pressa, errei o enunciado da questão!
Mil desculpas!

Segue o enunciado correto:

Quantos inteiros existem que não são divisíveis por qualquer que seja o
primo maior que 20 e não são divisíveis pelo quadrado de qualquer que seja o
primo?

Puxa vida... tenho prova amanha cedo, vou tentar tirar minhas duvidas de
ultima hora, tenho a sorte de voces existirem e ainda erro o enunciado da
questao... :~(

[]'s
David

 -Mensagem original-
 De: [EMAIL PROTECTED] 
 [mailto:[EMAIL PROTECTED] Em nome de Bruno França dos Reis
 Enviada em: terça-feira, 20 de julho de 2004 18:53
 Para: [EMAIL PROTECTED]
 Assunto: Re: [obm-l] Problema de Divisibilidade / Primos
 
 -BEGIN PGP SIGNED MESSAGE-
 Hash: SHA1
 
 On Tuesday 20 July 2004 18:26, David M. Cardoso wrote:
  Mais duas questoes que não consigo me mecher:
 
  Quantos inteiros existem que não são divisíveis por 
 qualquer que seja 
  o primo maior que 20 e não são divisiveis por qualquer que 
 seja o primo?
 
 a) infinitos: 2^n não é divisível por qualquer que seja o 
 primo maior que 20, pois é divisível apenas pelo primo 2, 
 qualquer que seja n natural.
 
 b) apenas o 1, pois qualquer outro número é divisível por ao 
 menos um primo: 
 se ele for composto, sabemos que ele é múltiplo de primos, e 
 se ele é primo, ele é divisível por si próprio, um número 
 primo. Já o 1 é divisível apenas por 1, que não é primo (e 
 não me venham com essa de que 1 é primo também!)
 
 acho que é isso!
 
 abraço
 
 - --
 Bruno França dos Reis
 brunoreis at terra com br
 icq: 12626000
 gpg-key: 
 http://planeta.terra.com.br/informatica/brunoreis/brunoreis.key
 
 -BEGIN PGP SIGNATURE-
 Version: GnuPG v1.2.4 (GNU/Linux)
 
 iD8DBQFA/ZREsHdDIT+qyroRAhQFAKDOZm/uCMp38TYe+uXT2rL+lkNPWQCfWTdb
 iMrCfq37UfF/7EZvrP6Qm3g=
 =qpSy
 -END PGP SIGNATURE-
 
 ==
 ===
 Instruções para entrar na lista, sair da lista e usar a lista 
 em http://www.mat.puc-rio.br/~nicolau/olimp/obm-l.html
 ==
 ===
 


=
Instruções para entrar na lista, sair da lista e usar a lista em
http://www.mat.puc-rio.br/~nicolau/olimp/obm-l.html
=


Re: RES: [obm-l] Problema de Divisibilidade / Primos

2004-07-20 Por tôpico Bernardo Freitas Paulo da Costa
Oi, David,

Enumere os primos menores do que 20:
2, 3, 5, 7, 11, 13, 17, 19: são 8.

Um número que satisfaça as condições do enunciado pode ter,
no máximo, um de cada um destes fatores, pela segunda parte, e nenhum 
outro fator, pela primeira parte.
Assim, temos um problema de combinatória, agora:
quantos números podemos formar utilizando apenas o produto de 8 primos, 
onde não podemos incluir um primo duas vezes. Ou, mais combinatória ainda,
quantos subconjuntos de um conjunto de 8 elementos existem?
Para ver que as soluções são iguais, associe a cada subconjunto
o número correspondente ao produto de seus elementos, e ao subconjunto 
vazio o número 1 (eis aqui mais uma boa justificativa para termos um 
produtório vazio valendo 1!!)

Bom, para este problema a resposta é conhecida: vale 2^8 = 256.
Pronto, são 256 números.

Abraços,
Bernardo Costa


On Tue, 20 Jul 2004, David M. Cardoso wrote:

 
 Droga droga droga !!!
 Na pressa, errei o enunciado da questão!
 Mil desculpas!
 
 Segue o enunciado correto:
 
 Quantos inteiros existem que não são divisíveis por qualquer que seja o
 primo maior que 20 e não são divisíveis pelo quadrado de qualquer que seja o
 primo?
 
 Puxa vida... tenho prova amanha cedo, vou tentar tirar minhas duvidas de
 ultima hora, tenho a sorte de voces existirem e ainda erro o enunciado da
 questao... :~(
 
 []'s
 David
 
  -Mensagem original-
  De: [EMAIL PROTECTED] 
  [mailto:[EMAIL PROTECTED] Em nome de Bruno França dos Reis
  Enviada em: terça-feira, 20 de julho de 2004 18:53
  Para: [EMAIL PROTECTED]
  Assunto: Re: [obm-l] Problema de Divisibilidade / Primos
  
  -BEGIN PGP SIGNED MESSAGE-
  Hash: SHA1
  
  On Tuesday 20 July 2004 18:26, David M. Cardoso wrote:
   Mais duas questoes que não consigo me mecher:
  
   Quantos inteiros existem que não são divisíveis por 
  qualquer que seja 
   o primo maior que 20 e não são divisiveis por qualquer que 
  seja o primo?
  
  a) infinitos: 2^n não é divisível por qualquer que seja o 
  primo maior que 20, pois é divisível apenas pelo primo 2, 
  qualquer que seja n natural.
  
  b) apenas o 1, pois qualquer outro número é divisível por ao 
  menos um primo: 
  se ele for composto, sabemos que ele é múltiplo de primos, e 
  se ele é primo, ele é divisível por si próprio, um número 
  primo. Já o 1 é divisível apenas por 1, que não é primo (e 
  não me venham com essa de que 1 é primo também!)
  
  acho que é isso!
  
  abraço
  
  - --
  Bruno França dos Reis
  brunoreis at terra com br
  icq: 12626000
  gpg-key: 
  http://planeta.terra.com.br/informatica/brunoreis/brunoreis.key
  
  -BEGIN PGP SIGNATURE-
  Version: GnuPG v1.2.4 (GNU/Linux)
  
  iD8DBQFA/ZREsHdDIT+qyroRAhQFAKDOZm/uCMp38TYe+uXT2rL+lkNPWQCfWTdb
  iMrCfq37UfF/7EZvrP6Qm3g=
  =qpSy
  -END PGP SIGNATURE-
  
  ==
  ===
  Instruções para entrar na lista, sair da lista e usar a lista 
  em http://www.mat.puc-rio.br/~nicolau/olimp/obm-l.html
  ==
  ===
  
 
 
 =
 Instruções para entrar na lista, sair da lista e usar a lista em
 http://www.mat.puc-rio.br/~nicolau/olimp/obm-l.html
 =
 

=
Instruções para entrar na lista, sair da lista e usar a lista em
http://www.mat.puc-rio.br/~nicolau/olimp/obm-l.html
=


RES: [obm-l] Problema de Divisibilidade / Primos

2004-07-20 Por tôpico David M. Cardoso
Aeee ... acabei de pensar na solucao, não sei se ta certo:

se n é o produto de k primos (i=k=8), entao
n = p_1 * p_2 * p_3 * ... * p_k

tal que p_i  20 (1 = i = k)
entao p_i pertence ao conjunto dos primos menores que 20 {
2,3,5,7,11,13,17,19 }
queremos contar os subconjuntos desse conjunto... menos o vazio..

temos entao 2^8 - 1 numeros deste tipo.

Ta certo?

[]'s
David

 -Mensagem original-
 De: [EMAIL PROTECTED] 
 [mailto:[EMAIL PROTECTED] Em nome de David M. Cardoso
 Enviada em: terça-feira, 20 de julho de 2004 20:11
 Para: [EMAIL PROTECTED]
 Assunto: RES: [obm-l] Problema de Divisibilidade / Primos
 
 
 Droga droga droga !!!
 Na pressa, errei o enunciado da questão!
 Mil desculpas!
 
 Segue o enunciado correto:
 
 Quantos inteiros existem que não são divisíveis por qualquer 
 que seja o primo maior que 20 e não são divisíveis pelo 
 quadrado de qualquer que seja o primo?
 
 Puxa vida... tenho prova amanha cedo, vou tentar tirar minhas 
 duvidas de ultima hora, tenho a sorte de voces existirem e 
 ainda erro o enunciado da questao... :~(
 
 []'s
 David
 
  -Mensagem original-
  De: [EMAIL PROTECTED]
  [mailto:[EMAIL PROTECTED] Em nome de Bruno França 
 dos Reis 
  Enviada em: terça-feira, 20 de julho de 2004 18:53
  Para: [EMAIL PROTECTED]
  Assunto: Re: [obm-l] Problema de Divisibilidade / Primos
  
  -BEGIN PGP SIGNED MESSAGE-
  Hash: SHA1
  
  On Tuesday 20 July 2004 18:26, David M. Cardoso wrote:
   Mais duas questoes que não consigo me mecher:
  
   Quantos inteiros existem que não são divisíveis por
  qualquer que seja
   o primo maior que 20 e não são divisiveis por qualquer que
  seja o primo?
  
  a) infinitos: 2^n não é divisível por qualquer que seja o 
 primo maior 
  que 20, pois é divisível apenas pelo primo 2, qualquer que seja n 
  natural.
  
  b) apenas o 1, pois qualquer outro número é divisível por 
 ao menos um 
  primo:
  se ele for composto, sabemos que ele é múltiplo de primos, 
 e se ele é 
  primo, ele é divisível por si próprio, um número primo. Já o 1 é 
  divisível apenas por 1, que não é primo (e não me venham 
 com essa de 
  que 1 é primo também!)
  
  acho que é isso!
  
  abraço
  
  - --
  Bruno França dos Reis
  brunoreis at terra com br
  icq: 12626000
  gpg-key: 
  http://planeta.terra.com.br/informatica/brunoreis/brunoreis.key
  
  -BEGIN PGP SIGNATURE-
  Version: GnuPG v1.2.4 (GNU/Linux)
  
  iD8DBQFA/ZREsHdDIT+qyroRAhQFAKDOZm/uCMp38TYe+uXT2rL+lkNPWQCfWTdb
  iMrCfq37UfF/7EZvrP6Qm3g=
  =qpSy
  -END PGP SIGNATURE-
  
  ==
  ===
  Instruções para entrar na lista, sair da lista e usar a lista em 
  http://www.mat.puc-rio.br/~nicolau/olimp/obm-l.html
  ==
  ===
  
 
 
 ==
 ===
 Instruções para entrar na lista, sair da lista e usar a lista 
 em http://www.mat.puc-rio.br/~nicolau/olimp/obm-l.html
 ==
 ===
 


=
Instruções para entrar na lista, sair da lista e usar a lista em
http://www.mat.puc-rio.br/~nicolau/olimp/obm-l.html
=


RES: [obm-l] Problema Subconjuntos

2004-07-20 Por tôpico David M. Cardoso

Cara, muito obrigado..
Sendo que ta dando trabalho pra eu entender algumas coisas,
como teremos T[n-3] - T[n-4] subconjuntos com os elementos n-1 e n-2..
hora eu penso que entendi, hora eu não entendo mais e fico tentando lembrar
pq eu fico entendido antes, talvez seja o nervosismo, talvez seja apenas
porque o raciocinio eh complicado demais pra mim..

Outra duvida que tenho é se é possível transformar a recorrência num
polinomiozinho em função de n ou se uma resposta desse tipo já esta
completa o suficiente..

[]'s
David

 -Mensagem original-
 De: [EMAIL PROTECTED] 
 [mailto:[EMAIL PROTECTED] Em nome de Helder Suzuki
 Enviada em: terça-feira, 20 de julho de 2004 19:30
 Para: [EMAIL PROTECTED]
 Assunto: Re: [obm-l] Problema Subconjuntos
 
 vamos ver, seguindo a dica de usar recorrencia
 
 se T[n] for igual ao numero de subconjuntos do conjunto {1, 
 2, ..., n} que nao contem 3 inteiros consecutivos.
 temos que:
 T[0] = 1
 {}
 
 T[1] = 2
 {} e {1}
 
 T[2] = 4
 {}, {1},
 {2} e {1, 2}
 
 T[3] = 7
 {}, {1}, {2}, {1, 2},
 {3}, {1, 3}, {2, 3}
 
 T[4] = 13
 {}, {1}, {2}, {3}, {1, 2}, {1, 3}, {2, 3}, {4}, {1, 4}, {2, 
 4}, {3, 4}, {1, 2, 4}, {1, 3, 4}
 
 bom, suponha que sabemos o valor de T[n-1], T[n-2], ..., 
 T[1]; como podemos achar T[n] em funcao de T[n-1]? humm...
 considere todos subconjuntos de {1, 2, 3, 4, ..., n-1} que 
 satisfazem a condicao do enunciado.
 se adicionarmos um elemento n, em quais desses subconjuntos o 
 n pode entrar e quais ele nao pode(para manter a condicao do 
 enunciado)?
 se n nao pode entrar em X subconjuntos, temos que T[n] = 
 T[n-1] + T[n-1] - X T[n] = 2*T[n-1] - X mas X eh o numero de 
 subconjuntos que tem os elementos
 n-1 e n-2.
 
 imagine que temos os subconjnutos de {1, 2, ..., n-3} e 
 queremos adicionar os elementos n-1 e n-2 a esses 
 subconjuntos ao mesmo tempo, nesse caso só nao poderemos 
 adicionar n-1 e n-2 aos subconjuntos que tem o elemento n-3, 
 entao teremos T[n-3] - T[n-4] subconjuntos com os elementos n-1 e n-2:
 X = T[n-3] - T[n-4]
 
 entao nossa recorrencia fica:
 T[n] = 2*T[n-1] - T[n-3] + T[n-4]
 
 []'s,
 Helder
 
 --- David M. Cardoso [EMAIL PROTECTED]
 escreveu:  
  
  Olá,
  
  Alguem pode me ajudar? Não consegui resolver o seguinte problema:
  
  Quantos subconjuntos o conjunto {1,2,3,...,n} tais que não contêm 
  três inteiros consecutivos?
  
  A dica dada na questão é: Encontre uma recorrência. 
 Porém, qualquer 
  solução (sem/com recorrência) vai ajudar.
  
  []'s
  David
 
 
   
   
   
 ___
 Yahoo! Mail agora com 100MB, anti-spam e antivírus grátis!
 http://br.info.mail.yahoo.com/
 ==
 ===
 Instruções para entrar na lista, sair da lista e usar a lista 
 em http://www.mat.puc-rio.br/~nicolau/olimp/obm-l.html
 ==
 ===
 


=
Instruções para entrar na lista, sair da lista e usar a lista em
http://www.mat.puc-rio.br/~nicolau/olimp/obm-l.html
=


RES: RES: [obm-l] Problema de Divisibilidade / Primos

2004-07-20 Por tôpico David M. Cardoso

Realmente.. realmente.. o vazio conta como o numero 1..
ok .. obrigado!

[]'s
David

 -Mensagem original-
 De: [EMAIL PROTECTED] 
 [mailto:[EMAIL PROTECTED] Em nome de Bernardo 
 Freitas Paulo da Costa
 Enviada em: terça-feira, 20 de julho de 2004 21:29
 Para: [EMAIL PROTECTED]
 Assunto: Re: RES: [obm-l] Problema de Divisibilidade / Primos
 
 Oi, David,
 
 Enumere os primos menores do que 20:
 2, 3, 5, 7, 11, 13, 17, 19: são 8.
 
 Um número que satisfaça as condições do enunciado pode ter, 
 no máximo, um de cada um destes fatores, pela segunda parte, 
 e nenhum outro fator, pela primeira parte.
 Assim, temos um problema de combinatória, agora:
 quantos números podemos formar utilizando apenas o produto de 
 8 primos, onde não podemos incluir um primo duas vezes. Ou, 
 mais combinatória ainda, quantos subconjuntos de um conjunto 
 de 8 elementos existem?
 Para ver que as soluções são iguais, associe a cada 
 subconjunto o número correspondente ao produto de seus 
 elementos, e ao subconjunto vazio o número 1 (eis aqui mais 
 uma boa justificativa para termos um produtório vazio valendo 1!!)
 
 Bom, para este problema a resposta é conhecida: vale 2^8 = 256.
 Pronto, são 256 números.
 
 Abraços,
 Bernardo Costa
 
 
 On Tue, 20 Jul 2004, David M. Cardoso wrote:
 
  
  Droga droga droga !!!
  Na pressa, errei o enunciado da questão!
  Mil desculpas!
  
  Segue o enunciado correto:
  
  Quantos inteiros existem que não são divisíveis por 
 qualquer que seja 
  o primo maior que 20 e não são divisíveis pelo quadrado de qualquer 
  que seja o primo?
  
  Puxa vida... tenho prova amanha cedo, vou tentar tirar 
 minhas duvidas 
  de ultima hora, tenho a sorte de voces existirem e ainda erro o 
  enunciado da questao... :~(
  
  []'s
  David
  
   -Mensagem original-
   De: [EMAIL PROTECTED]
   [mailto:[EMAIL PROTECTED] Em nome de Bruno 
 França dos Reis 
   Enviada em: terça-feira, 20 de julho de 2004 18:53
   Para: [EMAIL PROTECTED]
   Assunto: Re: [obm-l] Problema de Divisibilidade / Primos
   
   -BEGIN PGP SIGNED MESSAGE-
   Hash: SHA1
   
   On Tuesday 20 July 2004 18:26, David M. Cardoso wrote:
Mais duas questoes que não consigo me mecher:
   
Quantos inteiros existem que não são divisíveis por
   qualquer que seja
o primo maior que 20 e não são divisiveis por qualquer que
   seja o primo?
   
   a) infinitos: 2^n não é divisível por qualquer que seja o primo 
   maior que 20, pois é divisível apenas pelo primo 2, qualquer que 
   seja n natural.
   
   b) apenas o 1, pois qualquer outro número é divisível por 
 ao menos 
   um primo:
   se ele for composto, sabemos que ele é múltiplo de 
 primos, e se ele 
   é primo, ele é divisível por si próprio, um número primo. 
 Já o 1 é 
   divisível apenas por 1, que não é primo (e não me venham 
 com essa de 
   que 1 é primo também!)
   
   acho que é isso!
   
   abraço
   
   - --
   Bruno França dos Reis
   brunoreis at terra com br
   icq: 12626000
   gpg-key: 
   http://planeta.terra.com.br/informatica/brunoreis/brunoreis.key
   
   -BEGIN PGP SIGNATURE-
   Version: GnuPG v1.2.4 (GNU/Linux)
   
   iD8DBQFA/ZREsHdDIT+qyroRAhQFAKDOZm/uCMp38TYe+uXT2rL+lkNPWQCfWTdb
   iMrCfq37UfF/7EZvrP6Qm3g=
   =qpSy
   -END PGP SIGNATURE-
   
   ==
   ===
   Instruções para entrar na lista, sair da lista e usar a lista em 
   http://www.mat.puc-rio.br/~nicolau/olimp/obm-l.html
   ==
   ===
   
  
  
  
 ==
  === Instruções para entrar na lista, sair da lista e usar a 
 lista em 
  http://www.mat.puc-rio.br/~nicolau/olimp/obm-l.html
  
 ==
  ===
  
 
 ==
 ===
 Instruções para entrar na lista, sair da lista e usar a lista 
 em http://www.mat.puc-rio.br/~nicolau/olimp/obm-l.html
 ==
 ===
 


=
Instruções para entrar na lista, sair da lista e usar a lista em
http://www.mat.puc-rio.br/~nicolau/olimp/obm-l.html
=


RES: [obm-l] Problema - Primos

2004-07-20 Por tôpico David M. Cardoso
 
 Mostre que um número com 30 dígitos não pode ter mais que 
 100 fatores primos.
 

Bem.. talvez eu tenha feito, acho que eh soh mostrar que
Piso[Log_10[2^100]+1] = 31
e que portanto 2^100, que é o menor produto de 100 fatores primos, tem 31
dígitos.

[]'s
David


=
Instruções para entrar na lista, sair da lista e usar a lista em
http://www.mat.puc-rio.br/~nicolau/olimp/obm-l.html
=


[obm-l] RES: [obm-l] Problema - Recorrência / Fibonacci

2004-07-20 Por tôpico David M. Cardoso

Entendi.. entendi.. obrigado.

[]'s

 -Mensagem original-
 De: [EMAIL PROTECTED] 
 [mailto:[EMAIL PROTECTED] Em nome de Domingos Jr.
 Enviada em: terça-feira, 20 de julho de 2004 23:44
 Para: [EMAIL PROTECTED]
 Assunto: Re: [obm-l] Problema - Recorrência / Fibonacci
 
 David M. Cardoso wrote:
 
 Olá novamente,
 
 Seja F_n a recorrência definida por F_(n+1) = F_n + F_(n-1).
 Com F_1 = 1, F_2 = 1, ... (sequencia de fibonacci)
 
 Qual é o maior: 2^100 ou F_100 ?
 
 deu pra perceber, testando, que 2^100 é maior.
 Ateh porque 2^(n+1) / 2^n = 2
 Enquanto que F_(n+1) / F_(n) ~ 1,618 quando n é grande.
 
 Mas não sei formalizar/mostrar que 2^100 é de fato o maior.
 
 Você pode provar o resultado por indução para todo n, veja:
 para n = 1, 2, F_n = 1  2^n
 
 F_{n+1} = F_n + F{n-1}  2^n + 2^{n-1} = 3*2^{n-1}  
 4*2^{n-1} = 2^{n+1}
 
 e o resultado segue por indução.
 ==
 ===
 Instruções para entrar na lista, sair da lista e usar a lista 
 em http://www.mat.puc-rio.br/~nicolau/olimp/obm-l.html
 ==
 ===
 


=
Instruções para entrar na lista, sair da lista e usar a lista em
http://www.mat.puc-rio.br/~nicolau/olimp/obm-l.html
=


Re: RES: [obm-l] Problema Subconjuntos

2004-07-20 Por tôpico Augusto Cesar de Oliveira Morgado
C(n-2;3). Basta usar o primeiro lema de Kaplansky.

==
Mensagem  enviada  pelo  CIP  WebMAIL  - Nova Geração - v. 2.1
CentroIn Internet Provider  http://www.centroin.com.br
Tel: (21) 2542-4849, (21) 2295-3331Fax: (21) 2295-2978
Empresa 100% Brasileira - Desde 1992 prestando servicos online


-- Original Message ---
From: David M. Cardoso [EMAIL PROTECTED]
To: [EMAIL PROTECTED]
Sent: Tue, 20 Jul 2004 20:57:24 -0300
Subject: RES: [obm-l] Problema Subconjuntos

 Cara, muito obrigado..
 Sendo que ta dando trabalho pra eu entender algumas coisas,
 como teremos T[n-3] - T[n-4] subconjuntos com os elementos n-1 e n-2..
 hora eu penso que entendi, hora eu não entendo mais e fico tentando lembrar
 pq eu fico entendido antes, talvez seja o nervosismo, talvez seja apenas
 porque o raciocinio eh complicado demais pra mim..
 
 Outra duvida que tenho é se é possível transformar a recorrência num
 polinomiozinho em função de n ou se uma resposta desse tipo já esta
 completa o suficiente..
 
 []'s
 David
 
  -Mensagem original-
  De: [EMAIL PROTECTED] 
  [mailto:[EMAIL PROTECTED] Em nome de Helder Suzuki
  Enviada em: terça-feira, 20 de julho de 2004 19:30
  Para: [EMAIL PROTECTED]
  Assunto: Re: [obm-l] Problema Subconjuntos
  
  vamos ver, seguindo a dica de usar recorrencia
  
  se T[n] for igual ao numero de subconjuntos do conjunto {1, 
  2, ..., n} que nao contem 3 inteiros consecutivos.
  temos que:
  T[0] = 1
  {}
  
  T[1] = 2
  {} e {1}
  
  T[2] = 4
  {}, {1},
  {2} e {1, 2}
  
  T[3] = 7
  {}, {1}, {2}, {1, 2},
  {3}, {1, 3}, {2, 3}
  
  T[4] = 13
  {}, {1}, {2}, {3}, {1, 2}, {1, 3}, {2, 3}, {4}, {1, 4}, {2, 
  4}, {3, 4}, {1, 2, 4}, {1, 3, 4}
  
  bom, suponha que sabemos o valor de T[n-1], T[n-2], ..., 
  T[1]; como podemos achar T[n] em funcao de T[n-1]? humm...
  considere todos subconjuntos de {1, 2, 3, 4, ..., n-1} que 
  satisfazem a condicao do enunciado.
  se adicionarmos um elemento n, em quais desses subconjuntos o 
  n pode entrar e quais ele nao pode(para manter a condicao do 
  enunciado)?
  se n nao pode entrar em X subconjuntos, temos que T[n] = 
  T[n-1] + T[n-1] - X T[n] = 2*T[n-1] - X mas X eh o numero de 
  subconjuntos que tem os elementos
  n-1 e n-2.
  
  imagine que temos os subconjnutos de {1, 2, ..., n-3} e 
  queremos adicionar os elementos n-1 e n-2 a esses 
  subconjuntos ao mesmo tempo, nesse caso só nao poderemos 
  adicionar n-1 e n-2 aos subconjuntos que tem o elemento n-3, 
  entao teremos T[n-3] - T[n-4] subconjuntos com os elementos n-1 e n-2:
  X = T[n-3] - T[n-4]
  
  entao nossa recorrencia fica:
  T[n] = 2*T[n-1] - T[n-3] + T[n-4]
  
  []'s,
  Helder
  
  --- David M. Cardoso [EMAIL PROTECTED]
  escreveu:  
   
   Olá,
   
   Alguem pode me ajudar? Não consegui resolver o seguinte problema:
   
   Quantos subconjuntos o conjunto {1,2,3,...,n} tais que não contêm 
   três inteiros consecutivos?
   
   A dica dada na questão é: Encontre uma recorrência. 
  Porém, qualquer 
   solução (sem/com recorrência) vai ajudar.
   
   []'s
   David
  
  
  
  
  
  ___
  Yahoo! Mail agora com 100MB, anti-spam e antivírus grátis!
  http://br.info.mail.yahoo.com/
  ==
  ===
  Instruções para entrar na lista, sair da lista e usar a lista 
  em http://www.mat.puc-rio.br/~nicolau/olimp/obm-l.html
  ==
  ===
 
 
 =
 Instruções para entrar na lista, sair da lista e usar a lista em
 http://www.mat.puc-rio.br/~nicolau/olimp/obm-l.html
 =
--- End of Original Message ---

=
Instruções para entrar na lista, sair da lista e usar a lista em
http://www.mat.puc-rio.br/~nicolau/olimp/obm-l.html
=


Re: RES: [obm-l] Problema Subconjuntos

2004-07-20 Por tôpico Helder Suzuki
Eh, eu fiz uma confusao ali

quote
imagine que temos os subconjnutos de {1, 2, ..., n-3}
e 
queremos adicionar os elementos n-1 e n-2 a esses
subconjuntos ao mesmo tempo, nesse caso só nao
poderemos adicionar n-1 e n-2 aos subconjuntos que tem
o elemento n-3, 
errado
entao teremos T[n-3] - T[n-4] subconjuntos com os
elementos n-1 e n-2: 
X = T[n-3] - T[n-4]
/errado 
/quote

correcao
T[n-3] - T[n-4] eh o numero de subconjuntos que tem o
elemento n-3. podemos adicionar n-1 e n-2 a todos os
outros subconjuntos, entao podemos adicionar n-1 e n-2
a T[n-3] - (T[n-3] - T[n-4]) = T[n-4]
entao X = T[n-4]
e T[n] = 2*T[n-1] - T[n-4]
/correcao

--- David M. Cardoso [EMAIL PROTECTED]
escreveu:  
 Cara, muito obrigado..
 Sendo que ta dando trabalho pra eu entender algumas
 coisas,
 como teremos T[n-3] - T[n-4] subconjuntos com os
 elementos n-1 e n-2..
 hora eu penso que entendi, hora eu não entendo mais
 e fico tentando lembrar
 pq eu fico entendido antes, talvez seja o
 nervosismo, talvez seja apenas
 porque o raciocinio eh complicado demais pra mim..
 
 Outra duvida que tenho é se é possível transformar a
 recorrência num
 polinomiozinho em função de n ou se uma resposta
 desse tipo já esta
 completa o suficiente..
 
 []'s
 David
 
  -Mensagem original-
  De: [EMAIL PROTECTED] 
  [mailto:[EMAIL PROTECTED] Em nome de
 Helder Suzuki
  Enviada em: terça-feira, 20 de julho de 2004 19:30
  Para: [EMAIL PROTECTED]
  Assunto: Re: [obm-l] Problema Subconjuntos
  
  vamos ver, seguindo a dica de usar recorrencia
  
  se T[n] for igual ao numero de subconjuntos do
 conjunto {1, 
  2, ..., n} que nao contem 3 inteiros consecutivos.
  temos que:
  T[0] = 1
  {}
  
  T[1] = 2
  {} e {1}
  
  T[2] = 4
  {}, {1},
  {2} e {1, 2}
  
  T[3] = 7
  {}, {1}, {2}, {1, 2},
  {3}, {1, 3}, {2, 3}
  
  T[4] = 13
  {}, {1}, {2}, {3}, {1, 2}, {1, 3}, {2, 3}, {4},
 {1, 4}, {2, 
  4}, {3, 4}, {1, 2, 4}, {1, 3, 4}
  
  bom, suponha que sabemos o valor de T[n-1],
 T[n-2], ..., 
  T[1]; como podemos achar T[n] em funcao de T[n-1]?
 humm...
  considere todos subconjuntos de {1, 2, 3, 4, ...,
 n-1} que 
  satisfazem a condicao do enunciado.
  se adicionarmos um elemento n, em quais desses
 subconjuntos o 
  n pode entrar e quais ele nao pode(para manter a
 condicao do 
  enunciado)?
  se n nao pode entrar em X subconjuntos, temos que
 T[n] = 
  T[n-1] + T[n-1] - X T[n] = 2*T[n-1] - X mas X eh o
 numero de 
  subconjuntos que tem os elementos
  n-1 e n-2.
  
  imagine que temos os subconjnutos de {1, 2, ...,
 n-3} e 
  queremos adicionar os elementos n-1 e n-2 a esses 
  subconjuntos ao mesmo tempo, nesse caso só nao
 poderemos 
  adicionar n-1 e n-2 aos subconjuntos que tem o
 elemento n-3, 
  entao teremos T[n-3] - T[n-4] subconjuntos com os
 elementos n-1 e n-2:
  X = T[n-3] - T[n-4]
  
  entao nossa recorrencia fica:
  T[n] = 2*T[n-1] - T[n-3] + T[n-4]
  
  []'s,
  Helder
  
  --- David M. Cardoso [EMAIL PROTECTED]
  escreveu:  
   
   Olá,
   
   Alguem pode me ajudar? Não consegui resolver o
 seguinte problema:
   
   Quantos subconjuntos o conjunto {1,2,3,...,n}
 tais que não contêm 
   três inteiros consecutivos?
   
   A dica dada na questão é: Encontre uma
 recorrência. 
  Porém, qualquer 
   solução (sem/com recorrência) vai ajudar.
   
   []'s
   David
  
  
  
  
  
 

___
  Yahoo! Mail agora com 100MB, anti-spam e antivírus
 grátis!
  http://br.info.mail.yahoo.com/
 

==
  ===
  Instruções para entrar na lista, sair da lista e
 usar a lista 
  em
 http://www.mat.puc-rio.br/~nicolau/olimp/obm-l.html
 

==
  ===
  
 
 

=
 Instruções para entrar na lista, sair da lista e
 usar a lista em
 http://www.mat.puc-rio.br/~nicolau/olimp/obm-l.html

=
 bm-l.html

=
  





___
Yahoo! Mail agora com 100MB, anti-spam e antivírus grátis!
http://br.info.mail.yahoo.com/
=
Instruções para entrar na lista, sair da lista e usar a lista em
http://www.mat.puc-rio.br/~nicolau/olimp/obm-l.html
=


RES: [obm-l] Problema envolvendo sistema linear

2004-06-15 Por tôpico Wellington
L = nº laranjas
P = nº peras
X = nº pessoas

Faça:
3L = P
5X = L
8X + 21 = P



-Mensagem original-
De: [EMAIL PROTECTED] [mailto:[EMAIL PROTECTED] Em
nome de Maurizio
Enviada em: Tuesday, June 15, 2004 7:17 PM
Para: [EMAIL PROTECTED]
Assunto: [obm-l] Problema envolvendo sistema linear

Em uma cesta de frutas, há 3 vezes mais peras doq ue laranjas. Eu e meus

amigos vamos dividir as frutas. Se cada um de nós receber 5 laranjas e 8

peras, restarão 21 peras, e as laranjas serão todas distribuídas. 
Quantas laranjas há na cesta? Quantas pessoas somos?

=
Instruções para entrar na lista, sair da lista e usar a lista em
http://www.mat.puc-rio.br/~nicolau/olimp/obm-l.html

=

---
Incoming mail is certified Virus Free.
Checked by AVG anti-virus system (http://www.grisoft.com).
Version: 6.0.693 / Virus Database: 454 - Release Date: 5/31/2004
 

---
Outgoing mail is certified Virus Free.
Checked by AVG anti-virus system (http://www.grisoft.com).
Version: 6.0.693 / Virus Database: 454 - Release Date: 5/31/2004
 


=
Instruções para entrar na lista, sair da lista e usar a lista em
http://www.mat.puc-rio.br/~nicolau/olimp/obm-l.html
=


RES: [obm-l] Problema envolvendo sistema linear

2004-06-15 Por tôpico David M. Cardoso

 
 L = nº laranjas
 P = nº peras
 X = nº pessoas
 
 Faça:
 3L = P
 5X = L
 8X + 21 = P
 

Serei a calculadora:

-3L = -P
8x + 21 = P

8x - 3L = -21
5X - L  = 0 (vezes -3)
-15x +3L = 0

-7X = -21 == X = 3
  == L = 15
  == P = 45


=
Instruções para entrar na lista, sair da lista e usar a lista em
http://www.mat.puc-rio.br/~nicolau/olimp/obm-l.html
=


RES: [obm-l] Problema de Torneiras

2004-03-30 Por tôpico David M. Cardoso

(1/4) + (x/25) + (x/40) - (x/20) = 1

[... contas ...]

x = 50 horas


Princípio:

A primeira enxe o tanque em 25 horas

25 -- 1


 -Mensagem original-
 De: [EMAIL PROTECTED] [mailto:[EMAIL PROTECTED] Em nome
 de Fabio Contreiras
 Enviada em: terça-feira, 30 de março de 2004 23:08
 Para: [EMAIL PROTECTED]
 Assunto: [obm-l] Problema de Torneiras
 
 Esse é maneiro! Alguem sabe o caminho das pedras?
 
 
  1 ) Um tanque tem 3 torneiras. A primeira enxe o tanque em 25 horas, a
 segunda em 40 horas, ja a terceira, o esvazia em 20 horas. O tanque está
 com 1 / 4 de água. Abrindo-se simultaneamente as três torneiras, ele
 ficará cheio em :
 
 


=
Instruções para entrar na lista, sair da lista e usar a lista em
http://www.mat.puc-rio.br/~nicolau/olimp/obm-l.html
=


RES: [obm-l] Problema de Torneiras

2004-03-30 Por tôpico David M. Cardoso

Apertei control+enter e enviei a mensagem sem querer (desculpa!),
continuando:

 (1/4) + (x/25) + (x/40) - (x/20) = 1
 
 [... contas ...]
 
 x = 50 horas
 
 
 Princípio:
 
 A primeira enCHe o tanque em 25 horas

25 horas - 1 tanque
x horas  - (x/25) tanque.


=
Instruções para entrar na lista, sair da lista e usar a lista em
http://www.mat.puc-rio.br/~nicolau/olimp/obm-l.html
=


RES: [obm-l] Problema estranho..

2004-03-25 Por tôpico Cloves Jr
Augurios,

Eu tb achava que naum tinha solucao da maneira que o professor passou... Eu
cheguei em uma solucao igual e dai resolvi colocar em discussao na lista pra
ver se alguem tinha alguma ideia diferente que talvez resolvese o
problema... A todos que ajudaram meu mto obrigado..

[]s

Cloves Jr

-Mensagem original-
De: [EMAIL PROTECTED] [mailto:[EMAIL PROTECTED]
nome de Angelo Barone Netto
Enviada em: quinta-feira, 25 de março de 2004 16:45
Para: [EMAIL PROTECTED]
Assunto: Re: [obm-l] Problema estranho..


Caro Cloves Jr [EMAIL PROTECTED]:

Claro que se a soma dos elementos de cada fila e 12 a soma dos
nove elementos da matriz e 36.
Por outro lado, se os nove elemntos sao naturais sua soma e, no minimo,
0+1+2+3+4+5+6+7+8=36, os naturais tem que incluir o zero (nada mais natural)
e sao necessariamente os que figuram na linha acima.
Existem poucas matrizes que satisfazem isto (calcule seu 3),
uma delas e
048
561
723.
Augurios.


Angelo Barone Netto [EMAIL PROTECTED]
=
Instruções para entrar na lista, sair da lista e usar a lista em
http://www.mat.puc-rio.br/~nicolau/olimp/obm-l.html
=

=
Instruções para entrar na lista, sair da lista e usar a lista em
http://www.mat.puc-rio.br/~nicolau/olimp/obm-l.html
=


RES: [obm-l] Problema 05

2003-02-11 Por tôpico Eduardo
Elton,
Tente, antes de generalizar um sistema, estipular um outro problema.
Por exemplo, se você tivesse feito a prova, a quantidade de erros e acertos
somadas seria o total de questões?
Se você tivesse acertado 32 questões, ganharia quantos pontos com isto
(somente as certas)?
Que conta você fez para obter o resultado?
Se tivesse acertado 32 teria errado 28, quantos pontos perderia com isto?
Para saber sua nota, que conta faria?

as vezes fica mais fácil partir de uma resposta...

Segue a resolução do seu problema 5

A=Acerto
E=erro

A+E=60
2A-E=30

Donde A= 30 e E=30

Abraços

Edu

-Mensagem original-
De: [EMAIL PROTECTED]
[mailto:[EMAIL PROTECTED]]Em nome de elton francisco
ferreira
Enviada em: terça-feira, 11 de fevereiro de 2003 20:03
Para: [EMAIL PROTECTED]
Assunto: [obm-l] Problema 05


Numa prova de matemática, um aluno deve responder a 60
itens do tipo verdadeiro ou falso. para cada item
respondido corretamente, o aluno vai ganhar 2 pontos
e, para cada item que errar, vai perder 1 ponto. A
nota do aluno é função do número de itens que ele
acertar. Se o aluno obteve 30 pontos, quantos itens
ele acertou?



___
Busca Yahoo!
O serviço de busca mais completo da Internet. O que você pensar o Yahoo!
encontra.
http://br.busca.yahoo.com/
=
Instruções para entrar na lista, sair da lista e usar a lista em
http://www.mat.puc-rio.br/~nicolau/olimp/obm-l.html
O administrador desta lista é [EMAIL PROTECTED]
=
---
Incoming mail is certified Virus Free.
Checked by AVG anti-virus system (http://www.grisoft.com).
Version: 6.0.449 / Virus Database: 251 - Release Date: 27/1/2003

---
Outgoing mail is certified Virus Free.
Checked by AVG anti-virus system (http://www.grisoft.com).
Version: 6.0.449 / Virus Database: 251 - Release Date: 27/1/2003

___
Busca Yahoo!
O serviço de busca mais completo da Internet. O que você pensar o Yahoo! encontra.
http://br.busca.yahoo.com/

=
Instruções para entrar na lista, sair da lista e usar a lista em
http://www.mat.puc-rio.br/~nicolau/olimp/obm-l.html
O administrador desta lista é [EMAIL PROTECTED]
=



RES: [obm-l] Problema 04

2003-02-10 Por tôpico Eduardo
Elton...faça um sistema

abraços

edu

-Mensagem original-
De: [EMAIL PROTECTED]
[mailto:[EMAIL PROTECTED]]Em nome de elton francisco
ferreira
Enviada em: segunda-feira, 10 de fevereiro de 2003 16:10
Para: [EMAIL PROTECTED]
Assunto: [obm-l] Problema 04


Um caixa automática de um banco só libera notas de R$
5,00 e R$ 10,00. Uma pessoa retirou dessa caixa a
importância de R$ 65,00, recebendo 10 notas. O produto
do número de notas de R$ 5,00 pelo número de notas de
R$ 10,00 é igual a

16
25
24
21


___
Busca Yahoo!
O melhor lugar para encontrar tudo o que você procura na Internet
http://br.busca.yahoo.com/
=
Instruções para entrar na lista, sair da lista e usar a lista em
http://www.mat.puc-rio.br/~nicolau/olimp/obm-l.html
O administrador desta lista é [EMAIL PROTECTED]
=
---
Incoming mail is certified Virus Free.
Checked by AVG anti-virus system (http://www.grisoft.com).
Version: 6.0.449 / Virus Database: 251 - Release Date: 27/1/2003

---
Outgoing mail is certified Virus Free.
Checked by AVG anti-virus system (http://www.grisoft.com).
Version: 6.0.449 / Virus Database: 251 - Release Date: 27/1/2003

___
Busca Yahoo!
O serviço de busca mais completo da Internet. O que você pensar o Yahoo! encontra.
http://br.busca.yahoo.com/

=
Instruções para entrar na lista, sair da lista e usar a lista em
http://www.mat.puc-rio.br/~nicolau/olimp/obm-l.html
O administrador desta lista é [EMAIL PROTECTED]
=



RES: [obm-l] Problema 02

2003-02-07 Por tôpico Eduardo
Verifique esta solução, ou pelo menos o início...

Bem, a posição do garoto quando a garota sobe na roda gigante e quando desce
é a mesma..., se a garota deu 20 voltas o garoto deu 20 voltas mais a
diferença inicial, de 6 cadeiras...o resto é conta.

-Mensagem original-
De: [EMAIL PROTECTED]
[mailto:[EMAIL PROTECTED]]Em nome de elton francisco
ferreira
Enviada em: sexta-feira, 7 de fevereiro de 2003 21:18
Para: [EMAIL PROTECTED]
Assunto: [obm-l] Problema 02

Um casal de namorados foi a um parque de diversões. A
roda-gigante tem 10 cadeiras e 8 m de raio. O garoto,
que foi o primeiro a entrar, sentou-se na cadeira
número 1. A garota sentou-se na cadeira número 7 e
desceu depois de dar 20 voltas completas. Quantos
metros o garoto percorreu, do instante em que subiu no
brinquedo até o momento em que a garota desceu?
pi: 3,14

___
Busca Yahoo!
O melhor lugar para encontrar tudo o que você procura na Internet
http://br.busca.yahoo.com/
=
Instruções para entrar na lista, sair da lista e usar a lista em
http://www.mat.puc-rio.br/~nicolau/olimp/obm-l.html
O administrador desta lista é [EMAIL PROTECTED]
=
---
Incoming mail is certified Virus Free.
Checked by AVG anti-virus system (http://www.grisoft.com).
Version: 6.0.449 / Virus Database: 251 - Release Date: 27/1/2003

---
Outgoing mail is certified Virus Free.
Checked by AVG anti-virus system (http://www.grisoft.com).
Version: 6.0.449 / Virus Database: 251 - Release Date: 27/1/2003


___
Busca Yahoo!
O serviço de busca mais completo da Internet. O que você pensar o Yahoo! encontra.
http://br.busca.yahoo.com/

=
Instruções para entrar na lista, sair da lista e usar a lista em
http://www.mat.puc-rio.br/~nicolau/olimp/obm-l.html
O administrador desta lista é [EMAIL PROTECTED]
=



RES: [obm-l] Problema de LOg

2002-10-28 Por tôpico Ralph Teixeira

Como resolver? Acho que não resolve. Para começar, tem um montão de
raízes

De fato, sempre que x varia de 2KPi a 2Kpi+pi, o seno vai de 0 a 1 e
volta para 0, portanto sen(lnx) vai de -Inf a 0 e volta para -Inf. Então o
gráfico de sen(lnx) vai ser um bando de oscilações de -Inf a 0 e de volta a
-Inf, uma oscilação a cada 2Pi.

Por outro lado, sen(lnx) oscila a períodos cada vez maiores
Quer dizer, quando x vai de 1 a e^2Pi, tem uma oscilcao do seno (de 0 a 1 a
0 a -1 e de volta a 0). A próxima oscilação vai de e^2Pi a e^4Pi...
Oscilação demoraada Quer dizer que há vários locais onde sen(lnx)
vai ficar horizontal... Em particular, deve ter um bom pedação onde
sen(lnx) é um tanto negativo...

... e nesses pedaços, sen(lnx) deve cortar ln(senx) várias vezes (já
que este oscila rápido e é periódico).

Faça os gráficos e confira... :)

Abraço,
Ralph

-Mensagem original-
De: Marcos Eike Tinen dos Santos [mailto:mesantos;uai.com.br]
Enviada em: quinta-feira, 20 de março de 2008 17:01
Para: [EMAIL PROTECTED]
Assunto: [obm-l] Problema de LOg


Como calcular sen(log x) = log(sen x)

Ats,
Marcos Eike

=
Instruções para entrar na lista, sair da lista e usar a lista em
http://www.mat.puc-rio.br/~nicolau/olimp/obm-l.html
O administrador desta lista é [EMAIL PROTECTED]
=
=
Instruções para entrar na lista, sair da lista e usar a lista em
http://www.mat.puc-rio.br/~nicolau/olimp/obm-l.html
O administrador desta lista é [EMAIL PROTECTED]
=



RES: [obm-l] Problema

2002-05-22 Por tôpico Ralph Teixeira

resolva a equação :

x^(sqrt x) = 1/2

Deixa eu ver... Note que temos de ter x0. Então, vou fazer y=1/sqrt(x),
isto é,
x=1/y^2 para começar. Note que y0 também.

(1/y^2)^(1/y)=1/2
y^2^(-1/y)=1/2
y^2=2^y

Ah-ha! Esse problema eu já vi por aqui Se eu me lembro bem, a gente tem
três soluções: y=2, y=4 e uma outra solução negativa que usava o
Lambertiano (eu já escrevi isso aqui antes). A solução negativa neste caso
não presta, então ficamos só com x=1/4 ou x=1/16.

Abraço,
Ralph
=
Instruções para entrar na lista, sair da lista e usar a lista em
http://www.mat.puc-rio.br/~nicolau/olimp/obm-l.html
O administrador desta lista é [EMAIL PROTECTED]
=



[obm-l] RES: [obm-l] Problema de Função...

2002-04-14 Por tôpico Guilherme Pimentel



Como 
eu respondi no forum do só matematica, resposta é 20 com
m=5, 
x1=2 e x2=4

[]'s 
Guilherme Pimentel
http://sites.uol.com.br/guigous


  -Mensagem original-De: 
  [EMAIL PROTECTED] 
  [mailto:[EMAIL PROTECTED]]Em nome de Igor 
  CastroEnviada em: sábado, 13 de abril de 2002 16:20Para: 
  [EMAIL PROTECTED]Assunto: [obm-l] Problema de 
  Função...
  Olá colegas da lista,
  gostaria de ajuda neste problema por que na 
  minha resolução acho sempre 64... mas não está nas opções do problema, creio 
  que seja facil. Lá vai:
  Seja A um conjunto de numeros reais tais que para 
  toda m pertencente a A a função: (m/2 -2).x^2 -mx + 8, terá sempre duas 
  raizes reais x1 e x2 satisfazendo
  0 x1 = x2  5 . Tomando K como o menor 
  numero inteiro pertencente a A e fazendo na função m=K, as raizes obtidas x1 e 
  x2 tem por soma de seus quadrados:
  a) 20 b)25 
  c)30 d)41 e)49
  
  agradeço desde já...
  []'s
  ps: = significa maior ou 
igual...


[obm-l] Re:[obm-l] RES: [obm-l] Problema de Função...

2002-04-14 Por tôpico adr.scr.m

Não seria m=6 no lugar de m=5?
 Adriano.
   
 Como eu respondi no forum do só matematica, resposta é 
20 com
 m=5, x1=2 e x2=4
 
 []'s Guilherme Pimentel
 http://sites.uol.com.br/guigous
 
   -Mensagem original-
   De: [EMAIL PROTECTED]
 [mailto:[EMAIL PROTECTED]]Em nome de 
Igor Castro
   Enviada em: sábado, 13 de abril de 2002 16:20
   Para: [EMAIL PROTECTED]
   Assunto: [obm-l] Problema de Função...
 
 
   Olá colegas da lista,
gostaria de ajuda neste problema por que na minha 
resolução acho sempre
 64... mas não está nas opções do problema, creio que 
seja facil. Lá vai:
   Seja A um conjunto de numeros reais tais que para 
toda m pertencente a A a
 função: (m/2 -2).x^2  -mx + 8, terá sempre duas raizes 
reais x1 e x2
 satisfazendo
   0 x1 = x2  5 . Tomando K como o menor numero 
inteiro pertencente a A e
 fazendo na função m=K, as raizes obtidas x1 e x2 tem 
por soma de seus
 quadrados:
   a) 20b)25   c)30   d)41  e)49
 
   agradeço desde já...
   []'s
   ps: = significa maior ou igual...
 

 
__
Quer ter seu próprio endereço na Internet?
Garanta já o seu e ainda ganhe cinco e-mails personalizados.
DomíniosBOL - http://dominios.bol.com.br


=
Instruções para entrar na lista, sair da lista e usar a lista em
http://www.mat.puc-rio.br/~nicolau/olimp/obm-l.html
O administrador desta lista é [EMAIL PROTECTED]
=



[obm-l] RES: [obm-l] Re:[obm-l] RES: [obm-l] Problema de Função...

2002-04-14 Por tôpico Guilherme Pimentel

é m=6, escrevi errado.

-Mensagem original-
De: [EMAIL PROTECTED]
[mailto:[EMAIL PROTECTED]]Em nome de adr.scr.m
Enviada em: domingo, 14 de abril de 2002 11:50
Para: [EMAIL PROTECTED]
Assunto: [obm-l] Re:[obm-l] RES: [obm-l] Problema de Função...


Não seria m=6 no lugar de m=5?
 Adriano.

 Como eu respondi no forum do só matematica, resposta é
20 com
 m=5, x1=2 e x2=4

 []'s Guilherme Pimentel
 http://sites.uol.com.br/guigous

   -Mensagem original-
   De: [EMAIL PROTECTED]
 [mailto:[EMAIL PROTECTED]]Em nome de
Igor Castro
   Enviada em: sábado, 13 de abril de 2002 16:20
   Para: [EMAIL PROTECTED]
   Assunto: [obm-l] Problema de Função...


   Olá colegas da lista,
gostaria de ajuda neste problema por que na minha
resolução acho sempre
 64... mas não está nas opções do problema, creio que
seja facil. Lá vai:
   Seja A um conjunto de numeros reais tais que para
toda m pertencente a A a
 função: (m/2 -2).x^2  -mx + 8, terá sempre duas raizes
reais x1 e x2
 satisfazendo
   0 x1 = x2  5 . Tomando K como o menor numero
inteiro pertencente a A e
 fazendo na função m=K, as raizes obtidas x1 e x2 tem
por soma de seus
 quadrados:
   a) 20b)25   c)30   d)41  e)49

   agradeço desde já...
   []'s
   ps: = significa maior ou igual...



__
Quer ter seu próprio endereço na Internet?
Garanta já o seu e ainda ganhe cinco e-mails personalizados.
DomíniosBOL - http://dominios.bol.com.br


=
Instruções para entrar na lista, sair da lista e usar a lista em
http://www.mat.puc-rio.br/~nicolau/olimp/obm-l.html
O administrador desta lista é [EMAIL PROTECTED]
=

=
Instruções para entrar na lista, sair da lista e usar a lista em
http://www.mat.puc-rio.br/~nicolau/olimp/obm-l.html
O administrador desta lista é [EMAIL PROTECTED]
=